Path Quizzes and Docs

Download as pdf or txt
Download as pdf or txt
You are on page 1of 248

Pathology Block 1 Quiz

Question 1
A 45-year-old African American female presents to her primary care physician complaining of
Correct
fatigue. She reports a six-month history of gradually worsening malaise; cough, weight loss; and
Mark 1.00 out pain in her legs and ankles. Her past medical history is significant for diabetes. She drinks socially
of 1.00
and has a 10 pack-year smoking history. Physical examination reveals lymphadenopathy and
hepatosplenomegaly. A lymph node biopsy is performed and results are shown in Figure below. The
process shown is maintained by which of the following cytokines?

Select one:
a. IL-2

b. IL-10

c. IL-6

d. IL-1

e. INF- γ !

Your answer is correct.


The correct answer is: INF- γ
Correct
Marks for this submission: 1.00/1.00.
Question 2
A 27-year-old woman presents to a physician because her periods have diminished and then
Correct
ceased over the last 3 years, and she has not been able to conceive. Physical examination
Mark 1.00 out demonstrates a mildly obese woman with increased hair on her upper lip and chin, around her
of 1.00
nipples, and along the linea alba of the abdomen. Which of the following changes will be most likely
seen in this patient?

Select one:
a. Proliferation of the endometrial glands relative to the stroma !

b. Decreased androgen release from ovarian theca cells

c. Increased FSH : LH ratio

d. Hyperplastic glands are lined by two cell layers

e. Endometrial gland lined by columnar cells with sub nuclear vacuole.

Your answer is correct.


The correct answer is: Proliferation of the endometrial glands relative to the stroma
Correct
Marks for this submission: 1.00/1.00.

Question 3
Parents of a 7 years old boy are worried about recurrent upper respiratory infections of their son for
Correct
the past 4 years. Molecular studies confirmed the diagnosis of Leukocyte Adhesion Defect 2 (LAD
Mark 1.00 out 2). Which one of the following adhesion molecules predominantly fail to function normally in this
of 1.00
child?

Select one:
a. P- Selectin

b. Β1 integrin

c. L – Selectin

d. Β2 integrin

e. E –Selectin !

Your answer is correct.


The correct answer is: E –Selectin
Correct
Marks for this submission: 1.00/1.00.
Question 4
A 60-year-old woman with history of type I diabetes currently on hormone replacement therapy is
Correct
seen in your ED complaining of "bad indigestion", dizziness and nausea for the past several hours.
Mark 1.00 out Vitals are Temperature-98.9, HR- 102, BP- 130/100, RR- 25 and she is mildly diaphoretic. An EKG is
of 1.00
shown in Figure A. At what time point does the injury to her affected cells become irreversible?

Select one:
a. 5 seconds

b. 60 seconds

c. 10 Minutes

d. 30 Minutes !

e. 1 hour

Your answer is correct.


The correct answer is: 30 Minutes
Correct
Marks for this submission: 1.00/1.00.
Question 5
Following a road traffic accident, a 34 years old male has a bruise on his elbow. The skin color at
Correct
the site of injury was red for two days followed by yellow-brown discoloration in the next 7-10 days.
Mark 1.00 out Which of the following is most appropriate about the substance that has most likely accumulated at
of 1.00
the site of injury?

Select one:
a. Glycogen stained blue with Prussian blue stain

b. Hemosiderin stained salmon pink with Congo red stain

c. Melanin stained blue with Prussian blue stain

d. Hemosiderin stained blue with Prussian blue stain !

e. Lipofuscin stained blue with Prussian blue stain

Your answer is correct.


The correct answer is: Hemosiderin stained blue with Prussian blue stain
Correct
Marks for this submission: 1.00/1.00.
Question 6
A 76 years old male suffering from rheumatoid arthritis died with a heart attack. Following an
Correct
autopsy, tissue samples are taken from different organs to confirm the cause of death. Sections
Mark 1.00 out from muscle tissue show characteristic findings (details given) when a special stain was used and
of 1.00
studied under polarized light. Which of the following stains used in this case?

Select one:
a. Prussian blue

b. Oil red ‘O’

c. Sudan IV

d. PAS

e. Congo red !

Your answer is correct.


The correct answer is: Congo red
Correct
Marks for this submission: 1.00/1.00.
Question 7
A 50-year-old female comes to the emergency department with sudden onset left-sided calf pain
Correct
starting 20 minutes ago. Her past medical history is notable for diabetes, hypertension, and
Mark 1.00 out chronic atrial fibrillation controlled on an anti-arrhythmic. Upon arrival to the emergency
of 1.00
department, her blood pressure is 150/91 mmHg, pulse is 100/min, and respirations of 22/min.
Examination of the leg reveals a pale left calf with diminished dorsalis pedis and posterior tibial
pulses. Prompt intervention relieves the patient's symptoms. Which of the following, if seen
microscopically, would indicate that her calf myocytes are more likely to recover without permanent
damage?

Select one:
a. Plasma membrane damage

b. Cellular swelling !

c. Fragmented cells and debris

d. Nuclear pyknosis

e. Mitochondrial vacuolization

Your answer is correct.


The correct answer is: Cellular swelling
Correct
Marks for this submission: 1.00/1.00.

Question 8
A 12 years old child sustained a sharp cut by a knife 12 hours ago and complains of a lot of pain at
Correct
the site of injury. On physical examination, the skin at and around the site of the cut is swollen,
Mark 1.00 out erythematous. The pain is the result of which one of the following chemical mediators?
of 1.00

Select one:
a. Prostaglandin !

b. Thromboxane

c. Histamine

d. Leukotriene

e. Complement C3b

Your answer is correct.


The correct answer is: Prostaglandin
Correct
Marks for this submission: 1.00/1.00.
Question 9
A 35-year-old man returns to the clinic to follow up for his chronic stomach pain. At the last visit a
Correct
few months ago, he explained that he had been experiencing discomfort in his upper abdomen for a
Mark 1.00 out while. He had never vomited up any blood and had not had any substantial weight loss. He did not
of 1.00
take any medications; did not smoke; and had no family history of gastric cancer. At that time, the
doctor empirically started him on a proton pump inhibitor (PPI). Today; despite the PPI; the patient
says he is still experiencing discomfort. Hearing this; the doctor decides to order a urease breath
test, which turns out to be positive . Which of the following pathological changes will be most likely
seen in the stomach of this patient?

Select one:
a.
Hypertrophy of smooth muscle

b.
Columnar cells with Goblet cells !

c.
Columnar to squamous metaplasia

d.
Mucous secreting foveolar cell hyperplasia

e.
Columnar cells with parietal cell hyperplasia

Your answer is correct.


The correct answer is:
Columnar cells with Goblet cells
Correct
Marks for this submission: 1.00/1.00.
Question 10
A 45 years old chronic alcoholic male presents with dragging sensation of right upper abdomen
Correct
and loss of appetite for the past three months. Further evaluation including liver biopsy is
Mark 1.00 out performed. Histopathologic findings of liver biopsy are shown. Which one of the following
of 1.00
substances most likely accumulated in the liver?

Select one:
a. Glycogen

b. Cholesterol

c. Alpha-1 Antitrypsin

d. Amyloid

e. Triglycerides !

Your answer is correct.


The correct answer is: Triglycerides
Correct
Marks for this submission: 1.00/1.00.

◀︎ Wound healing Table Jump to... Neoplasia Part 1. ▶︎


accomolationin
Acute Inflammation Vascularevents Edemaxoncoticp
Hydrop
via
Ecmointerstitialspace
either No
Histamine
robortumorCalor Dover cossoeronation CellulareventsRollingadhesionTransmigration
1. The sequence of events in the normal course following Cell injury àInflammation à Repair
2. Complex host response to injurious agents
3. Purpose of Acute InflammationàMainly protective but can be harmful (like Diseases -
Rheumatoid arthritis (RA), Atherosclerosis (AS), type 2 DM, Alzheimer's, and cancer) chronic
4. The main components are Vascular events, Cellular events
5. Cardinal signs of inflammation 1. Rubor- redness, 2. Tumor- swelling, 3. Calor- heat, 4. Dolor –
5
pain, 5. Loss of function
6. Pusis a Purulent inflammatory exudate rich in leukocytes (Neutrophils – pus cells), cellular
debris, microbes
7. Edema is the hallmark of Vascular events of Acute Inflammation, Exudate (Sp. gr. – >1.020); and
excess fluid in the interstitial or serous cavities
8. Know the stimuli of Acute Inflammation 6stimuli
9. Vascular Reactions - mediated by histamine, nitric oxide; Edema is hallmark; MCmechanism of
vascular leakage is EC gaps in Venules (mediated by histamine); in Severe burns or lytic bacterial
infections vascular leakage byDirect EC injury; Increased transcytosis is by VEGF
10. Cellular Reactionslumen of the vessel – 1. Margination, 2. Rolling-by Selectins; 3. Adhesionby
Integrins ( Leukocytes)& Ig superfamily (on EC), 4. Pavmenting, 5.Transmigration across the
endothelium or diapedesis – by PECAM/CD31
11. Adhesion molecules – for cell-cell & cell-ECM adhesions; 1.Selectins ,2. Immunoglobulin
superfamily - (VCAM-1à for β1containing s)& (ICAM-1à forβ2containing) on EC, on Platelets,
EC & leukocytes,3. Integrins- on the leukocytes{VLA-4 (β1- containing) and LFA-1, Mac-1 (β2-
containing)} , 4. Mucin -like glycoproteins - ligands for CD44responsible for retaining
leukocytesat the site of injury
12. Type of leukocyte response in Neutrophils à during the up to 24 hours, Mononuclear cells
(Lympho, Mono, Plasma cells) from 2nd day (Exceptions - Eosinophils in hypersensitivity
reactions, the prolonged Neutrophil response in Pseudomonas infection, lymphocytes in Viral
and TB
13. Cellular Reactions outside the lumen
a. Chemotaxis - Chemo attractants- MC exogenous are bacterial products; Endogenous-
C5a, LTB4IL-8(bind with to Leukocyte seven-transmembrane G-protein-coupled
receptors)
b. Steps in Phagocytosis – 1. Recognition and attachment – by Mannose & scavenger
receptors on leukocytes; 2. Engulfmentwith extensions pseudopods; 3.Killing or
degradation in phagolysosome – by 1.Oxygen-dependent mechanisms (MC); 2. Oxygen-
independent mechanisms –by BPI for GN bacteria, Lysozyme,Lactoferrin, Major basic
protein (MBP) of Eosinophils, Defensins, enzymes- elastase
14. Defects in Leukocyte Functions - the table is very high yield (slide – 23 of PPt -3)
15. Chemical Mediators of Inflammation- (slide –3 is very high yield);
a. Histamine richest in mast cells, Stimuli for histamine release ànumerous (histamine-
releasing proteins (HRP) of WBC, Cytokines, Trauma, Anaphylotoxins, substance P, etc.);
Effects -Dilation of arterioles, constriction of large arteries and Increase vascular
(venular) permeability by inducing EC gaps;
b. Serotonin has similar actions to histamine and the Sourcesare PLTS, entero-chromaffin
cells
c. AA metabolites – released frommembrane phospholipids by action of phospholipase A2
(PLA2)
i. Prostaglandins {PGs which include Thromboxanes (TXs)} by the action of
cyclooxygenases; PGE2àHyperalgesia (↑↑pain) like Bradykinin; PGE2, D2à
Vasodilation & edema, Fever; COX-1 important in inflammation & hemostasis
(PGs, TXs);COX-2 important only inflammation (by PGs); Imbalance of PGI2,
TXA2àthrombi formation in CVS & CNS (MI & Stroke) pots
ii. LPX ( Lipoxygenase ) pathway Tim imbalance
aggptaetgeaq.to
Thrombi
1. 5-LPX ( in PMNs)àproduce LTs ( LT B4 important); VC, Bronchospasm,
↑vascular permeability;Vasodilation 5HETE Chemotaxis
2. 12-LPX (in PLTS) àendogenous negative regulators of Leukotriene
action, inhibit PMN chemotaxis
d. ComplementsàVascular effects;
aux
i. Anaphylotoxins = C3a, C5a(also C4a);
ii. Leukocyte adhesion, chemotaxis, and activation =C5a;
iii. Phagocytosis =C3b and iC3b (inactive C3b);
iv. DAF (Decay-accelerating factor ) = decays C3 & C5 convertases;
v. CINH = inhibits C1 binding; if defective àHereditary Angioneurotic Edema
vi. CD59/ MAC complex inhibitor (membrane inhibitor of reactive lysis) àdefective
formation lead to Neisseriainfections
vii. Defective C3 à↑risk of fatal infections
viii. Defective C2 & C4 result in Autoimmune diseases
8
ix. 9Paroxysmal nocturnal hemoglobinuria (PNH) àMutations in the genes finally
cause defective DAF and CD59 (uncontrolled complement activation)
I
e. Kinin pathway - Important product isBradykinin; Increases vascular permeability;
Contraction of smooth muscle; Dilation of blood vessels; Pain when injected into the
skin(like PGE2)
I I
f. Clotting pathway àThrombi + PAR interaction is most important which result in ;
1. Mobilization of P-selectin; 2. Production of chemokines, 3. Expression of ICAM-1,
VAM-1; 4. COX-2 activation; 5. Production of PGs; 6. Production of PAF and NO;7. EC
shape change mobilizationPselectinexpr.oflCAMVAM
1 1 ECShapechange
g. PAF - VC (VD in very low conc.), Broncho-constriction, ↑vascular permeability (edema);
Important role in Necrotising Enterocolitis (disease of preterm babies put on artificial
feed)
h. Chemokines -Most important ones= CXCR-4, CCR-5 (also act as co-receptors for HIV-1
virus)
i. NO – cGMP pathway (cGMP has a role in ocular function – phototransduction,
Apoptosis; effects are 1. VD, ↓PLTS adhesion & aggression, ↓mast cell degranulation;2.
↑NO production à ↓replication of pathogens (microbicidal)
CHRONIC INFLAMMATION
Def: inflammation of prolonged duration (weeks à months à years) in which:
- Active inflammation, tissue destruction and attempts at repair proceed simultaneously
- Often results from persistence of an injury-causing agent
- Predominant cell type is mononuclear à macrophages, lymphocytes, plasma cells

Characteristics: Differs from acute inflammation by: 3Histological ofcar.I


features
-tissue destruction byproduct of inflammatory cells • Mononuclear cell filtrate
notions iTissue
destruction

-infiltration w/ mononuclear cells • Induration of affected tissue notsott aFibrosis


-replacement of damaged tissue by c.t. • Less swelling/less exudates s.cnr.int
cells
• Less hyperemia (redness)

CAUSES 4
1. Following acute inflammation Persistence of injury
2. Infections w/ certain organisms i Viralefongal Viral; tuberculosis; syphilis (treponema pallidum); fungal
(histoplasma); parasitic
3. Prolonged exposure to toxic agents Silica (à silicosis), lipids (à atherosclerosis), silicone
(breast implants)
4. Autoimmune dz Rheumatoid arthritis, SLE (lupus)

Macrophages belong to the MONONUCLEAR PHAGOCYTE system


Two types Fixed Kupffer cells (liver), sinus histiocytes (spleen & lymph node),
osteoclasts (bone)
Wandering
C Microglia (CNS), alveolar macrophages/dust cells (lungs)
During inflammation macrophages are recruited from blood (circulating monocytes) e
Chemotactic factor: MCP-1 Menocyte
Cytokines active macrophages Limicpy Macrophage em
Activated Activation signal: IFN-gamma by T-cells, bacterial endotoxins
macrophages -greater ability to kill ingested organisms SC
epitheloidcells
-larger size after activation
-increased lysosomal enzymes, more active metabolisms
-Micro: bigger, flatter, pink (H&E), called epitheloid cells b/c similar in appearance to
squamous cells

KEY CONCEPT: Tissue macrophages are derived from blood monocytes!

CELLS OF CHRONIC INFLAMMATION

MACROPHAGES – primary cell in chronic inflammation!


Functions:
1.phagocytosis, scavenger cells
2. receptor for IgG + C3b à opsonization
3. microbicidal activity (no O2 dependent – MPO system, have superoxide FR)
4.secrete NO à vasodilation
5. process antigens (act as antigen presenting cells [APC]) for T-cells
6. secrete cytokines: IL-1, TNF
7. secrete growth factors, like PDGF
8. have proteolytic enzymes

LYMPHOCYTES
Recall: lymphocytes primary inflammatory cells in certain infections (see Outcomes of Acute Inflammation)
Two types
T-cells Activated B cells à plasma cells
Two types: CD4 (helper T-cells), CD8 (cytotoxic T cells)
-interact w/ macrophages in chronic inflammation Functions:
-PROCESS: activated by ag presented on macrophage -produces antibodies (immunoglobulins)
à activated lymphocytes produce IFN-gamma which in -IgM: produced on 1st exposure in acute inflammation
turn activates macrophages à activated macrophages -IgG: main Ig of chronic inflammation, synthesized
produce IL-1 & TNF à activates lymphocytes & other after 10-14 days
cells
OTHER CELLS (act in either acute or chronic inflammation)
EOSINOPHILS BASOPHILSiintheBlood Histamine
-imp in à parasitic (invasive helminthes) & IgE -when IgE binds, release histamine Careasor hignexpskiniongs
mediated allergic rxns -basophils in blood, mast cells in tissue
-chemotactic/chemokine agent à eotaxin -present in high #s in areas where exposure to certain
-eosinophil granules contain major basic protein antigens is more common (lungs, skin)
(MBP), a charged cationic protein that is toxic to -plays imp role in IgE mediated rxns (allergies &
parasites and causes epithelial cell lysis/necrosis anaphylaxis)

Patterns of chronic inflammation


Chronic nonspecific inflammation Granulomatous inflammation
Def: specialized form of C.I. chr by formation of granulomas
Granulomas: localized collections of epitheloid cells (modified
macrophages)
• Caseating granulomas – if necrosis present
• Non-caseating granulomas

Epitheloid cells
• IFN-gamma Δ macrophages à
Chr by: epitheloid cells (hallmark of
-infiltration of mononuclear cells granulomas)
-inflammatory response not specific for any Multinucleated giant cells
particular etiological agent (e.g. irritation) • Mediated by IFN-gamma à fusion of
Composition of epitheloid cells
granulomas • In TB: Langhans type giant cells
• Not necessary to be granuloma
Lymphocytes, plasma cells
Central caseous necrosis
• Seen in granulomas due to TB, fungal
i
infections
• Rare in other granulomatous dz

Outcome of chronic inflammation


Development of 2o amyloidosis Chronic inflammation à repair by fibrosis
Due to production of serum associated amyloid (SAA) by Fibrosis can result in:
liver Adhesions: intestinal obstruction by fibrous bands
LiverIGAD Tamyloidosis CA: induced by scarring Induction cancer
Stricture/narrowing formation (ofof
lumen)scar
1 lumen
narrowed

Lymphatic system in inflammation


-filters extravascular/interstitial fluid channels Lymphatics Nodest Lymph nodes
-Presents a secondary line of defense to -helps drain edema fluid during Lymphadenitis: inflamm of lymph
clear injurious areas inflamm nodes
Lymphangitis: inflammation of Lymphadenopathy: enlargement of
lymphatics lymph nodes
Bacteremia occurs when lymphatics
fail to filter microbes which eventually
make into the general circulation
REPAIR + WOUND HEALING

Repair involves two processes:


Regeneration Fibrosis
Definition: replacement of dead cells by cells of the same type
Depends on cells ability to replicate:
a/k/a continuously dividing cells
continuously lost + replaced by maturation from stem
cells + by proliferation of mature cells Definition: replacement of damaged
Labile cells
e.g. bone marrow, surface epithelia tissue by connective tissue
-readily regenerate after injury as long as pool of stem
cells is preserved Occurs if:
Quiescent (in G0 stage – cell cycle) -tissue architecture is destroyed
Stable cells
Parenchyma of solid tissues à liver, kidney, pancreas,
endothelial cells, fibroblasts, SMC
I
-basement membrane not intact
*e.g. 3rd degree burn, when deeper
Limited capacity to regenerated (EXCEPTION: liver) tissue affected
Terminally differentiated + nonproliferative in postnatal
life, do not divide
Permanent cells Injury to tissues irreversible à repair dominated by scar
formation
e.g. cardiac muscle, skeletal muscle, neurons

Steps involved in repair by connective tissue

1.Neutrophil transmigration To liquefy injured tissue


2.Macrophage
To remove debris
transmigration
Characteristic of healing à seen beneath scab
Specialized type of newly formed connective tissue
3.Granulation tissue
Gross: pink, soft, granular appearance, ↑↑ vascularized
formation
Chr by: angiogenesis, fibroblasts à produce collagen (Type III)
Accumulates Δ à dense fibrotic scar
Collagen is the major component of fibrous tissue, formed by fibroblasts
-triple helix of cross-linked alpha chains à ↑ tensile strength
-lysyl oxidase – enzyme imp for X linking, hydroxylation of collagen requires vit C
4.Initial production of type (prolyl hydroxylase), cross linking requires Cu2+
III collagen Type Ibone Most common form, >> tensile strength >> type III Fibrotic scar
Type II Found in cartilage
Type III Immature form found in granulation tissue
Type IV Found in basement membranes
Granulation tissue produces a dense scar that must be remodeled
Remodeling à ↑ tensile strength of scar tissue
5.Remodeling of scar tissue
Collagenases a/k/a Metalloproteinases – called as such b/c it requires Zn – replace
regzinc type III collagen with type I à ↑ tensile strength to approx. 80% of original

EXAMPLE OF WOUND HEALING: Cutaneous wound healing


First intention (primary union)
Occurs with clean, uninfected woulds when there is little tissue damage and wound edges are closely approximated
Regeneration predominates >> fibrosis
e.g. surgical incision

Immediately -Incision fills with clotted blood (hematoma/fibrin clot) à Scab covers wound

-Neutrophils begin to accumulate at margin


1st day (w/in 24 hrs) -Epidermal cells begin proliferation/mitosis at the margin à depositing basement membrane
components as they progress
-Epithelial cells meet in the midline beneath the surface scab, yielding a thin but continuous
2nd day epithelial layer à seal off wound
-Neutrophils largely replaced by macrophages
-granulation tissue forms, progressively invades incision space, laying down type III collagen
3rd day -angiogenesis begins due to BFGF

-neovascularization reaches its peak as granulation tissue fills incisional space


th th -collagen fibrils abundant à bridge incision
4 – 6 days
-epidermis recovers its normal thickness as differentiation of surface cells à mature epidermal
architecture w/ surface keraitinization
-collagen (type III) compresses blood vessels in fibrous tissue à ↓ blood flow
Week 2 -tensile strength ~10% of normal
-leukocyte infiltrate, edema, and ↑ vascularity diminish
-collagenase (imp for remodeling) converts type III à type I collagen to ↑ tensile strength, which
increases over time to, 80% w/in 3 months
Weeks to months
-scar comprised of cellular c.t. devoid of adnexal structures (hair, sweat glands) and inflammatory
cells
Key chemical mediator
Derived from: macrophages, fibroblasts, endothelial cells
Chemotactic to:
Fibronectin
• Fibroblasts to produce collagen
• Endothelial cells to stimulate angiogenesis
-opsonizing and adhesion agent
Second intention (secondary union)
Occurs in wounds that have large tissue defects, >> tissue loss (two skin edges not in contact)
e.g. infarcts, ulcers, abscesses
Differs from 1st intention by:
1. Inflammatory response is more intense b/c large tissue defects have greater volume of necrotic debris,
exudates & fibrin that must be removed; injury by inflammation greater here
2. Large [granulation tissue]: large defects require greater volume of granulation tissue to fill gaps and provide
underlying framework for regrowth of tissue epithelium à greater mass of scar tissue
3. Large clot of scab rich in fibrin and fibronectin forms at surface of wound
4. Secondary healing involves wound contraction
* myofibroblasts: modified fibroblasts w/ features of smooth muscle cells
* large defects may be reduced to 5-10% of original size

Wound Strength: carefully sutured skin wounds


1 week old Completely healed
• 10% of unwounded skin (type III collagen) • 70-80% of unwounded skin
• Rapid ↑ in tensile strength as scar tissue
accumulates over 2 months (type III Δ à type I)
IMPORTANT!: scar tissue is never as strong as the original tissue!!

Factors that impair healing


Single most important cause of delayed healing à persistent injury + inflammation
INFECTION
Most common pathogen: staphylococcus
FOREIGN MATERIAL Fragments of steel, glass, and even bone impede healing
-wounds in vascularized areas (face) heal faster >> poorly vascularized areas (tendon, feet)
-small wounds >> large wounds
SIZE, LOCATION, & TYPE
-incisions >> blunt trauma (contusion)
OF WOUND
-labile, stable >> permanent à inevitably result in scarring, ↓function
-inflammation arising in tissue spaces (pleural, peritoneal) à develops extensive exudates
-protein deficiency
-vitamin deficiency à vit. C leads to weak collagen
-trace elements
NUTRITION
* ↓ Zn = unable to replace type III collagen metal rea
proteinases
zinc
* ↓ Cu = unable to cross-link alpha chains (to form triple helix) in collagen, lysyl oxidase
affected
Diabetes mellitus = prototypical e.g, susceptibility to infection caused by:
METABOLIC STATUS • Impaired circulation
• ↑ glucose à bacteria thrive on this
Impaired healing due to inadequate blood flow
CIRCULATORY STATUS
e.g. atherosclerosis, vascular defects
Inhibit collagen synthesis, decrease tensile strength à prevents fibrosis
GLUCOCORTICOIDS + à anti-inflammatory response desirable in corneal infections b/c it ↓ opacity from collagen
deposition

Complications of wound healing


Leads to wound dehiscence – rupture of wound
Deficient scar formation
Most common in: after abdominal surgery; exacerbated by coughing, vomitting
Keloids Exuberant granulation (proud flesh)
Def: tumor-like, raised scar caused by
excessive synthesis of type III collagen -Excessive granulation tissue that protrudes
Excessive formation of Genetic à common in blacks above surrounding skin
repair components Affects: earlobes, face, neck, sternum -Prevents re-epithelization thereby delaying
Micro: irregular thick collagen bundles, wound healing
extend beyond original injury, no vascular -tx: remove excess tissue
or adnexal elements
Exaggerated contraction due to:
Contracture • Excessive scar tissue, that can compromise joint movement
Commonly seen in: palms, soles, anterior thorax following severe burns

Defects in collagen synthesis


Ehlers-Danlos syndrome** Scury Osteogenesis imperfecta
Group of genetic d/o characterized by defects in
synthesis + structure of type I + type III collagen
Vit. C def. first affecting collagen
Clinical finds: skin hyperextensible, fragile, vulnerable
w/ highest hydroxyproline content,
to trauma, ecchymoses (“bruise” - a small hemorrhagic
such as that found in blood
spot in the skin or a mucous membrane), poor wound
vessels. Defect in collagen type I
healing, hypermotile
Early sx: bleeding gums
Common cause of death = aortic dissection - tear in
Enzyme affected: prolyl
the wall of the aorta that causes blood to flow between
hydroxylase
the layers of the wall of the aorta and force the layers
apart)

Important growth factors involved in wound healing


PDGF: platelet-derived growth factor Chemotactic for monocytes, PMNs,fibroblasts
Promotes migration and proliferation of fibroblasts & SMC
EGF: epidermal growth factor Promotes granulation tissue formation
VEGF: vascular endothelial cell growth factor Stimulates angiogenesis by attracting endothelial cells, ↑ increases
vascular permeability
BFGF: basic fibroblast growth factor Stimulates angiogenesis by attracting endothelial cells
TGF-beta: transforming growth factor beta Chemotactic for macrophages + fibroblasts
anti
infamy Modulates repair process by mediating type III Δ à type I
Growth inhibitor for many cells involved in wound healing
HEMODYNAMIC Disorders
Edema: abnormal accumulation of fluid in interstitial tissue spaces or body cavaities (effusions)
4 types:
See acute inflammation
- Protein poor (<3g/dL) and cell poor fluid
- Associated w/ alteration in Starling’s forces:
inf
Transudate Acute
• ↑ hydrostatic pressure (HP)
edema
Pitting RHF • Decreased (COP) colloid oncotic pressure (hypoalbuminemia)
- PRODUCES PITTING EDEMA!!!!!!!!
See acute inflammation
- Protein rich (>3g/dL) and cell rich (i.e. neutrophils) fluid
Exudate
- Assoc. w/ ↑ vascular permeability as in acute inflamm
- Produces swelling of tissue but NO PITTING EDEMA!!!

- Protein rich fluid associated w/ lymphatic obstruction


Lymphedema
- Produces NON-PITTING EDEMA

- ↑ hyaluronic acid and chondroitin sulfate


Glycosaminoglycans (GAGs) - Myxedema – NOT PITTING edema

CLINICAL CORRELATES OF Effects may range from annoying (in leg) to rapidly fatal (in lungs, brain)
EDEMA Distribution can indicate etiology:
• Pitting dependent edema found in rt. Heart failure
• Generalized edema w/ proteinuria is indicative of renal dz (nephrotic
syn)

Broad causes of Edema


Inflammatory causes Non-inflammatory causes
Results in exudate accumulation (↑ vascular permeability) Results in transudate accumulation (↑ HP)

Pathophysiologic categories of edema


↑↑ Hydrostatic forces fluid out of vessels, seen in:
Heart failure Venous blockage
↑ HP *pulmonary edema – Lt VF *lower leg edema: deep saphenous vein,
*peripheral pitting edema – Rt. <3 pregnancy
failture *ascites: portal HTN due to cirrhosis
Usually due to hypoalbuminemia:
1. Protein deficiency à kwashiorkor (protein calorie deficiency)
2. Liver dz à cirrhosis = reduced protein/albumin synthesis
3. Nephrotic syndrome
↓ COP
• >3.5g urine protein loss/day
• Minimal change dz MCC in children
• Membranous glomerulonephritis MCC in adults
4. Protein losing enteropathies à celiac dz
Blocks removal of interstitial fluid à lymphedema
Clinical examples:
1. Lymphedema of arm: following modified radical mastectomy (MCC in US)
Lymphatic obstruction 2. Elephantitis: due to Wuchereria bancrofti
3. Scrotal/vulval edema: due to Lymphogranuloma venerum (a STD)
4. Breast lymphedema: due to plugging of dermal lymphatics by tumor cells à
called Peau d’orange (skin looks like orange peel)
↑ HP (↑ plasma volume) and ↓ COP (b/c of dilutioning effect on albumin)
1o Na/H2O
Acute glomerulonephritis
retention
Sodium retention Clinical
↓ CO (LVF) à ↓ renal blood flow à
examples 2o Na/H2O
activation of renin-angiotension-aldosterone
retention
system à reabsorbs Na à water follows
- Occurs in acute inflammation or with injury to capillary endothelium
Inflammation (↑ vessel
- Produces exudates
permeability)
- E.g. bee sting à tissue swelling, burn injury
- Produces myxedema
- T-cell cytokines stimulate fibroblasts to produce GAGs
↑ GAG synthesis
- E.g.: exopthalmos & pretibial myxedema in Grave’s dz
- GAGs hold on to water well, forming lubricating, viscous fluid

Two important forms of edema


Pulmonary edema Cerebral edema
LVF (transudate formation)
Cause Could be generalized or localized to site of injury
ARDS, pneumonia (exudates)
Generalized
Gross: swollen, narrow sulci, flattened gyri
Clinical fx Cough w/ frothy sputum
Assoc w/: encephalitis, severe HTN, obstruction of venous
outflow, trauma
Heavy lungs, fluid-filled, ooze fluid & foam Localized
Gross
on cut section Associated w/ abscess, neoplasia, trauma
Clinical significance
can impede ventilation & O2 diffusion *compression of vascularure, CN à HA, papilledema,
Clinical
extra fluid creates favorable environment for behavioral/personality Δ
correlations
bacterial growth à pneumonia *if severe à herniation of brainsteam through foramen
magnum à fatal!

TERMS TO MEMORIZE (to understand later)


Anasarca Severe generalized edema
Hydrothorax/pleural effusion Accumulation of fluid in pleural cavity
Hydropericardium Accumulation of fluid in pericardial cavity
Hydroperitoneum/ascities Accumulation of fluid in peritoneal cavity
Dependent edema Accumulation of f luid in dependent parts of body
• Ankles/lower legs in ambulatory pts
• Presacral region in bedridden pts
Periorbital edema Accumulation of fluid around the eyes
Pitting edema Finger pressure leaves an indentation in the edematous tissue
ONLY TRANSUDATES produce this
Not-pitting edema Finger pressure does not leave an indentation
Hyperemia, Congestion, Hemorrhage, Thrombosis

HYPEREMIA + CONGESTION = increased blood volume in a tissue


Hyperemia Active process/active hyperemia
Def: a localized ↑ in the volume of blood in the capillaries/small vessels, resulting
from localized arteriolar dilation
2o to: neurogenic reflexes (blushing); chemical induced VD (via histamine)
Gross: tissue is erythemic (red)
Congestion Passive process/passive hyperemia
Results from: impaired venous outflow or ↑ pressure from heart failure
Gross: tissue is cyanotic (reddish-blue)
Two types:
• Acute passive congestion – occurs in sudden RHF + LHF
• Chronic passive congestion – due to chronic RHF + LHF
- Two types of CPC:
Chronic venous May occur w/ chronic LHF
congestion of Congestion & distension of alveolar capillaries à rupture à
lung (CVC lung) RBC in alveoli
Phagocytosis & degradation of RBC à heart failure cells =
hemosiderin laden macrophages w/in alveoli
Long standing cases: fibrosis of alveolar septa, hemosiderin
deposition (brown induration of lung)
Chronic venous Most common w/ chronic RHF
congestion of Gross: “NUTMEG LIVER” (!!)– speckled nutmeg like
liver appearance on cut section
• Produced by a combo of : dilated, congested central
veins & surrounding brownish yellow, often fatty, liver
Micro: congested central vein, atrophy of centrilobular
hepatocytes, congested sinusoids, periportal hepatocyes are
less/not really affected

Nutmeg liver

HEMORRHAGE
Background Extravasation of blood due to vessel rupture
May be: external or enclosed w/in tissue (hematoma)
Causes: bleeding d/o (scurvy, thrombocytopenia, clotting factor deficiency), vessel
injury (trauma, vasculitis, tumor invasion, HTN)
Grouped according to size • Petechiae – 1-2 mm, usually associated w/ ↑ intravascular pressure, ↓
platelets (thrombocytopenia)
• Purpura – 3-5 mm, same cause as petechia + vasculitis or ↑ vascular
fragility
• Ecchymoses – 1-2 cm, commonly called bruises (subcutaneous
hematomas)
Within body cavities 1. Hemothorax – pleural cavity
2. Hemopericardium – pericardial sac
3. Hemoperitoneum – peritoneal cavity
4. Hemoarthrosis – joint space
Clinical significance Depends on rate, volume, and location of blood loss
• Significant blood loss à hypovolemic shock + death
• Small amts w/in brain à can cause severe damage
• Chronic or recurrent external hemorrhage (peptic ulcers, menstruation) à
Fe-deficiency anemia (IDA) Deficient
Iron Anemia
• Hemorrhage w/in body cavities à less likely to result in IDA
THROMBOSIS
Definition: intravascular coagulation of blood – often causing interruption of blood flow
It’s a solid mass formed w/in uninterrupted blood vessel/heart chamber
- Composed of coagulation factors, platelets, fibrin, and entrapped RBCs and WBCs
- Formed during life
- Always attached to the vessel wall/chamber of heart
PATHOGENSIS OF THROMBUS FORMATION – VIRCHOW’s TRIAD

Most important cause of thrombus formation (arterial thrombi)


Intact endothelial cells resist formation of thrombus, but endothelial damage exposes
platelets to underlying collagen & other activating factors
1. Chemicals in cigarette smoke a
Endothelial cell
injury 2. Turbulent blood flow @ arterial bifurcation
5
Causes of endothelial
3. Ruptured atherosclerotic plaques
injury expose
underlying
collagen

4. MI (endocardial damage)

5. Increased homocysteine levels

Turbulent blood flow


Stasis of blood (more important of 2,
venous thrombi)
Seen in: surgery (prolonged bed rest),
Causes: atherosclerotic plaque, prolonged sitting (int’l flights)
aneurysms Assoc w/: thrombus formation in veins
Alterations in normal
blood flow Assoc w/: thrombus formation in arteries 3 consequences:
2 consequences: 1.disrupts laminar flow & brings the platelets in
1.disrupts laminar flow & brings platelets contact w/ endothelium
in contact w/ endothelium 2.prevents dilution of activated clotting factor
2. causes endothelial damage 3. retards inflow of inhibitors of clotting factors

Uncommon cause of thrombosis


Heritable causes Acquired causes
-factor V gene mutation: -oral contraceptives, pregnancy
* results in formation of Factor Vleiden * ↑ hepatic synthesis of coagulation factors (V
* activated factor Vleiden resistant to & VIII)
Hypercoagulable
inactivation by protein C * ↓ production of antithrombin III
blood
*associated w/ recurrent deep vein -other acquired causes
thrombosis * secretion of thrombogenic substances by
-deficiency of anticoagulants tumors (e.g. adenocarcinoma of the pancreas)
* antithrombin III deficiency à mucin à migratory thrombophlebitis =
* protein C & protein S deficiency Trousseau’s syndrome
TYPES OF THROMBI
Can develop anywhere in the CV system
Major causes Stasis of blood, hypercoagulative states
Fibrin, entrapped RBCs, platelets, WBCs, appear red (red thrombus),
Composition
Venous thrombi adherent to vessel wall
(phlebothrombosis) 1. Surgery (worst à hip replacement)
2. Immobility: post-op state
Risk factors
3. Stasis of blood --> via CHF, burn, long flight
4. Malignancy (adenocarcinoma of pancreas)
5. Hypercoagulation à oral contraceptives
MC à deep vein in lower leg below knee
Sites
Others: superficial saphenous, hepatic, renal veins, dural sinuses
Venous thrombi in deep veins of LE
DVT Propagate in direction of blood flow (toward heart) --> danger of
pulmonary artery embolization (PE)
Platelets held together by fibrin strands, RBC less in #, appears pale
Composition
red (pale thrombus), lines of Zahn (laminations?)
Usually begins @ site of endothelial injury, platelets will adhere to this
Pathogenesis
site
Risk factors All factors responsible for endothelial injury (cig smoke, MI, etc)
Overlie atheromatous plaques in coronary, internal carotid, cerebral
Sites and SMA; aortic aneurysms, heart chambers (mural thrombi); heart
Arterial thrombi valves (vegetations)
Mural thrombi Vegetations
Thrombi which develop over heart
Attached to large vessel wall
valves
Important terms (aorta), heart wall
Seen in: infective endocarditis,
Causes: MI, myocarditis,
nonbacterial thrombotic endocarditis
aneurysms
(NBTE) – in malignancies
Clin significance Infarction, stroke, embolization
CLINICAL SIGNIFICANCE
Arterial thrombi Venous thrombi
Possible source of emboli (Recall: pulmonary
May cause obstruction à infarction & stroke
thrombembolism)
FATE OF A THROMBUS
1. Propagation – thrombus may increase in size and length
2. Embolization – thrombus may break off and travel in the blood vessel
3. Dissolution – thrombus may be dissolved
4. Organization – covered by cells
5. Recanalization – lumen may develop which will allow blood flow
6. Infection + calcification – may get infected or may get calicified

DIFFERENCES B/W A THROMBUS + POST-MORTEM CLOT


Thrombus Post-mortem clot
* friable (readily crumbled)
* firm * not attached to the wall
* attached to the wall at the point of origin * separate dark red basal portion – “currant jelly”
* red in color (erythrocyte-rich layer) and yellow top portion – “chicken
fat” (erythrocyte poor layer)
EMBOLISM, DECOMPRESSION SICKNESS, INFARCTS

EMBOLUS
Definition: any detached intravascular mass (solid, liquid, gas) that travels along the blood stream to a site distance from
its origin (often resulting in occlusion of a vessel)
• Occlusion of vessel à ischemia à necrosis
• Infarction = ischemic necrosis (coagulative or liquifactive)

Clinical outcomes of thromboemboli, depend on:


• Site of origin & site of lodgment of the emboli
ONE OF THE GREAT KILLERS OF HOSPITALIZED PTS!
Source: most (95%) from deep veins of the lower extremity above the popliteal vein (i.e. femoral
v.)
Risk factors: post-op states, trauma, oral contraceptives, prolonged sitting (planes)
Clinical outcome depends on the size of the embolus:
Small size Medium Large
Pulmonary
Usually asymp or causes
circulation
transient dyspnea/tachypnea May/may not cause infarction May lodge bifurcation (saddle
PE – pulmonary
No infarction (dual blood of lung, often not b/c lung has embolus) or large pulmonary a.
thromboembolism
supply) dual blood supply branches and cause death (5%)
Complete resolution Infarction: if compromised Obstruction of >60% pulmonary
Recurrent small emboli à cardiac function (↓ CO) circulation required
pulmonary HTN
REMEMBER!: pulmonary thromboembolism can cause sudden death, pulmonary infarction
(protected by bronchial artery), pulmonary HTM (recurrent PE), most small pulmonary emboli lyse
Refers to emboli w/in arterial circulation
Source: most (80%) from lt. side <3 due to: lt. atrial thrombus (mitral stenosis); mural thrombus in
System circulation acute MI; vegetations in aortic/mitral valve
(systemic arterial Outcome: most à infarction
emboli) common sites of embolism: LE (most common!), brain (MCA), small intestine (SM artery),
kidney, spleen
clinical fx: pale infarcts (digits, spleen, kidneys); hemorrhagic infarcts in brain & small bowel

Paradoxical emboli: arise in the venous system and pass through an atrial septal defect (ASD) into systemic circulation

Types of emboli
Most common – fx of long bones (femur) and pelvis
Cause
Others causes – trauma to fat laden tissue & fatty liver
-Microglobules of fat obstruct microvasculature à ischemia & hemorrhage
Pathogenesis -mobilized fat à fatty acids then damage endothelium à platelet thrombi (Recall:
Fat basement membrane of endothelium very attracted to platelets!)
Most asympt!
Symptomatic cases (after 24-72 hrs): *CNS dysfunction, respiratory failure,
Clinical fx
thrombocytopenia (platelet consumption in thrombi) à petechiae over chest and
UE, death (in <10% cases)
Cause Amniotic fluid infusion into maternal circulation during delivery
Tear in the placental membrane or uterine veins à infusion of amniotic fluid w/ fetal
Pathogenesis squamous cells and lanugo hair (normal hair on fetus during gestation) and
procoagulatins into maternal circulation
Amniotic fluid Sudden onset of dyspnea, cyanosis and hypotensive shock & bleeding
• Dypsnea: due to pulmonary edema or ARDS
Clinical fx
• Bleeding: due to DIC à amniotic fluid rich in thromboplastin
Mortality rate > 80%
Dx Confirmed at autopsy (look for fetal squamous cells, hair etc in pulmonary vessels)
Refers to gas bubbles w/in circulation obstructing vascular flow, causing ischemia
Causes:
• Ruptured uterine veins à delivery & abortion
Air embolism
• Ruptured chest veins from chest trauma
• Iatrogenic: CVP lines and surgical procedures on head & neck
• Decompression sickness (see below)
DECOMPRESSION SICKNESS (Caisson Disease)
What is it? A particular form of gas embolism caused by sudden Δs in atmospheric pressure
@ risk Scuba and deep sea divers
Pathogenesis Atmospheric pressure ↑ by 1 for every 33 feet of descent into water à under increased atm,
pressure nitrogen gas dissolves in blood and tissues à rapid ascent – forces N2 bubbles to
develop in tissues & lumen of blood vessels
Clinical findings • Bends – pain in jts, skeletal muscles, bones
• Chokes – gas bubbles in lung
• Pneumothorax – air in pleural cavity complication of sudden rise to surface à ruptures
subpleural bleb causes dyspnea & pleuritic chest pain
• PE - ~60 ft in water à ↑ pressure in LE à stasis, thrombus formation – PTE à
dyspnea, pleuritic chest pain
• Chronic Δ – aseptic necrosis in bones (head of femur, tibia, humerus)
TX: recompression (N2 forced into solution) followed by slow decompression

INFARCTS
Definition Localized area of necrosis due to ischemia – gross manifestation of coagulation necrosis
Most (99%) caused by occlusion of artery by thrombus or emboli
Pathogenesis
Other causes: vasospasm, torsion of artery and veins (volvulus, ovarian & testicular torsion)
• Vulnerability to hypoxia: neurons >> myocardial cells >> skeletal muscles
• Degree and rate of development of occlusion
Influential factors
• Nature of blood supply – dual (lungs, liver) or single (spleen, kidney)
• Oxygen content of blood -- anemia
Depends on color, 2 types Depending on presence/absence of microbial infection
Types of Infarct - Red infarct (from hemorrhage) - Septic infarct
- White infarct (anemic) - Bland infarct
Often wedge-shaped
Apex of wedge = point of occlusion!
Anemic (pale,white) Hemorrhagic infarcts (red color)
Occurs in organs w/ dual blood supply
or collateral circulation (lungs, liver,
Gross Occurs in solid organs w/ single
intestine); in loose textured tissues (lungs,
Morphology blood supply (spleen, kidney, <3)
small bowel)l in venous occlusions
↑ density of tissue prevents RBCs
(testicular torsion); @ a site of previous
from diffusing through necrotic tissue
occlusion and necrosis when flow is
reestablished
Microscopy Coagulative necrosis: most organs
of infarct Liquifactive: brain!
PATH BLOCK 1 REVIEW:

I - GROWTH ADAPTATIONS

1) HYPERTROPHY - increase in size of the cells and NO increase in # of cells

◦ it can only happen in cells that can't divide (post-mitotic cells)

◦ most important clinical association: Left Ventricle Hypertrophy (due to increased peripheral resistance)

‣ mechanism = cell signaling pathways

‣ growth factor: TGF-B

‣ pathway: G-protein

‣ transcription factor: GATA4 -----> transcripts more contractile proteins

‣ other safety measures the body does:

• switch in the type of fibers (adult to fetal) due to fetal being more economical

• re-expression of ANF -> help in removing salt from the body = water also removed = decreases
blood volume = eases the heart

‣ overtime all of these effects result in regressive changes:

• random cells dying by apoptosis

• production of collagen in these areas = can't contract = compensate heart (leading to CHF)

◦ perform echocardiogram: pt will show reduced ejection volume

****LINK THIS WITH HEMODYNAMIC PPT: PULMONARY EDEMA****


• In microscopy characteristic you'll find in pulmonary edema pts: hemosiderin-laden macrophages (heart failure cells)

2) HYPERPLASIA - increase in # of cells = increase in organ size

◦ Males -> Prostate Hyperplasia

‣ aging males, testosterone declines, BUT prostate still enlarges because it's dependent on DHT

‣ prostate glands contain glands & stroma, and both respond and increase upon DHT

‣ KNOW THE PIC

***Infolding of lining

• glands should always be tubular - when you


see this infolding it empasizes that more cells
are piling up in the glandular lining and push
out
• they'll also look more blue (due to more nuclei)
• picheale al stroma

Prostate: note glandular crowding

‣ Clinical relevance: we divide the prostatic gland based on outer and inner zone

• BHP affects the inner zone -> compresses prostatic urethra = causing sx

◦ retention of urine (always a source of infection via stagnate fluid = prostatitis)

◦ incomplete evacuation of the bladder

◦ frequent sensation of passing urine

◦ Post-Menopausal Women -> Endometrial Hyperplasia

‣ seen mostly in caucasians -> have higher rate of bone loss

‣ bone loss is due to loss of estrogen (estrogen has a protective effect)

‣ as tx: women are given HRT and this exogenous estrogen not only acts on the bones, but also on the
endometrium inducing proliferation

‣ proliferation = cells dividing = mutations can arise --> adenocarcinoma of the endometrium

‣ KNOW THE PIC

• A LOT of glands and no interviening

stroma

• Post-menopausal woman should have

very sparse glands!!!!!


Endometrium: note proliferation of the glands and reduced stroma

3) ATROPHY - decrease in cell size & #

◦ not actual cell death, they just function at a suboptimal level

◦ has MANY causes (know them!)

‣ Fracture of a lower limb

• muscle doesn't break down proteins that aren't required at that time causing the cells to become
smaller (he'll show you a muscle biopsy pic)

◦ reversible - once pt puts weight on leg everything returns to normal

‣ Loss of innervation

• due to trauma or infection

‣ Compression of a blood vessel by a tumor

◦ Mechanism: (super important)

‣ Ubiquitin-Proteosome

• **** seen here & in mallory bodies (cytokeratin damage in alcoholic liver)

◦ Implemented to cell death: (depending on tissue)

‣ @ CNS: Alzheimer's disease - amyloid plaques causes damage to neurons that initially undergo
atrophic change & then cell death

‣ Associated with Cystic Fibrosis (@ pancreas) -> thick secretions = pressure builds up within the duct
system into the acinar cells = these cells then show atrophic change and die

‣ Stones -> @ Densen's duct in salivary gland & stones in the ureter = pressure builds up -> atrophic
changes -> apoptosis

4) METAPLASIA - adult cell type changes to another

◦ focus on junctional areas!

• GERD/Barrett's:

◦ esophageal & gastric junction -> have two different linings (esophagus lined by squamous epithelium) &
(distal 1-2cm of esophagus is lined by the columnar epithelium of the stomach)

◦ when you look at the lower esophageal sphincter in people who are obese and smokers - they lose it's
function and the acidic contents of the stomach empty into the squamous lining epithelium -> transforms to
intestinal type epithelium

‣ obviously it can't be gastric lining because it would cause even more acid, so the body converts to a
type of epithelium that would neutralize these acid secretions: intestinal always recognized by presence
of goblet cells

◦ What is GERD & what is Barrett's?

‣ GERD: symptomatic, pt will mention heartburn, epigastric distress, belching

‣ Barrett's: when you do a biopsy and you find the epithelium has transformed

"Creeping" intestinal type epithelium

*** GOBLET CELLS!!***

• Smoker's Lung:

◦ prenative epithelium is ciliated columnar epithelium gets transformed to squamous in patches

‣ when you do an endoscopy: shiny epithelium (normal) & dull patchy areas (metaplastic)

◦ know the sequence of events:

‣ native ciliated columnar epithelium -> squamous epithelium -> undergoes dysplasia -> malignancy
(squamous cell carcinoma)

NO cilia and NO goblet cells!

• Bladder: (Transitional to Squamous metaplasia)

◦ due to Schistosoma hematobium -> produces intense inflammation of the bladder mucosa causing the
epithelium change in patches

‣ when you do a cystoscopy you'll see indurated patches that are reddish in color

◦ risk of squamous cell carcinoma in the bladder

• Stomach: (gastric to intestinal type)

◦ due to H. pylori

◦ lining changes because the area changes from acidic environment to alkaline environment (due to urease
production)

◦ recognize epithelium change by presence of goblet cells

• Pathogenesis of metaplasia (in general) - KNOW IT

◦ Underlying mechanism: inflammation -> produce cytokines -> send signals to stem cells -> stem cells
differentiate accordingly

• Osceous Metaplasia: Myositis Ossificans

◦ blunt trauma to bulk muscle (deltoid, biceps, thigh muscle, gluteus) -> produce rupture of blood vessels =
forms intramuscular hematomas

◦ intramuscular hematomas in some instances cells differentiate into cartilage or bone

◦ there is NO connection to the underlying bone (it's a separate thing)

II - CELL ADAPTATION & INJURY

• Reversible VS Irreversible

◦ Reversible = body volume regulation being upset = cell swelling and swelling of the organelles

◦ It's irreversible when of all the reversible changes cumulate & the earliest landmark for irreversible change
(point of no return) = amorphous densities in the mitochondria (Ca2+ entry) -> best seen in ischemic
myocardium

seen only in electron microscopy*

Note: amorpheous density in mitochondria

• Mechanism of Cell Injury

◦ Know: CYANIDE -> affects the oxidative phosphorylation (cytochrome oxidase)

‣ depletion of ATP

◦ Influx of Ca2+: (important concept!)

‣ once Ca2+ enters it's going to act on various enzymes

• most importantly: phospholipases = damages all the cellular constituents (whether mitochondrial,
plasma membrane or lysosomes)

◦ Free Radicals: (how it produces damage in various situations) - KNOW ALL THE EXAMPLES

‣ in relation to drugs (acetaminophen OD) & chemical (CCl4) -> both act thru p450 system = increases
enzymatic activity and makes enzymes highly inducible and convert these drugs to intermediates (a.k.a
free radicals) -> body can't scavenge them completely so they produce damage => massive hepatic
necrosis

‣ UV rays: causes damage cuz cells contain water so light hydrolyze it and converts to free radicals

‣ Iron Overload: dangerous because it's never left free in the body (RBC's bind it to Hb, or liver/bone
marrow it's kept in storage form: ferritin or hemosidirin) too much iron being stored -> can't be held
leading to tissue damage

• classic ex: kid with beta-thal or sickle cell given standard tx - blood transfusion = so you add more
iron to the body = produces free radicals and when the kid grows up their liver undergoes cirrhosis

***LINK THIS WITH HEALING & REPAIR PPT***


◦ if there is constant injury to the liver, initially it can fix it but after some time there is a bunch of TGF-B being
produced by the inflammatory response and you have fibrous tissue being predominant = cirrhotic liver

‣ Ischemic Repurfusion: window period you can give plasminogen activators to break the thrombi
(thrombolytics) -> if given out of this window ischemic myocytes would convert the oxygen to free
radicals producing an extension of injury

*** LINK THIS WITH HEMODYNAMICS PART II PPT***


◦ normally the type of infarct you would see on the heart would be red, but if it's repurfusion injury you'd see white
infarct

‣ Rertinopathy of prematurity: hypoxia -> hypoxia-inducing factor -> blood vessels proliferate
(angiogenesis) - these new blood vessels are thin walled and very fragile = liable for rupturing

• babies are give a lot of O2 -> free radicals produced in those areas -> blood vessels rupture ->
leading to separation of retina (retinal detachment)

‣ KNOW the molecular targets for free radicals:

• phospholipids = lipid peroxidation


of the membrane

• proteins = cross-linking of the


cytoplasmic proteins

• DNA = single or double stranded


breaks (leads to mutations ->
cancer)

◦ Know clinical correlations of defects in membrane permeability:

‣ intermediate filament (cytokeratin) damage in alcoholic hepatitis = produces mallory bodies via the
ubiquitin-proteasome mechanism

‣ know the pic!!!!

**smudgy pink

material in

hepatocytes =

mallory body

** DO NOT CONFUSE WITH

APOPTOSIS!!!!!!! (they'll

have retraction artifacts)

‣ neurofilaments damage seen in Alzheimer's Disease

• Tau protein normally organizes all the tubules, in


AD it gets hyperphosphorylated =
disorganization = twisting and tangles ->
neurofibrillary tangles

• exam questions can mention use a special stain:


silver stain (shows blackened tissue)

‣ damage to microtubules: Kartagener's (primary ciliary dyskinesia) (super important!)

• falls under the category: repeated infections in kids

• x-ray will show situs inversus

• dynien arms = cilia can't function

◦ Know examples of Hypoxia:

‣ Anemia

• iron deficiency anemia seen mostly in young women in child bearing years

• almost no iron in diet = hemoglobinization doesn't happen properly = delivery is going to frail

• reduction in O2 delivery

‣ CO poisining

• seen in cold weather when people aren't careful about their indoor heating

• CO is invisible, odorless and produces color change in the blood and tissue (cherry red skin)

• mechanism: CO has a higher affinity for Hb than O2

‣ Methemoglobinemia

• seen in pts who have MI who forget they already took nitroglycerin and take even more

• pt will then have low O2 saturation = weakness, breathing difficulty, also color changes to the skin
(gray-slate color) & chocolate colored blood

◦ Know examples of Ischemia:

‣ Thrombotic or embolic occlusion

‣ Venous

• Budd-Chiari (you will have clinical scenario


on this!) -> hepatic vein being blocked

◦ Know the tissues suceptible to hypoxia:

‣ Watershed areas: shared between two blood vessels

• ex: CHF pt scenario -> reduced perfusion in the cerebral arteries (anterior & middle) => "Man in the
Barrell Syndrome" (can't move)

‣ Subendocardial tissue: seen when a transmural thrombi -> entire LV is being affected = area that is
going to get maximum damage is going to be the area furthest severed from the blood vessel
(subendocardial)

‣ Renal cortex & medulla:

• ex: pt with MI and ends up with reduced perfusion in the kidney -> the medulla is the area that is
going to be affected the most because it has LESS blood supply -> PCT have straight portions that
bend and dip into the medulla (this portion is the MOST vulnerable) because it requires more
energy than the rest of the kidney due to it's job to reabsorption

*** LINK THIS WITH ATRIAL FIBRILLATION ***


‣ You'll see reduced perfusion in the kidney = renal infarcts are common with atrial fib pts. They'll present with discoloration &
blood in the urine (hematuria) & pain in the flanks due to the tissues dying and starting to breaking down

‣ Purkinje Fibers in cerebellum: the cells here are layered and have preferential blood supply, so in the
setting of overall reduction perfusion these cells may become damage

• Ischemic Diseases:

◦ Buerger Disease (Thromboangitis Obliterans - TAO): seen in young male smokers

‣ damage to the endothelium -> inflammation -> the downstream areas undergo ischemia

*** CONNECT HERE: FIBRINOID NECROSIS ***

‣ TOA is an example of this because there is some component of the ciggarrete that is causing damage to
the endothelium producing inflammation & coagulation cascade

◦ Testicular torsion: know the anatomical/embriological reason -> tunica vaginalis thingy

◦ Budd-Chiari -> again SUPER IMPORTANT


Hepatic Vein blocked

◦ Ischemic - Repurfusion injury: know it's caused mostly by free radicals, but also intracellular calcium and
inflammatory cytokines play a role here

• KNOW THIS PIC!!!!!!!

***Massive Hepatic Necrosis**


• acetaminophen or CCl4 poisoning

coagulative necrosis (note abscence


of nuclei in individual hepatocytes BUT


architecture still preserved)


inflammation


hemorrhage

III - NECROSIS & APOPTOSIS:

• Know the nuclear changes seen in necrosis:

◦ The most common one you'll see in histology: karyorrhexis

◦ Pyknosis is a term shared both in necrosis and apoptosis

• KNOW THE TYPES OF NECROSIS!!!!

◦ Coagulative necrosis:

‣ ischemia seen in every tissue (except CNS) -> infarcts

‣ called coagulative due to denaturation of proteins and enzymes

‣ tombtstone appearance: preservation of the cell outline but absence on nuclei (karyolysis)

‣ clinical: breakdown of cells cause leakage of specific enzymes in the blood (ex: MI & troponins, CK)

◦ Liquefactive necrosis:
*** cell debris + inflammatory cells ***
‣ two settings:

• bacterial/fungal infections -> abscesses

• CNS ischemia

◦ Caseous necrosis:

‣ modified coagulative necrosis

‣ TB gets ingested my the macrophages and in the process of trying to kill it -> leaks out enzymes that
can damage to the surrounding tissue leading to necrosis in those areas

*** LINK WITH CHRONIC INFLAMMATION PPT: TUBERCULOSIS ***


◦ Formation of granulomas: collection of modified macrophages (epitheliod cells - they have more power to kill - have function of
containment)

‣ can be seen in small amounts in Leprosy

‣ Calcifications -> indicates past exposure to TB (not necessarily infective since pt isn't symptomatic)

• screen test for TB => x-ray shows hilar node calcifications

TB Granuloma: note horse-shoe arrangement of nuclei in

Langhan Giant Cell

‣ REMEMBER: before the adaptive immunity is activated, TB can disseminate ANYWHERE in the body

• Pott's Disease -> dissemination to the spine, caused via


the lung being anatomically close to the paravertebral
region and the bacteria can keep spreading from there

Key Point: you don't need a giant

cell to be called as TB, all you need


is caseous necrosis and epitheliod

cells for dx

◦ Fat Necrosis:
‣ most common cause: alcohol (super important!!!!)

• damages acinar cells of the pancreas due to the metabolites of alcohol, especially acetaldehyde

• once the pancreatic cells are injured -> they release innappropriately enzymes in the activated form

• acute pancreatitis characterized by: acute abdomen

• Fat saponification: free FA being released from the adipose tissue are acted upon by calcium
(relevant for X-rays)

• KNOW THE PIC

*** shadowy necrotic


fat cells***

inflammation

Fat necrosis: Acute Pancreatitis

◦ Fibrinoid Necrosis:
‣ seen in two settings...

• Immune Vasculitis -> circulating Ag-Ab complexes that damage endothelium -> stimulate
inflammatory response and activate coagulation cascade -> fibrin makes thrombin -> occlude
small/med sized vessels

note: vasculitis

note: inflammation!!!

note: pink material (aka fibrin)

‣ HTN -> causes vascular changes (hyaline change with benign & fibrinoid change with malignant)

• Malignant HTN (Hyperplastic Arteriolosclerosis) -> pressures are so high they damage endothelium
and also underlying tissue (smooth muscle)

◦ when the damage is high-grade the endothelial cells are trying to put out even more
endothelial cells -> underlying BM gets duplicated and smooth muscle proliferates ->
produces onion skin pattern

Note: onion-skin pattern

NO INFLAMMATION!!!!!

Fibrinoid necrosis: Hyperplastic artetiolosclerosis

• Apoptosis

◦ seen in:

‣ Cancer tx -> chemo/radiotherapy -> inducing nuclear damage to the tumor cells = beyond repair so
cells are shed off

‣ Hepatitis -> hepatocytes harboring the virus are recognized by CTL's (communicate thru death
receptor pathway) -> hepatocyte shuts down

‣ KNOW PIC - DO NOT CONFUSE WITH MALLORY BODY

Remember:

cell shrinkage = apoptosis &

retraction artifacts

‣ Sequence of events regarding anything that causes obstruction -> atrophy -> apoptosis

• ex: CF pancreas are always small due to atrophy and acinar cells dying, stones/caliculi in the
kidney or denser's duct (salivary gland)...

◦ Mechanism:

‣ some the apoptotic pathways can be manipulated

• ex: HPV - upsets these apoptotic pathways therefor allowing the cells harboring the virus to survive

‣ growth factors stimulate production of anti-apoptotic members of Bcl-2 family

• balance between proapoptotic & antiapoptotic molecules

◦ normally in positive signaling proapoptotic levels are kept LOW

◦ in cancer we induce apoptosis is by damaging the nucleus to the extent it can be repaired, so
the tumor cells are gonna synthesize more proapoptotic molecules

‣ problem when certain tumor cells start synthesizing a whole lot of antiapoptotic
molecules (ex: Follicular lymphoma due to translocation) -> chemotherapy won't do
anything to it (the proapoptotic molecules can't overwhelm the antiapoptotic molecules)

IV - INTRACELLULAR ACCUMULATIONS

1) Fatty change (hella important)

◦ Alcohol Fatty Liver: upset between NADH/NAD (trigger point)

◦ NAFLD: due to insulin resistance -> upsets lipolytic pathways in the liver and fat accumulates

◦ Protein malnutrition

‣ Kwashiorkor - child is given an only carb diet, lacking proteins -> so for caloric requirements the body
breaks down fat but apolipoproteins aren't available so fat can't be transported out and gets
accumulated in the liver

‣ Starvation - similar

◦ Diabetes Mellitus -> insulin resistance again

◦ Anoxia (hypoxia) - Right Heart Failure -> congestion of the liver -> hepatocytes around the central vein are
gonna get damaged more, and the hepatocytes near portal triad would get some amount of O2 and undergo
the fatty change

*** SPECIAL STAIN:

OIL RED O ***

Fatty change: routine stain

2) Cholesterol/Cholesterol esters

◦ Xanthomas: around eyes, elbows, achilles tendons

‣ biopsy you'll find: lipid-laden macrophages

◦ Can also be seen with inflammation: cells are dying (they contain
fat-phospholipids) and macrophages ingest this fat

3) Proteins:

◦ Nephrotic Syndrome: glomerular basement membrane damage ->


proteins escape in the urine

‣ Kidney tries to save these proteins by having the tubular cells


lining the PCT grabbing hold on to them = producing protein
droplets

◦ Diabetes Melitus: AGE + RAGE -> damages basement membrane of the kidney = allowing proteins to
escape

◦ Russell Bodies: chronic or persistant antigenic stimuli -> plasma cells produce a lot of Ig's (leading to
morphology changes)

‣ identify: pink globules inside the plasma cell

4) Hyaline Change: (two different causes -> same result -> same pic)
Dont confuse with fibrinoid necrosis-
hyperplastic arteriolosclerosis

◦ HTN -> hemodynamic stress on the walls of the small


blood vessels and they start leaking plasma proteins into
subendothelium and interact with smooth muscle and
collagen = produce the hyaline change

◦ DM -> AGE + RAGE (expressed by endothelial cells) ->


release cytokines, GF, ROS

◦ happens in small blood vessels because it's usually seen


in renal biopsies

5) Lipofuscin:

◦ seen in: "wear and tear" and aging

‣ old person dying in sleep with yellow/brown pigment in kidney

‣ athlete's synovial fluid showing lipofuscin

◦ differential from hemosiderin (won't stain with prussian blue)

6) Hemosiderin:

◦ seen in:

‣ hematomas (local)

‣ beta-thal/sickle cell (systemic): RBC's life-span is shortened -> destruction of RBC leads to
accumulation of iron which is stored and once this storage becomes plentiful you get accumulation of
hemosiderin

◦ Prussian Blue +++

7) Calcifications:

◦ Dystrophic -> dead and dying tissue (aka necrosis)

‣ serum Ca2+ ALWAYS NORMAL

‣ Psammoma bodies - peculiar histology (laminated pattern) and tumors associated with it

• Meningiomas (head)

• Papillary carcinoma thyroid

• Serous (papillary) carcinoma ovary

◦ Metastatic -> PTH

‣ chronic renal failure -> increased serum Ca2+ (hypercalcemia) -> pts get stones formed = "bones,
stones, groans"

V - INFLAMMATION PART I:

• Know acute vs chronic inflammation

◦ Acute -> few days, high grade fever, neutrophils

◦ Chronic -> months, low grade or "on and off" fever, lymphocytes/macrophages/plasma cells

‣ Presence of lymphocytes alone (lymphocytosis) --> viral infection

‣ Infection with only eosinophils (eosinophilia) --> parasites/fungal/helminth/bronchial asthma

• ID pics!


eosinophil

(pink/red granules)

neutrophil

• Know exudate vs transudate:

◦ Exudate: (always infective) when you have considerable amount of vascular permeability which permits the
escape of cells along with plasma proteins

◦ Transudate: situation where the cell junctions are so tight only because of the one force that is driving them
out: hydrostatic pressure that is so high-> only drives fluid out of capillaries (ultrafiltrate)

‣ seen in pulmonary edema

• Know what these adhesion molecules do:

◦ Selectins: only slows cells


rolling

◦ Integrins: ensures firm binding

• Know what produces chemostaxis:

◦ Endogenous agents: (chemical mediators)

‣ complement factors C5a

‣ leukotriene B4 (LKB4)

‣ cytokines (IL-8)

• Know the diseases:

◦ Leukocyte Adhesion Deficiency

‣ LAD1: B2 integrin deficiency

‣ LAD2: selectin deficiency

‣ repeated infections in early childhood

‣ blood labs -> neutrophilia, but none appear in tissue biopsy (because PMN's can't bind or leave the
blood vessels)

‣ umbilical stump not dropping after two weeks

◦ Chediak-Higashi Syndrome
‣ mutation in LYST

‣ falls under albino/vitaligo (depigmentation syndromes) -> *oculocutaneous albinism + silver hair

‣ labs: neutropenia

‣ know how to explain why bleeding/pigmentation issues

◦ Chronic Granulomatous Disease (CGD)

‣ NADPH oxidase deficiency

‣ child repeated infections and biopsy shows granuloma

‣ NBT test (has to be NEGATIVE in order to say kid has CGD)


‣ catalase + organism

VI- INFLAMMATION PART II


• Know how to apply diseases with the diagram

IgE/Ag

BASOPHILS

HISTAMINE

MAST CELLS (degranulate)


*** BRONCHIAL

ASTHMA ***

*** C1 INHIBITOR ENDOTHELIUM SMOOTH MUSCLE

DEFIENCY -> HEREDITARY (ALSO BRONCHIOLES)


ANGIOEDEMA ***

COMPLEMENT C5a, C3a (anaphylotoxins)


OXIDATIVE

CELL INJURY

PROSTAGLANDINS PGE2, PGI2 VASODILATION & INCREASED

POTENTIATION OF
VASCULAR PERMEABILITY

PAIN

BRADYKYNIN SMOOTH MUSCLE CONTRACTION


TXA2 PLATELET AGGREGATION &

VASOCONSTRICTION

LEUKOTRIENES

(LKC4/D4/E4) BRONCHOCONSTRICTION (***BRONCHIAL ASTHMA***)

(LKB4) CHEMOTAXIS
VII- CHRONIC INFLAMMATION

• Know EVERYTHING on granulomas

◦ definition: focal collection of activated (modified) macrophages -> “epithelioid cells”

◦ seen in:

‣ TB (horse-shoe shaped granuloma)

‣ other diseases (Chron's) = non-caseating granulomas

‣ Foreign-body (know pics - giant cells have a bizarre organization)


suture

fragments


Multi-nucleated

foreign body giant

cell

• Systemic effects/Lab Investigations:

◦ Lab counts:

‣ ex: pt with fever, CBC = > 50,000 ------> sepsis


(leukemoid rxn)

‣ shift to left - more than 10% band cells in


peripheral smears

◦ C-reactive proteins levels (acute phase reactants) & predictive values

‣ Use CRP levels to monitor pts who are in imminent threat for MI (a lot of atherosclerotic process)

‣ RA pts = CRP levels high = worsening joint disability (due to a lot of inflam. cells coming here)

‣ Used to monitor unnoticed infections: PID!! & chronic osteomyelitis

◦ ESR elevated = chronic inflammatory state

‣ Multiple Myeloma: plasma cell malignancy -> RBC's are going to be sticky so ESR increases

◦ Serous Inflammations and fibrinous which means fibrin escaping areas -> big problem for two important
body cavities: pericardium and pleura (visceral and parietal pleura glued together and produces constrictive
pericarditis - characterized by a friction rub)

VIII- HEALING AND REPAIR

• MMP's

◦ Atherosclerosis, in the atheroma (plaque) -> MMP's are still working to break down areas enlargement of
these atherosclerotic lesions

◦ they remodel tissue -> by reducing amount of fibrous tissue accumulating there (in scarring have a lot of
fibrous tissue)

• First intention Healing: (know what happens


in each time frame)

◦ Day 3-5 (most important): granulation


tissue -> if healthy then wound heals
but if any signs of unhealthy granulation
then leads to more trouble

• Second Intention Healing:

◦ myofibroblast -> allow wound


contraction so can bring wound edges
closer

• Local/Systemic Factors affecting wound healing (delayed) - know all of them

◦ Poor blood supply in lower limbs -> varicose veins/diabetic pts

◦ Hyperglycemia

◦ Steroids

◦ Vitamin C (subclinical deficiency so don't have overt features of scurvy but do have poor wound healing)

• Keloid: you have too much fibronistic response, based on skin type (dark skin people), commonly in head and
neck area have more chance of keloid forming

◦ microscopy -> dense collagen without blood vessels

◦ differentiate from desmoid tumors

‣ shows fibers going between muscle fibers, seen in anterior abdominal wall

keloid desmoid tumor

• Wound Contractures: seen in burns or a luquid being poured on someone (scalding) -> hands/soles

• Healing in specific organs ---> liver

◦ Liver Cirrhosis - know role of TGF-B here -> encourages fibrosis (major feature)

IX - HEMODYNAMIC DISORDERS PART I

• Edema:

◦ seen in the setting of congestive heart failure (connect with what we saw with the heart failure cells)

LVH -> Pulmonary Edema -> congestion of lung

• Right-sided Heart Failure

◦ typically occurs as Cor Pulmonale or it can happen as a continuum of the Left-sided Heart Failure

◦ in Cor Pulmonale: problem with lung -> affects the pulmonary vasculature and right side gets affected
because it has to use more effort to push more blood into lung because of pulmonary hypertension

‣ reciprocal changes you are going to get in the liver -> nutmeg liver

◦ JVP mentioned with right side - increased

◦ pedal edema more with right side

• Osmotic Pressure

◦ Liver impaired and don't produce albumin (cirrhotic liver - ascites)

• Protein lost in nephrotic syndrome or GIT in childhood diarrheas

• Lymphedema

◦ surgical or radiotherapy that can damage the axillary group of nodes

◦ pitted appearance you see if the superficial lymphatics are being plugged by tumor cells -> breast cancer

• Cerebral Edema: two types -> vasogenic & cytotoxic

• Hemopericardium - rare complication of MI

◦ you have a transmural infarct leading to rupture -> blood leaking into pericardium and produces cardiac
tamponade (characterized by faint heart sounds, JVP increase & low BP = Beck's Triad)

X- HEMODYNAMIC DISORDERS PART II

• Virchow's Triad:

◦ endothelial injury

‣ abdominal aorta undergoing ulcerated atherosclerotic lesion

◦ turbulence
‣ abdominal aorta -> aneurysm

◦ hypercoagubility

‣ Factor V Leiden -> caucasians

‣ Antiphospholipid Antibody Syndrome -> different race

‣ fetal loss can be seen in BOTH

‣ if pt has SLE and thrombotic syndrome -> antiphospholipid antibody syndrome

• KNOW ALL THE EMBOLISM!

◦ Pulmonary embolism

‣ know the setting of each type

hemorrhagic red infarct

*X-ray = Hampton's

Hump*

◦ Fat embolism

◦ oil red o staining infarct - clinical association

◦ Air embolism -> mentions Nitrogen and a diver

◦ Amniotic fluid embolism -> rare event of


premature separation of the placenta from the
uterine bed permitting the escape of amniotic fluid

‣ laminated squames with pulmonary vessels

• Infarcts
◦ Venous -> testicular or ovarian

◦ Mesenteric vessels in diabetics can involve both arteries and veins -> bowel infarction -> can extend to wet
gangrene necrosis

◦ Infarct colour - only one exception in heart following reperfusion -> red infarct

◦ Septic Infarction

‣ in the setting of infective endocarditis by sequelae of rheumatic heart disease, or another more virulent
organism or IV abuse -> tricuspid valve being the most damaged -> multiple microabscesses in the
lung

XI - AMYLOIDOSIS

• Know physical characteristics:

◦ Congo red staining

◦ contains non-branching fibrillar proteins

◦ composed of crossed beta-pleated proteins

• Important clinical scenarios:

◦ Multiple myeloma -> plasma cell malignancy where light chains converted to AL type of amyloids

◦ Chronic illnesses - inflammation or infections -> associated with increased SAA so AA type of amyloid

◦ Alzheimer's -> AB type

◦ B2-Microglobulin -> pts on hemodyalisis get carpal tunnel because B2 microglobulin doesn't get filtered
and deposits in joints

◦ Familial Mediterranean Fever -> mutated pyrin which stimulates IL-1 & IL-6 triggering inflammation and
can mimic peritonitis or joint pains

◦ Macroglossia -> seen in cretinism, GH, Beckwidth-Wiedmann


Practice Questions Hemodynamic disorders

(Please ignore questions in RED)

1.A 40-year-old woman dies after a long history of an illness characterized by


dyspnea, orthopnea, hepatomegaly, distended neck veins, and peripheral edema.
The cut surface of the liver as it appears at autopsy is shown in the first panel. The
second panel shows the microscopic appearance of the liver. Which of the
following disorders is the most likely cause of these findings?

(A)Chronic alcoholism

(B)Diabetes mellitus

(C)Niemann-Pick disease
(D)Right-sided heart failure

(E)Viral hepatitis

2.A 70-year-old man seeks medical attention because of shortness of breath on


minimal exertion. A poster anterior chest radiograph reveals blunting of the right
costophrenic sulcus interpreted as a right-sided pleural effusion. The aspirated
fluid is straw-colored and clear. The protein concentration is low, and the specific
gravity is 1.011. Microscopic examination reveals an occasional mesothelial cell.
Which of the following is the most likely cause of the effusion?

(A)Decreased oncotic pressure

(B)Left ventricular heart failure

(C)Mesothelioma

(D)Pneumonia

(E)Tuberculosis

3.A 26-year-old woman dies after a short illness beginning in the late stages of
labor. At autopsy, blood vessels in the lungs contained fetal debris (e.g.,
squamous cells, vernix, mucin), as did other vessels of multiple organs. Review of
the clinical history reveals that she had become acutely ill with dyspnea,
hypotension, and seizures, and a chest radiograph had demonstrated evidence of
pulmonary edema. This was all followed by prolonged hemorrhage from the
vagina and generalized bleeding from multiple other sites. The changes that were
found within multiple blood vessels most likely are

(A)bone marrow emboli.

(B)fat emboli.

(C)gas emboli.

(D)septic emboli.

(E)widespread thrombosis.
4.Two days following a cholecystectomy, a32-year-old hospitalized woman has
sudden onset of dyspnea, pleural pain, and cough productive of frothy, blood-
tinged sputum. Ventilation-perfusion scintigraphy indicates a perfusion defect. If
it were possible to examine a portion of the affected lung, which of the following
would most likely have been found?

(A)Air embolism

(B)Anemic (white or pale) infarct

(C)Disseminated intravascular coagulation (DIC)

(D)Generalized thrombosis

(E)Hemorrhagic (red) infarct

5.A 50-year-old right-handed man with a long history of rheumatic heart disease
with mitral stenosis and atrial fibrillation is brought to the emergency department
after collapsing to the floor at home. He is unable to speak or walk and has right
hemiplegia with a right extensor plantar response. These findings most likely
result from embolism to which of the following arteries?

(A)Anterior cerebral

(B)Anterior communicating

(C)Middle cerebral

(D)Posterior communicating

(E)Superior cerebellar

6.An 86-year-old man with a history of recurrent urinary tract infection presents
with fever, tachypnea, tachycardia, mental obtundation, and reduced blood
pressure. Which of the following forms of shock is most likely?

(A)Anaphylactic shock

(B)Cardiogenic shock
(C)Hypovolemic shock

(D)Neurogenic shock

(E)Septic shock

7.A 60-year-old man with unstable angina (a form of acute coronary syndrome) is
treated with an intravenously administered glycoprotein IIb-IIIa inhibitor. The
mechanism of action of this agent is the ability to

(A)dilate coronary arteries.

(B)inhibit atherogenesis.

(C)inhibit platelet adhesion.

(D)inhibit platelet aggregation.

(E)lyse thrombi.

8.A 23-year-old man undergoes surgery for fractures of the pelvis and left femur
resulting from a high-speed motor vehicle accident. The following day he
develops dyspnea, speech difficulties, and a petechial skin rash. Which of the
following types of embolism is the likely cause of these findings?

(A)Air

(B)Amniotic fluid

(C)Fat

(D)Paradoxical

(E)Thrombotic

9.A 56-year-old man is surgically treated by a four-vessel coronary artery bypass


graft procedure and placed on prophylactic daily aspirin therapy. Aspirin has been
shown to prevent recurrent myocardial infarction through its ability to inhibit the
synthesis of
(A)adenosine diphosphate (ADP).

(B)leukotriene B4(LTB4).

(C)nitric oxide (NO).

(D)prostaglandin I2(PGI2).

(E)thromboxane A2(TxA2).

10.A bedridden elderly patient experiences the sudden onset of pleuritic pain and
hemoptysis. The underlying lesion that led to this complication was most likely
located in which of the following sites?

(A)Hepatic veins

(B)Lower extremity veins

(C)Pelvic veins

(D)Portal vein

(E)Pulmonary vein

11.Fluid is aspirated from the grossly distended abdomen of a 47-year-old chronic


alcoholic man. The fluid is straw-colored and clear and is found to have a protein
content (largely albumin) of 2.5 g/dL. Which of the following is a major
contributor to the fluid accumulation in this patient?

(A)Blockage of lymphatics

(B)Decreased oncotic pressure

(C)Decreased sodium retention

(D)Increased capillary permeability

(E)Inflammatory exudation
12.A 36-year-old man dies during cardiac surgery. He had a history of long-
standing rheumatic heart disease with mitral stenosis. At autopsy, the pathologist
reports findings consistent with mitral stenosis and noted the presence of “heart
failure cells.” This finding results from

(A)activation of the coagulation cascade.

(B)chronic passive congestion of the lungs.

(C)hypoxic myocardial injury.

(D)myocardial hyperemia.

13.During a laboratory exercise on coagulation testing, a 23-year-old medical


student is found to have a prolonged bleeding time. She has had a long history of
“easy bleeding,” with frequent bleeding of the gums, epistaxis, cutaneous
bleeding, and menorrhagia. Further testing revealed a deficiency of
vonWillebrand factor. Which of the following thrombogenic processes involving
platelets is most directly impaired?

(A)Adhesion

(B)Conformational change with activation of phospholipid surface

(C)Formation of fibrinogen bridges

(D)Release reaction

(E)Stabilization of platelet plug

14.A 28-year-old woman is evaluated for possible thrombophilia since she has had
two episodes of deep vein thrombosis, as well as two pregnancies that
terminated in spontaneous abortion. The activated partial thromboplastin time is
prolonged, and she has a positive VDRL screening test for syphilis. This
combination of findings is most suggestive of

(A)antiphospholipid antibody syndrome.

(B)disseminated intravascular coagulation.


(C)factor V Leiden.

(D)methylene tetrahydrofolate reductase mutation.

(E)prothrombin 20210A transition.


1.The answer is D.

The clinical findings described in the question are typical of right-sided heart
failure, as are the illustrations, which reveal the nutmeg-like appearance of
hepatic chronic passive congestion. The gross morphologic appearance is caused
by congested centrilobular areas alternating with pale portal areas.

2.The answer is B.

A clear, straw-colored fluid with low protein and low specific gravity is a
transudate, and the term hydrothorax refers to the accumulation of a significant
volume of transudate within the pleural cavities (to be detected by chest
radiograph, about 200 to 400 mL of pleural fluid must be present). The most
common cause of hydrothorax is cardiac failure, which may be either unilateral or
bilateral (bilateral is more common). It is incumbent on the clinician to distinguish
pleural transudates from exudates, because the causes of each are quite
different.

3.The answer is E.

The history is typical of amniotic fluid embolism, one of the major obstetric
causes of disseminated intravascular coagulation (DIC). Other obstetric
complications associated with DIC include retained dead fetus and abruptio
placentae (premature separation of the placenta). Non obstetric causes include
neoplasms or tissue damage from infection, immunologic mechanisms, or trauma.
Neoplastic causes include tumors of the lung, pancreas, prostate, and stomach,
and FAB M3 acute myeloblastic (promyelocytic hypergranular) leukemia. Tissue
damage can result from trauma such as lung surgery, from hemolysis or hemolytic
transfusion reactions, and from inflammatory causes, such as gram-negative
sepsis and immune complex disease.

4.The answer is E.
The history is that of pulmonary embolism and infarction, a danger of
immobilization and the postoperative state. The infarct consists of an area of
coagulative necrosis with superimposed hemorrhage, a combination referred to
as a hemorrhagic, or red, infarct. Red infarcts are typical of tissues with a
redundant arterial blood supply. Prominent examples are the lung with its double
circulation from the pulmonary and bronchial arteries and the gastrointestinal
tract with its multiple anastomoses between branches of the mesenteric artery.
When a portion of the blood supply is obstructed, other portions remain patent,
which can lead to hemorrhage into the infarcted area.

5.The answer is C.

The findings are consistent with occlusion of the middle cerebral artery, the most
common site of arrest of arterial emboli in branches of the carotid artery. Such
emboli usually arise from a mural thrombus in the left atrium or left ventricle. Left
atrial mural thrombi are especially associated with mitral stenosis with atrial
fibrillation. Mural thrombi in the left ventricle are caused by myocardial
infarction. Thrombi at the junction of the internal and external carotid arteries are
a cause of thrombotic brain infarcts and can also be a site of origin of emboli.

6.The answer is E.

The diagnosis is septic shock, most likely a result of gram-negative sepsis


originating from a urinary tract infection. Gram-negative organisms contain
lipopolysaccharide in the outer membrane, which triggers the release of
cytokines, such as tumor necrosis factor, resulting in a cascade of events
culminating in increased capillary permeability and redistribution of circulatory
volume into the interstitium. Anaphylactic shock is a result of a type I
hypersensitivity. Cardiogenic shock often results from myocardial infarction.
Hypovolemic shock is a result of blood or fluid loss. Neurogenic shock can result
from spinal cord injuries.

7.The answer is D.
Glycoprotein IIb-IIIa inhibitors prevent the action of the corresponding platelet
surface receptor glycoprotein complex, which is required for formation of
fibrinogen bridges between adjacent platelet

8.The answer is C.

The patient has fat embolism syndrome, which is characterized by pulmonary


distress, cutaneous petechiae, and various neurologic manifestations. Fat
embolism is a well-known complication of fractures of long bones, such as the
femur, and other bones with abundant fatty marrow. On fracture, marrow fat can
enter the circulation, and small fat droplets can lodge in vessels of the skin, lung,
and microvasculature of the brain, resulting in the clinical manifestations of this
disorder.

9.The answer is E.

Thromboxane A2(TxA2) promotes platelet aggregation, as does ADP.Aspirin


irreversibly inhibits the enzymes cyclooxygenase 1 and 2 and thereby the
synthesis of TxA2, thus inhibiting platelet aggregation, which is thought to be an
important early step in atherogenesis. A negative but apparently unimportant
consequence of aspirin prophylaxis is the parallel inhibition of synthesis of the
antiaggregant endothelial PGI2, also a product of the cyclooxygenase pathway.

10.The answer is B.

The clinical description is characteristic of pulmonary infarction, which, in turn,


most often results from thromboembolism originating from thrombosis in the
lower extremity veins. Because venous thrombosis is associated with impaired
blood flow, this condition is particularly characteristic of immobilization, which is
often seen in elderly, debilitated, or chronically bedridden persons.

11.The answer is B.

The patient has cirrhosis of the liver secondary to chronic alcoholism. A


prominent manifestation of this disorder is decreased hepatic synthesis of
albumin, the most significant contributor to plasma oncotic pressure. In addition,
ascites is associated with increased sodium and water retention because of
stimulation of the renin-angiotensin system. Also, hydrostatic forces (because of
intrahepatic scarring and partial obstruction of the portal venous return) result in
fluid transudation and increased secretion of hepatic lymph.

12.The answer is B.

“Heart failure cells” are intra-alveolar hemosiderin-laden macrophages and are


indicative of marked chronic passive congestion of the lung. Red cells leak from
congested alveolar capillaries into the alveoli, where they are engulfed and
degraded by macrophages.

13.The answer is A.

von Willebrand factor is required for platelet adhesion to the subendothelium of


damaged blood vessels.

14.The answer is A.

The combination of a prolonged activated partial thromboplastin time (APTT), a


positive VDRL test for syphilis, recurrent thromboses (arterial or venous), and
spontaneous abortion is highly suggestive of the antiphospholipid antibody
syndrome. As the name implies, antibodies directed at phospholipids are a
characteristic finding. Because of the prolonged APTT and frequent association
with systemic lupus erythematosus (SLE), the antibody has been referred to as the
“lupus anticoagulant,” a misleading term because affected subjects have a
thrombotic rather than hemorrhagic diathesis and not all subjects have SLE. The
term primary antiphospholipid antibody syndrome is used when there is no
evident underlying disease. It should be contrasted to secondary antiphospholipid
antibody syndrome, in which the patient has a well-defined autoimmune disorder
such as SLE.
BLOCK 2 QUIZ
Home / My courses / Pathology I / Block Two / Quiz 2 Pathology -1

Started on Sunday, 28 February 2021, 11:34 PM


State Finished
Completed on Sunday, 28 February 2021, 11:37 PM
Time taken 2 mins 54 secs
Marks 10.00/10.00
Grade 100.00 out of 100.00

Question 1
A genetic study is done on a newborn that indicates the child was born with a single functional
Correct
allele of a tumor suppressor gene. At the age of five, the genetic study was repeated only to find
Mark 1.00 out that the remaining normal allele was lost through a point mutation. As a result, the ability to halt the
of 1.00
transition from G1 to the S phase of the cell cycle is lost. If he lives till the age of twenties, which
one of the following additional malignancy would most likely to arise in this child?

Select one:
a. Malignant cartilaginous tumor

b. Malignant carcinoma

c. Malignant bone tumor !

d. Benign bone tumor

e. Malignant cartilaginous tumor

Your answer is correct.


The correct answer is: Malignant bone tumor
Correct
Marks for this submission: 1.00/1.00.
Question 2
A 38-year-old female undergoes a routine Pap smear examination. The smear is reported as
Correct
“Atypical cell seen with a high N/C ratio, hyperchromatic and pleomorphic nuclei.” A cervical biopsy
Mark 1.00 out is advised. The biopsy shows a tumor composed of sheets and nests of malignant squamous cells
of 1.00
but the basement membrane is intact. Which of the following is the best diagnosis of her lesion?

Select one:
a. Invasive squamous cell carcinoma, poorly differentiated

b. Invasive squamous cell carcinoma, well-differentiated

c. Mild dysplasia

d. Severe dysplasia !

e. Invasive adenocarcinoma, well-differentiated

f. Moderate dysplasia

Your answer is correct.


The correct answer is: Severe dysplasia
Correct
Marks for this submission: 1.00/1.00.
Question 3
A 36-year-old healthy man has a routine check of his health status. He has no symptoms. A chest
Correct
x-ray shows a peripheral 2.5 cm diameter in the left upper lobe of the lung. Biopsy findings are
Mark 1.00 out given. Which of the following characteristics expected this lesion?
of 1.00

Select one:
a. Severe dysplasia

b. Disorganized overgrowth of native tissue !

c. Disorganized overgrowth of ectopic tissue

d. Normal growth but ectopic tissue

e. Pleomorphism

Your answer is correct.


The correct answer is: Disorganized overgrowth of native tissue
Correct
Marks for this submission: 1.00/1.00.

Question 4
A 38-year-old woman sees her physician for a routine physical examination. A 3.5 cm non-tender
Correct
mass is palpated in her left breast. On physical exam another three masses of 2-4 cm non-tender
Mark 1.00 out masses are palpable in the left axilla. The further evaluation confirmed that she has mass lesions in
of 1.00
the ovaries also. Which of the following genetic mutations can lead to the above condition?

Select one:
a. TGFβ

b. RAS

c. BRCA 1 !

d. MYC

Your answer is correct.


The correct answer is: BRCA 1
Correct
Marks for this submission: 1.00/1.00.
Question 5
A 52-year-old man has had increasing fatigue for the past 6 months. On evaluation including
Correct
cytogenetic analysis of cells obtained via bone marrow aspiration reveals a diagnosis of chronic
Mark 1.00 out myeloid leukemia with marked proliferation of immature and mature myeloid cells and t(9:22)
of 1.00
translocation with a fusion gene formation. By which one of the following actions the fusion gene
causes abnormal proliferation of immature and mature myeloid cells in this patient?

Select one:
a. Decreased tyrosine kinase activity

b. Increased telomerase activity

c. Increased Caspase activity

d. Increased tyrosine kinase activity !

Your answer is correct.


The correct answer is: Increased tyrosine kinase activity
Correct
Marks for this submission: 1.00/1.00.

Question 6
A clinical study is performed on biopsy specimens obtained from patients who had a neoplasm.
Correct
Multiple features are studied which include different sizes and shapes of tumor cells, high N/C
Mark 1.00 out ratio, loss of polarity, tumor giant cells, and abnormal mitoses. Which one of the following will show
of 1.00
all the above features?

Select one:
a. Desmoplastic Tumor

b. Benign tumor

c. Pleomorphic tumor

d. Metaplastic tumor

e. Anaplastic tumor !

Your answer is correct.


The correct answer is: Anaplastic tumor
Correct
Marks for this submission: 1.00/1.00.
Question 7
A 24-year-old young female worried about the palpable lump in her left breast for three months.
Correct
Her physician notes a 2 cm firm, well-defined, movable mass located in the upper outer quadrant of
Mark 1.00 out her left breast on physical examination. Fine needle aspiration of this mass and subsequently
of 1.00
lumpectomy were done. Gross findings of the mass are given. Which of the following findings is
the best diagnosis?

Select one:
a.
Malignant mesenchymal neoplasm

b.
Benign neoplasm !

c.
Metastasis

d.
Malignant epithelial neoplasm

Your answer is correct.


The correct answer is:
Benign neoplasm
Correct
Marks for this submission: 1.00/1.00.
Question 8
A 23-year-old young male sees his family physician due to altered bowel habits. On physical
Correct
examination, there are no lesions noted on digital rectal examination, but his stool is positive for
Mark 1.00 out occult blood. A colonoscopy is performed and followed by excision of his intestine. Gross findings
of 1.00
of the specimen are given. Which of the following most appropriate molecular abnormality seen
in this patient?

Select one:
a. APC – Oncogene

b. APC – DNA repair gene

c. APC – Timor Suppressor gene !

d. APC – Apoptosis Regulating gene

Your answer is correct.


The correct answer is: APC – Timor Suppressor gene
Correct
Marks for this submission: 1.00/1.00.
Question 9
A 40-year-old woman has had a feeling of abdominal discomfort for the past 8 months. On pelvic
Correct
examination, there is a right adnexal mass. An abdominal CT scan demonstrates a 7 cm cystic mass
Mark 1.00 out involving the right ovary with small areas of calcification. The uterus is normal in size. The right
of 1.00
fallopian tube and ovary are removed surgically. Grossly, the mass on sectioning is filled with
abundant hair and sebum. Microscopically, the mass has glandular spaces lined by columnar
epithelium, a squamous epithelium with hair follicles, cartilage, and dense connective tissue. Which
of the following statements most appropriate regarding this type of neoplasm?

Select one:
a.
Malignant mesenchymal tumor

b.
Desmoid tumor

c.
Benign epithelial tumor

d.
Tumor-derived from more than one germ layer !

e.
Tumor-derived from one germ layer

Your answer is correct.


The correct answer is:
Tumor-derived from more than one germ layer
Correct
Marks for this submission: 1.00/1.00.

Question 10
A study is performed to analyze characteristics of malignant neoplasms in biopsy specimens. The
Correct
biopsies were performed on patients who had palpable mass lesions on physical examination. Of
Mark 1.00 out the following microscopic findings, which is most likely to indicate that the neoplasm is malignant?
of 1.00

Select one:
a. Necrosis

b. Metastasis !

c. Intact basement membrane

d. Tumor giant cells

e. Invasion

Your answer is correct.


The correct answer is: Metastasis
Correct
Marks for this submission: 1.00/1.00.
Neoplasia

1. 2nd leading cause of death in the USA


a. cancer
2. Treatment response
a. Usually expressed asà 5-year survival rate
3. Components of tumors
a. Parenchyma – clonal neoplastic cells
i. For Classification and behavior based on Parenchyma
b. Stroma -
i. connective tissue & Vessels
ii. Determines Growth & spread of tumors
4. Desmoplasia
a. Abundant collagen
b. Scirrhous- stony hard
5. Hamartoma
a. Disorganized, excess tissue but indigenous/native to the particular site,
6. Choristoma
a. congenital anomaly, Heterotopic rest of cells, Ectopic and normal tissue (not a
neoplasm), MC example -pancreatic tissue in the submucosa of the GIT
7. Benign & Mesenchymal
a. Cell origin + oma (Chondroma)
8. Malignant & Mesenchymal
a. Cell origin + sarcoma (Chondroma)
9. Benign & Epithelial
a. Adenoma à glandular/acinar pattern
b. Papilloma à finger-like growth
c. Polyp à fist-like growth
d. Cystadenoma: glandular/acinar pattern with cystic spaces
e. Papillary cystadenoma à finger-like growth and acinar pattern along with cystic spaces
10. Malignant & Epithelial
a. Adenocarcinomaà Malignancy with a glandular growth pattern
b. Squamous cell carcinomaà squamous cell differentiation
c. Mixed tumors àPleomorphic adenoma, derived from single germline
d. Teratoma à derived from more than one germ layer { Dermoid cyst/Ovarian Cystic
Teratoma), Totipotential cells
11. Sarcoma
a. Malignant & Mesenchymal neoplasm, deep-seated, advanced stage at the time of
diagnosis, hematogenous spread, Poor prognosis (as compared to carcinoma)
12. Carcinoma
a. Malignant & Epithelial neoplasm, superficial, early in stage at the time of diagnosis,
Lymphatic spread (commonly), good prognosis (as compared to Sarcoma)
13. Differentiation
a. Morphological and functional resemblance of tumor cells to normal parenchymal cells
Neoplasia

b. Benign tumors à well-differentiated cells (Grade -1)


c. Malignant tumors à range from well to Undifferentiated (G-1 to G-IV)
14. Anaplasia -Hallmark of malignancy = Anaplasia
15. Features of differentiation & Anaplasia
a. Cellular pleomorphism – changes in cellular size and shape
b. Nuclear pleomorphism - ↑DNA content, Hyperchromasia, irregular chromatin
distribution, irregular nuclear borders, ↑N: C ratio
c. Prominent nucleoli
d. Atypical or abnormal mitoses
e. Loss of Polarity or orientation
f. Tumor Giant cell (very large or cells with multiple nuclei)
g. Hemorrhages, Central Necrosis
h. Dysplasia - Disordered growth of pleomorphic epithelium
i. Severe dysplasia or Carcinoma –in-situ is differentiated from Invasive carcinoma by
intactness of basement membrane
i. Secretions (in case of both benign & malignant tumors)
i. Squamous cell Ca.à keratin
ii. Hepatocellular carcinoma à Bile
j. New or unexpected production of substances-paraneoplastic
k. Small (Oat) cell ca. of Lungà ACTH (Cushing's syndrome); ADH (SIADH)
16. Precancerous conditions
a. Metaplasia – Intestinal (columnar) metaplasia in Barrett's Esophagus, Squamous
Metaplasia of the lung in smokers
b. Dysplasia - Disordered growth of pleomorphic epithelium
c. Chronic Inflammations – HBV, HCV infections
17. The rate of Tumor growth is
a. determined by doubling time (proliferation) and death of tumor cells (apoptosis)
b. Rapidly growing/ High GF à 20% (Leukemia, Lymphoma, Lung Ca. - Small cell type)
Burkitts is fastest
c. Slow growing/ Low GF à 5% (Cancers of Colon, Breast)
d. General rules - Benign & well-differentiated malignant tumors àslow growing
e. Malignant tumors (G2,3,4) – grow rapidly or erratic growth
f. Exceptions –
i. Leiomyoma (Fibroid – MC site – Uterus) – benign but grows rapidly during
pregnancy (estrogen effect) and undergoes red degeneration (hemorrhagic
infarction)
ii. Spontaneous remission or benign transformation of malignancies
àNeuroblastomas (to Ganglioneuromas- the benign counterpart), Malignant
melanoma

18. Features of
Neoplasia

a. Benign tumor – encapsulated (exception – Hemangiomas); slow growing (Low GF-5%);


well-demarcated, cohesive masses; No invasion; no metastasis; easily palpable & easily
removable
b. Malignant tumor – un-capsulated (exception Thymomas); fast or erratic growth rate;
ill-defined, invasion and metastasis –present; difficult to remove and Need wide excision

19. Metastasis
a. The most reliable feature to differentiate malignant from benign tumors
b. All cancers can metastasize
c. Exceptions
i. Gliomas –malignant but don't metastasize (brain has no lymphatics)
ii. Basal cell carcinoma( of skin) à rarely/don’t metastasize
iii.
20. Significance of malignancies with metastasis
a. More aggressive
b. More rapidly growing àLarger neoplasms
c. Less likelihood of cure
d. High chance of morbidity and mortality
21. Features of metastasis
a. Multiple
b. Superficial
c. Central Umbilicated (like umbilicus – due to central necrosis)
22. Pathways of Metastasis
a. Seeding of body cavities(peritoneum) Ovarian Ca.
b. Lymphatic spread = MC in carcinomas; skipped metastasis” – if immediate drainage
group of LN skipped and metastasized to next level(Malignant melanoma)
c. Hematogenous spread = MC in sarcomas; Veins > arteries; MC sites – Lungs, Liver (Caval
and portal circulations);
d. Malignancies that invade veins àRCC (renal vein); HCC(hepatic vein)
e.
23. Sentinel LN
a. The first node in a regional lymphatic basin that receives lymph flow from the primary tumor.”
b. Procedure to map SLN
i. injection of radio-labeled tracers and blue dyes 9Methylene blue)
ii. frozen section and examination of section (Intra-operative)
c. Significance - avoid unnecessary morbidity and guide in appropriate therapy & SLN is used in
detecting the spread of melanomas, colon cancers

24. Epidemiology of Cancer


Neoplasia

a. Study of cancer patterns in populations


b. Helps in understanding the role of smoking in lung cancer, high dietary fat and colonic
cancers, knowledge of preneoplastic conditions, and interrelations of genetics and
environment in causing cancers, etc.,
25. Impact of cancers
a. 2nd MCC of all the deaths in the USA
b. MC cancers in Men – Prostate (25%), Lung (15%), Colorectal (10%)
c. MC cancers in women – Breast (26%),Lung(14%), Colorectal (10%)
d. MC cancer-related deaths are due to Lung {(in both males (31%)and females26%)}
e. Risk of dying with cancer in the USA – one in 5
f. Cancer deaths- 23% of all the deaths in the year 2008
g. Half of the cancer diagnoses and cancer deaths are due to Lung, Breast, Prostate,
Colorectal
h. Cancer with the decrease in incidence à stomach and cervix
i. Cancers with increase deaths à Lung cancer of women, Liver and bile duct in men (due
HCV)
26. Racial difference
a. The largest decline in cancer mortality in American blacks
b. The lowest frequency of common cancers of the USA in à Hispanics
27. Environmental factors (slides 15, 16 of PPT 3)
a. Very significant in sporadic cancers – 65%
b. Stomach ca à 7-8 times higher in Japan (as compared to the USA)
c. Skin cancers à high in Newzeland and Australia
d. Occupational – Asbestos, Vinyl chloride, 2-Napthalamine
e. Obesity – 52% risk in males and 62% in females (Prostate, Breast, Uterus)
f. Tobacco smoke – Lung cancers
g. Sexual practices à cervical Ca.
28. Age
a. Cancer age – 50s’
b. Blastomas – MC in young children
c. Acute Leukemias and Gliomas – older children and adolescents
29. Genetic factors(heritable) –slides 21 and 22 very important
a. Riske is 32%
b. Inherited mutations lead to cancer – 10%
30. Non-hereditary predisposing conditions
i. Regenerative
ii. Metaplastic
iii. Hyperplastic
iv. Dysplastic
v. Chronic inflammatory
b. Examples
i. Chronic gastritis à Gastric ca
Neoplasia

ii. Solar keratosis à Skin cancers


iii. Ulcerative colitis à Colonic cancer
iv. Leukoplakia à Oral, genital squamous cell carcinomas
v. Villous polyps of intestinesà Adenocarcinoma of the colon
31. Molecular basis of cancers
a. Clonality
i. MC method used to determine clonality is polymorphic locus of ***AR gene
ii. For clonality in B- and T-cell lymphomas - Ig receptor (in B cell) and T-cell receptor
gene (T-cell) rearrangements
b. Hallmarks of Cancer à Essential alterations
i. Self-sufficiency in growth signals
1. No need for external stimuli
ii. Insensitivity of growth inhibitory signals
1. TGFβ inhibits the proliferation of normal cells
2. Cyclin-Dependent Kinase Inhibitors (CDKI)
iii. Evasion of Apoptosis
1. inactivation of p53)
2. activation of anti-apoptotic genes
iv. Defects in DNA repair
1. NER (Nucleotide Excision Repair)
v. Limitless replication potential
1. telomerase reactivation
vi. Sustained angiogenesis
1. VEGF
vii. Ability to invade & metastasis
1. ↑cancer deaths and morbidity
32. Proto-oncogenes
i. Essential for the growth and development of normal cells
ii. Any of the genes encoding Growth Factors, Growth Factor receptors Proteins,
Kinases – signal pathways, Gene Regulatory Phosphoproteins, and transcription
factors are proto-Oncogenes
iii. Mutations of Proto-Oncogenes produce Oncogenes
iv. Oncogenes make cells self-sufficient and multiply without signals
v. Slides (tables) -8,9,13,22,25,32 important
vi. mutations of ERBB1 à activation of EGFR (RTK) lead to Lung Adenocarcinoma
vii. mutations of ERBB2à activation of HER2 (RTK) lead to Breast Ca.
viii. deletion on Ch.5: lead to a fusion of part of ALK gene (RTK activity) with that of
EML4 à Lung Adenocarcinoma
ix. RAS mutations in mutations in pancreatic, Cholangiocarcinoma – 90%, Colon,
Endometrial Ca.--> 50%, Lung (Adeno), Myeloid Leukemias -à 30%
x. NF-1 (Neurofibromatosis -1 à LOF of GAP (GTPase activating proteins)
Neoplasia

xi. Mutations of BRAFà activate TFs’,100% of cases of Hairy cell Leukemia (B cell
neoplasm), 60% of Melanoma have Mutations of BRAF, Specific Anti -BRAF
antibodiesà Very successful in treating melanomas, Oncogene addiction – only
melanomas with BRAF mutations show best treatment response
xii. PI3K through AKT acts on mTOR, BAD, FOX
1. PI3K is negatively regulated by PTEN (TSG)
2. GOF mutations of PI3K seen in30% of cases of Breast Ca
3. PTEN mutations( LOF) cause Endometrial Ca
xiii. NonRTK (Non-Receptor Tyrosine Kinase) BCR: ABL reciprocal translocationsà
CML, ALL (Leukemias); JAK/STAT pathway- JAK2 mutations seen in Myelo
Proliferative Disorders (MPD)
xiv. MYC -is the Master transcription regulator of cell growth, SNPs strictly control MYC
concentration, MYC cell translocations seen in Burkitt's, MYC amplifications in Ca.
Breast, Colon, Lung
xv. P16CDKN2A ( CDKIs') germline mutations seen in 75% of Ca. Pancreas, 55% of
Glioblastomas, 50% of Ca. Esophagus, 35% of ALL

33. Tumor suppressor genes (TSG) involved in


i. inhibition of cell proliferation
ii. Cell differentiation
iii. Prevention of uncontrolled cell growth (in both normal cells and tumor cells)
iv. LOF (Loss Of Function )mutations à uncontrolled cell growth
v. Proto-Oncogene mutation in the cell without TSG mutations à cells undergo
quiescence (permanent cell cycle arrest)àApoptosis
vi. Genotoxic stress lead to increased TSG à↑ p53 production à Apoptosis
34. RB gene (chromosome 13 q14)
a. First TSG discovered
b. Knudson “two-hit” hypothesis
c. active hypophosphorylated in cell quiescence & inactive
hyperphosphorylated state in G1/S cell cycle transition
d. Effects of RB gene
• Blocks E2F-mediated transcription
• Controls stability of p27 (cell cycle inhibitor)
• Induce senescence
e. Mutations in RB
• Germline loss or mutations à Familial retinoblastoma (bilateral) and
Osteosarcoma (2nd Cancer), Childhood type, 40% of retinoblastomas
• Somatic loss or mutations à sporadic retinoblastoma (unilateral), adult type,
60% of retinoblastomas
• HPV E7 of HPV type 16à binds E2F site of RB pocket à release E2F from RB
control àCervical cancers (Squamous cell carcinoma – SQ.CC)
Neoplasia

35. Tp53 gene (chromosome 17p13.1)


a. Guardian of the Genome, “molecular policeman” /traffic cop
b. MC target for genetic alteration(> 50%) in human tumors
c. Every type of cancer has homozygous loss of p53 2/2
d. p53 mutations are acquired or in somatic cells (unlike RB)
i. Exception - Li-Fraumeni syndrome– inherited mutations of one p53
e. 80% mutations present in DNA-binding domain of p53 protein
f. MDM2 and MDMX proteins regulate p%3 ( by the degradation of p53)
g. E6 protein of HPV causes degradation of p53p protein
h. Normal p53 blocks neoplastic transformation by
i. temporary cell cycle arrest (quiescence)
ii. permanent cell cycle arrest (senescence)
iii. programmed cell death (apoptosis)
i. Genes triggered by p53 cause
i. cell cycle arrest (temporary or permanent)
ii. Apoptosis

j. p53 activates the mir34 family of miRNAs (mir34a–mir34c)


i. miRNAs - induce growth arrest and apoptosis
ii. Targets of mir34s àpro-proliferative -cyclins, and anti-apoptotic -BCL2
k. p53 senses DNA damage & determines the adequacy of DNA repair by
i. Ataxia-telangiectasia mutated (ATM) and ataxia-telangiectasia related
(ATR) genes
ii. ATM & ATR à phosphorylatep53 and DNA-repair proteins
l. Events following DNA damage
i. Early in G1phase
1. Preventing the entry of cells into G1 by P21 (which inhibits cyclin-
CDK)
ii. Late in the G1 phase
1. Activates CDK inhibitor CDKN1 A (p21) by Inducing GADD45
(growth arrest and DNA damage)
m. If DNA damage is reparable,
i. p53 ↑MDM2 proteinà MDM2 degrades p53 àreleasing cell cycle block
n. If DNA damage is irreparable,
i. senescence & apoptosis by p53
o. Therapeutic implications of p53 mutations
i. Cancers with normal p53 à respond to Irradiation and chemotherapy
(RT & CT), DNA damage of tumor cells by CT/RTà p53 mediated
senescence & apoptosis of tumor cells, Examples-testicular carcinomas &
childhood ALL
Neoplasia

ii. Cancers with mutated p53 à Resistant to RT & CT, DNA damage of
tumor cells by CT/RTà mutated p53 à no senescence & apoptosis of
tumor cells, Examples-lung & colorectal cancers
p. P63 (Function - differentiation of stratified Squamous cell differentiation)and p73 (strong
pro-apoptotic) are other members of the p53 family
36. APC/β-Catenin Pathway (Ch.5q21)
i. Component of the WNT signaling pathway
ii. Down regulate the β-Catenin activity
iii. In the presence of WNT signalingà Blocks APCà β-catenin to translocates into
nucleus à cellular proliferation by c-MYC, cyclin D1
iv. In the absence of WNT signaling, à APC activates ubiquitin + Proteasome pathwayà
degrades of β-catenin
v. Germ-line mutations of APC àFAP (familial adenomatous polyposis)
1. For Diagnosis – at least 100 polyps (by age of 20s’)
2. If untreated – 100% risk of Adeno ca. arise from polyps by age of 50s’
vi. Sporadic mutations of APC à associated with 80% of Colorectal Ca.
vii. Adenoma à Adenocarcinoma (multistep process)
viii. Mutations in the β-catenin & E-cadherinà Hepatoblastomas (50%) and HCC (20%)

ix. E- cadherin
1. For cell-cell contact
2. SNAIL down-regulate E-cadherin (mutations à metastasis)
3. Germline mutations of the E- cadherin à Ca. stomach
37. INK4a/ARF
i. Encodes two protein productsà p16/INK4a CDKI (controls RB), p14/ARF (activates
the p53 pathway)
ii. Mutations or silencing of p16, p14 à impacts RB and p53 pathways
1. Mutations of p16 à bladder, head & neck tumors, ALL, and
cholangiocarcinomas
2. Hypermethylation of p16 à Cervical Ca.
38. TGF-β Pathway
A. Inhibits cell proliferation by repression of c-MYC, CDK2, CDK4, and Cyclins A and E
B. TGF-β activation à phosphorylation of R-SMADs; R-SMADs + SMAD-4 in the
nucleus cause à transcription of CDKIs p21 and p15 (inhibitors of the cell cycle)
C. Mutations in the TGF-β àaffect type II TGF-β receptor lead to àca. colon, stomach,
and endometrium
i. Inactivation of SMAD4à pancreatic ca.
ii. Mutation components of TGF-βàpancreatic ca. (100%); colon Ca. (83%)
iii. Tumor cells use TGF-β–induced Pathway for
1. immune system suppression/evasion
Neoplasia

2. promotion of angiogenesis
39. PTEN - Phosphatase& Tensin (ch. 10q23)
i. Acts through PI3K/AKT pathway (Proliferation/cell survival)
ii. Inherited mutations (AD)àCowden syndrome
1. Cowden syndrome benign skin growths – Hamartomas & epithelial cancers
(breast, endometrium, and thyroid)
iii. Acquired mutations (LOF) à up-regulate PI3K/AKT pathway (Proliferation/cell
survival)
40. NF1
i. NF1 gene encodes protein àNeurofibromin
ii. Neurofibromin proteinàGTPase-activating Protein (GAPs’)
iii. Disease à Neurofibromatosis type 1 (Lisch nodules -eye, multiple Neurofibromas,
Plexiform Neurofibromasà risk of neurofibrosarcomas, Cafe-au-lait on the skin)
41. NF2
i. NF1 gene encodes protein à Neurofibromin 2 or merlin
ii. Disease à Neurofibromatosis type 2 (acoustic nerve schwannomas (bilateral)
iii. Somatic mutations à meningiomas and ependymomas

42. VHL (von Hippel-Lindau gen- (ch. 3p)


i. HIF1α + VHL proteinà activate ubiquitination + proteasomal degradation
ii. Germline mutations à RCC, pheochromocytomas, hemangioblastomas (CNS),
retinal angiomas, and renal cysts
iii. Sporadic mutations VHL lead to à RCC
43. Patched (PTCH)
i. PTCH1 and PTCH2 genes encode a cell membrane protein (PATCHED) which is a
receptor for the Hedgehog family that regulates TGF-β and PDGFRA and PDGFRB
ii. Germline PTCH mutations àGorlin syndrome
1. Gorlin syndromeàNevoid basal cell carcinoma(BCC) syndrome
iii. Somatic PTCH mutations
1. 20% to 50% of sporadic cases of BCC
2. 50% of mutations caused by UV exposure
44. Apoptosis
a. Basic function – direct cells with genomic damage to programmed cell death
b. Pathways
i. Extrinsic – Fas + FasL
ii. Intrinsic – Leak of Cytochrome “C” from mitochondria
c. Apoptosis regulating proteins
i. Pro – BAX, BAK
ii. Anti- BCL2. BCL-XL
Neoplasia

iii. Balancing – BAD, BID, PUMA


d. Initiation & Execution by
i. Caspases (C- Cytokine protease, aspase- cleave after aspartic acid)
ii. Initiators –Caspases -8,9
iii. Executioner –Caspase 3
e. FLIP
i. The protein produced by tumors
ii. Prevents activation of Caspase 8 (affects which pathway?)
f. t(14:18) – upregulation of BCL2 of BCL2 family, Anti-apoptotic, Follicular Lymphoma (MC
slow/indolent B-cell Lymphoma in the USA (tumor grows due to survival of neoplastic
cells), Low GF (5%), CT & RT not very effective,
45. DNA repair genes

Repair gene Disease/condition due to defects


Mismatch repair HNPCC
Nucleotide Excision Repair (NER) Xeroderma pigmentosum
Recombination repair Bloom’s syndrome

a. Not Oncogenic by themselves but mutations lead to ↑ tumor susceptibility following


Oncogenic or TSG mutations, Spell checkers
b. Mutations cause à Genomic instability
c. Mutator Phenotype – this phenotype is the result of mutations in genes that
function in the maintenance of genomic stability.
d. HNPCC
i. Mismatch repair genes – MSH2, MLH1
ii. Mutations of MSH2 or MLH1 à replication errors/microsatellite instability
iii. Familial colonic carcinomas (cecum & proximal colon)
iv. Different from FAP à caners develop
1. At younger age (20s’ as compared to 50s’ ),
2. Cancer does not arise from exciting polyps (Polypà cancer feature of
FAP)
3. low risk – 2-4% (100% in FAP)

e. Xeroderma Pigmentosum (XP)


i. Defect in NER genes
ii. Pyrimidine cross-links formed by UVB rays are not repaired
iii. 2000 times risk of skin cancers if exposed to the sun
f. BRCA genes
i. Involved in transcription regulation
Neoplasia

ii. BRCA-1 mutations: female - Breast Cancer, ovarian Cancer; Male – prostatic
cancer, in both males and females- Colonic cancer
iii. BRCA-2 mutations – male breast cancer
g. Ataxia Telangiectasia
i. Senses double-stranded DNA breaks + activates p53
ii. 1% population – heterozygous
iii. Increase risk of leukemias, lymphomas
h. Bloom syndrome
i. Recombination repair gene (helicase) defects
ii. Mutations lead to - leukemias, GI cancers
i. Fanconi’s Anemia
i. AML (Acute Myeloid Leukemia)
ii. Aplastic anemia
46. Telomerase
a. Somatic cells – no telomerase; after 60-70 cell doublings – telomere shortening (at
chromosome ends by double-stranded breaks)
i. Senescence (by p53) – mitotic catastrophe (bridge-fusion and breakage of
chromosome ends – massive cell death due to absence of telomerase
b. Stem cells (bone marrow) – have telomerase- continuously dividing
c. Tumor/cancer cells – reactivation of telomerase – 95% of cancers have increased
telomerase activity, Tumor cells are immortal
47. Angiogenesis
a. Essential for the supply of nutrients (like normal cells)
b. Stimuli- hypoxia-HIF1a -VEGF, b-FGF, PDGF
c. Helps in growth and metastasis
d. Tumor vessels – irregular, tortuous, leaky, uncontrolled growth
e. Uncontrolled growth – due to lack of inhibition by thrombospondin 1 of p53 mechanism
f. Anti-angiogenic Rx – endostatin, bevacizumab (Anti-VEGF antibody)
48. Invasion & Metastasis
1) Detachment/loosening – downregulation of E cadherins & β -catenins
2) Receptor-mediated attachment to laminin and fibronectin
3) Degradation of ECM –Proteases (MMPs’ – Matrix Metallo Proteases 9,2) cleave IV
collagen of BM
a. Benign tumors – have low concentrations of MMPs in blood
b. Malignant, invasive tumors– concentrations of MMPs in blood
4) Adhesion molecules that help in homing of tumor cells- CD44
5) Tumor emboli protected in circulation by binding to Platelets (PLTS)
6) Prostatic Ca metastasis is bone (lumbar), Lung cancer to– Brain and adrenals,
Neuroblastoma to – liver and bones
Neoplasia

49. Organ tropism


a. Adhesion molecules expressed on– tumor cells
b. Ligands on endothelial cells of target organ vessels
c. Chemokines for tumor cells –IGF (Insulin Growth Factor) I &II by target organ tissue
50. Stroma
1) Immune/inflammatory cells - carcinogenic, survival, the spread of cancers
2) Macrophages – secrete metastasis promoting factors
3) Fibroblasts – Desmoplasia, Role studied in tumor growth in breast cancer
51. Warburg effect
i. Aerobic glycolysis
ii. For growth advantage in a hypoxic environment
iii. Hypoxia à HIF1a à angiogenesis by stimulating VEGF, bFGF
iv. Used in PET scan to detect tumors location and spread of tumor (metastasis)
52. Chromosomal alterations (table in slide 10 of PPT 7 – very important)
1) Most Common (MC) Chromosomal alterations – translocations
a. Hematopoietic neoplasms
b. Affect oncogenes
a. C-Myc – t(8:14) – the proliferation of tumor cells – Burkitt's (fastest growing tumor
in humans)
b. Overexpression of BCL2 of BCL2 family – t(14:18); survival of tumor cells – very slow-
growing tumor - Follicular Lymphoma (MC indolent (slow-growing) Non-Hodgkin's
Lymphoma –NHL in the USA)
c. Hybrid gene – ABL- BCR t(9:22), Philadelphia chromosome, à Tyrosine Kinase
activity – CML, oncogene addiction
d. Carcinoma with translocation –best studied in prostatic cancer – TMPRSS2 (21 q
22), and ETS family
nd
2) 2 MC chromosomal abnormality – Deletions
i. Non- hemopoietic (solid tumors)
ii. LOF, TSG – RB
iii. Del 17, 18, 5 – colorectal Carcinoma
iv. 3p- – small cell carcinoma of the lung
3) Gene amplification
i. Over-expression – oncogenes
ii. N-MYC amplification – two patterns
1. Doubling minutes
2. HSR
3. Present Neuroblastomas but both are not seen in the same patient
iii. Her2 (ERBB2) amplification is seen in – Breast Cancer
4) Epigenetic changes (methylation of DNA, Histamine modifications)
i. DNA methylation – affect mainly TSG, CDK1
ii. Silencing of Genes – examples
Neoplasia

a. BRCA-1 – female breast cancer


b. VHL – Renal Cell Carcinoma (RCC)
c. MLH-1, MSH -2 – colorectal cancer
5) MiRNA
i. Mediate gene silencing
ii. Downregulation of miRNA – lead to overexpression of BCL2 à Follicular
lymphomas
iii. Upregulation of miRNA – RAS, MYC
53. Carcinogens
1) Chemical
i. Initiator- initiates carcinogenesis, rapid & irreversible action, cause memorable
DNA change, not sufficient for tumorigenesis
ii. Promotor -non-tumarogenic, reversible, change not directly by damaged DNA,
can't act without initiation
a. Initiators
i. Direct-acting – no need of metabolic conversion, electrophiles- bind to damage
neutrophiles (DNA, RNA, proteins)
1. Primary target – DNA
2. Damage – non-lethal, non-enzymatic
ii. Indirect – require metabolic activation by
I. CYP450 (CYP1A1 for Benzopyrene of smoke) à Lung cancers
II. Glutathione -s-transferase for PAH à Bladder, Lung cancers
III. Both CYP450 & Glutathione -S-transferase are in polymorphic forms in the
White population
iii. A test used to know the carcinogenic potential of a chemical – AMES TEST
1. Organism used to a culture in AMES test – S. typhimurium
iv. Molecular fingerprinting/signature mutations studied in
1. Point mutations of p53 in Chinese, Africans exposed to aflatoxin B1 of
Aspergillus and association with HCC)
2. As compared to HBV with HCC
b. Promotors
i. Role- proliferation and clonal expansion of initiated (mutated cells)
ii. Examples – phorbol esters, hormones, phenols, drugs, bile, etc.
iii. Examples cont. phenobarbital – Block TGF-B, phorbol esters - protein kinase c
activators – transcription
c. Examples of Initiators
i. Alkylating agents – weak carcinogens
ii. PAH –Polycyclic Aromatic Hydrocarbons (smoke – MOST potent carcinogenic)
1. Tobacco smoke, animal fats, smoked food
iii. Aromatic amines ( B-naphthylamine of dye and rubber industries) à glucuronidase
of urinary bladder reconverts in carcinogen à Bladder cancer (Urothelial or
Transitional cell cancer –TCC)
Neoplasia

iv. Azo dyes – food coloring agents


d. Examples of Promotors
i. Endogenous – Hormones, Bile
ii. Exogenous
1. DES (Di Ethyl Stilbesterol – used in threatened abortion)
1) Mother has the risk of endometrial cancer, daughter – vaginal cancer
2. Fats - colon cancer
3. Exogenous Estrogens can cause liver adenomas
4. Tobacco has both initiators and promotors
e. The table on slide 18 – PPT-8 – very important
2) Radiation
a. UVB –
a. oncogene (GOF), TSG (LOF) mutations,
b. cause Pyrimidine dimers
c. People with XP – increase risk of skin cancers (due to defective NER genes)
b. Ionizing radiation – x-ray, y-rays
a. Cause Double-stranded breaks
b. Organs which highly susceptible – blood, thyroid
3) Microbial
DNA Viruses
I. EBV – B-cell lymphomas {(Hodgkin's Lymphoma, NHL, Burkitt's – Fastest
growing neoplasm, African/endemic type, common in kids, t(8:14), c-Myc
amplification, Naso-pharyngeal carcinoma}; LMP -1 viral oncoprotein causes –
up-regulation of NFKB
II. HPV - Squamous cell carcinomas; High-risk serotypes -16, 18; E7 viral
oncoprotein mainly affects Rb; E6 viral oncoprotein affects p53
III. HBV causes HCC; HBx viral protein affects p53

RNA Viruses
IV. HTLV-1 – non-neoplastic tropical spastic paraparesis; Tropism for CD4+ T cells;
1% of – T-cell leukemia/lymphoma; Tax region – increases transcription
V. HCV – chronic inflammationàleads to HCC
4) Bacterial
i. H. pylori – non-neoplastic – Chronic gastritis, Peptic ulcers (in the stomach,
Duodenum)
ii. Neoplastic – MALToma or gastric lymphoma
- chronic inflammation leads to Gastric carcinoma
-
54. Tumor antigens
a. Overexpressed normal cellular proteins
a. Tyrosinase – melanomas; Tyrosinase vaccine developed for
melanomas
Neoplasia

b. MAGE -1 – melanomas (37%), also present but not expressed in testicular tissues
i. Viral Oncogenes in tumor Antigens – HPV in Treatment of cervical cancer
55. Oncofetal proteins
a. Not present in adults
b. Positive in cancer cells
c. Helps with tumor diagnoses
d. CEA – GI cancers
e. AFP – HCC, yolk sac tumors (of testis and Ovaries)
f. Glycolipids, Glycoproteins, GM2 – melanomas (vaccines)
g. Mucins- “O” linked carbohydrate side chains - Used in diagnostic and therapeutic
studies
i. CA-125 – ovarian
ii. CA -19.9 – Pancreatic
iii. MUC-1 – breast
1. Uniquely expressed on ductal epithelium & sequestered from the immune
system
2. Candidate Vaccine
h. Differentiation Ag – present on the cell of origin; Helps with diagnosis (surface marker)
i. CD 20 – B cells
ii. Treatment- Anti CD20 for B cell lymphomas
56. Anti -Tumor mechanismdownreg of MHC1
a. NK cells – kills tumor cells that are not recognized by CTL 8; NKG2D protein –
recognize tumor cell receptors
b. Evasion of immune Surveillance
i. XLP (X-linked lymphoproliferative syndrome)
ii. Boys with severe chronic EBV infection, B cell Lymphomas

57. Cachexia - anorexia = decreased weight ( due to Both fat and muscle mass)
o Unlike Starvation where there is only loss of fat
o Cytokines responsible for Cachexia – TNF, IL-1, IFN-y
58. Paraneoplastic Syndrome – Table of slide 4
- Cushing’s, SIADH, Eaton Lambert à Small cell carcinoma of the lung
- Hypercalcemia, PTH like substance) à Squamous cell carcinoma of the lung
- Polycythemia – RCC
- Acanthosis nigricans – Gastric Ca.
- Hypertrophic osteoarthropathy, clubbing- lung ca.
- Trousseau syndrome - pancreatic Ca.
59. Grading & Staging
- Grading - G-I to G-IV
- Staging (TNM) - T -Size of Primary tumor, N- Lymph node involvement, M- metastasis
Neoplasia

- For prognosis, Staging is most important


- An exception is prostatic ca. ( Gleason’s grading is used in prostatic ca)
60. Lab diagnosis
1. IHC – for differentiation cancers
2. Cytokeratin – differentiate undifferentiated carcinomas from sarcomas
3. Vimentin, Desmin à sarcoma
4. S-100, HMB-45 à Melanoma
5. GFAP à Gliomas
6. CD-20 à B-cell
7. CD3 à T-cell
61. Tumor makers – Table of slide 16 most important
1) βhcG – choriocarcinoma
2) Calcitonin – medullary carcinoma of Thyroid
3) !FP – HCC, yolk sac tumors of gonads
4) CA- 19.9 – pancreatic ca
5) CA -125 – ovarian ca
6) Muc-1 - Breast ca
7) PSA – prostatic ca
8) CEA – colorectal Ca

---The end---
Keyla Galloza Acevedo

Neoplasias Part 1:
Desmoplastic tumors Ej. Scirrhous carcinoma of the breast
(Abundant Collagenous -Difficult surgical incision (Stroma/
Stroma) → hard on fibrous mass can entrap important
palpation structures)

Benign tumors: Ej.


1. Cyst-adenoma of ovary
-Usually end in “oma”
-Exception: Melanoma, 2. Follicular adenoma of the
Glioblastoma, Thyroid
Lymphoma,
Mesothelioma, (1)
Seminoma 3. Laryngeal Papilloma/polyp
(MCC is HPV infection)
-Well differentiated
(Morphology and 4. Verruca vulgaris (Squamous
Function resemble Papilloma)
parent cell), never
invade or metastasize, 5. Intraductal papilloma of the (2)
remain localized, always breast (MC arise in the
have a boundary Lactiferous sinus, Bloody
-Mucin containing tumor discharge from nipples,
cells usually have a branching fibrovascular core,
better prognosis subareolar mass is felt)
(3)
-Epithelial or 6. Ovarian Fibroma: Dense
Mesenchymal origin fibrous tissue

Epithelial: 7. Periosteal chondroma (Glassy


-Glandular origin= appearance-Key word)
adenoma
-Glands + finger like 8. Leiomyoma (4)
proyections=Papilloma
-Glands + cyst = -Very common benign tumor with
Cystadenoma erratic growth
-Smooth muscle neoplasm. Seen as
Mesenchymal: sharply circumscribed, discrete, round,
-Fibrocollagen=Fibroma firm, gray-white tumors
-Cartilage=Chondroma -Most are in the myometrium (5)
-Bone= Osteoma (intramural), but can be Submucosal
(endometrium), Subserosal (serosa)
and wandering leiomyoma
Keyla Galloza Acevedo

-Express estrogen receptors→ show


increased growth during pregnancy
and shows atrophic changes during
menopause
-Could lead to miscarriage because it’s
a space occupying lesion in the uterine
cavity (6)

(7)

(8)

Malignant tumors: Ej.


1. Squamous cell carcinoma:
-Can be well a. Esophagus
differentiated or poorly -Risk: smoking, drinking
differentiated (No hard spirits (vodka),
resemblance to parent malnutrition, HPV
cell, difficult to treat and -MC in the middle third of (a)
more aggressive), high the esophagus (2nd part)
risk of invasion and -Dysphagia for solids and
metastasis, malignant weight loss
tumors spread
-Well differentiated b. Ectocervix
adenocarcinoma shows
glandular pattern 2. Adenocarcinoma:
-Poorly differentiated a. Stomach
adenocarcinoma -High incidence in Japan
requires (flavored rice) (b)
immunohistochemistry -May be familial
to confirm epithelial -MC affects the pylorus
origin and
antrum>cardia>lesser>gre
A. Epithelial origin: ater curvature
Carcinoma
-Squamous lining: b. Pancreas
Keyla Galloza Acevedo

Squamous cell
carcinoma (Keratin c. Colon
Pearls)
-Glandular lining: d. Thyroid
Adenocarcinoma -Looks like a benign tumor
but shows capsular invasion (a)
B. Mesenchymal origin: -Well differentiated malignancy
Sarcoma (fleshy tumors) -Encapsulated microfollicles

-Osteoid: Osteosarcoma 3. Osteosarcoma:


-Chondroid: -MC in male adolescents
Chondrosarcoma -Aggressive synthesis of
-Fibrous tissue: disorganized osteoid or (b)
Fibrosarcoma mineralized bone
-SM: -MC affects long bones near
Rhabdomyosarcoma metaphyseal growth plates
-SmM: Leiomyosarcoma -Pain and soft tissue swelling
which may be triggered by
minor trauma (Patient will tell
you he feels pain and swelling)
-May lead to limb amputation (c)

4. Leiomyosarcoma

5. Synovial sarcoma:
-t(X;18)
-Cancer that can arise from different
tissues such as muscle, ligaments
-Often found in the leg, arm, foot,
wrist, ankle
(d)
-Also called Malignant synovioma

(5)
(3)

(4)
Keyla Galloza Acevedo

Special types of A. Divergent differentiation of a single


Neoplasia: neoplastic clone along two lines
-Mixed Salivary Gland tumor (Parotid
A. Pleomorphic glad)
adenoma (Both -Need to preserve Facial nerve (CN 7)
epithelial and during resection
mesenchymal origin) -Mixture of ductal and myoepithelial
cells
-Stroma may show myxoid (Wharton
jelly-Umbilical cord), hyaline,
chondroid and osteoid contents
B. Tumors representing more than one
germ layer (gonads)
-Ovarian teratomas are the most
common (Mature, Immature,
Monodermal)
a. Mature: Benign, women in
B. Teratoma: reproductive age group, unilocular
with solid and cystic areas
-Cyst wall is composed of Stratified
Squamous Epithelium with underlying
sebaceous glands, hair shafts and
adnexal structures (sweat glands,
apocrine, nerves)
b. Immature: Malignant, not common
-Embryonal tissue
c. Monodermal: Struma ovarii and
carcinoid , always unilateral
-Struma ovarii: composed of mature
functional thyroid tissue (can lead to
hyperthyroidism)
-Ovarian carcinoid: arise from
intestinal tissue, may produce 5-
hydroxytryptamine → Carcinoid
syndrome
C. Not quite a tumor but a
developmental error
C. Choristoma: -Ectopic rests of normal tissue (normal
tissue in abnormal locations)
Ej. Pancreatic tissue in stomach wall,
gastric mucosa in Meckels
diverticulum (Ileum)-remnants of
Vitelline duct → Rule of 2 (Symptoms
before age 2, 2 feet away from the
ileo-cecal valve)
-
Keyla Galloza Acevedo

D. Mass of disorganized, but mature


specialized cells or tissue indigenous to
D. Hamartoma: that site
-Benign tumor like malformation
-Tissues present are those specific to
the part from which arise
-Its growth is coordinated with that of
the surrounding tissues (no more
growth after adolescence)

Ej.Hamartoma in the lung (solitary


primary nodule) → Bronchial
epithelium, Tracheal epithelium,
Cartilage, mucous glands
-Seen in young people, cough + chest
pain, Coin lesion in X-ray

Lab Diagnosis for Cancer


Immunohistochemistry Cytokeratin Vimentin CD20 HMB 40/45- GFAP
-Using known specific (general S-100
monoclonal antibodies stain)/Desmin
(Muscle derived)
Epithelial (Carcinoma) + - - -
Sarcoma - + - -
Lymphoma - - + -
Melanoma (not always - - - +
pigmented)
Astrocytic tumor - - - - +
(GBM)

Immunohistochemistry is useful in categorization of undifferentiated malignant tumors: 1) Poorly


differentiated (anaplastic) carcinomas, 2) Lymphomas, 3) Melanomas, 4) Sarcomas

-Helps determine the site of origin of the metastatic tumor:

A. Prostatic Specific Antigen (PSA) in bone metastasis → Ej. 80 year old male pt., presents with back
pain, no prostate related symptoms and X-ray lesions in L3/L4→ after biopsy I detect Adenocarcinoma
in microcopy → Need to think of all the possible primary tumors that can lead to bone metastasis. Based
on the symptoms I should suspect prostatic cancer, therefore I should look for the PSA. I will report it as
Prostatic Adenocarcinoma with metastasis to bone
Keyla Galloza Acevedo

B. Thyroglobulin in carcinoma thyroid → Pt. normally presents with upper back pain and no related
thyroid symptoms. S&S: A lump in the neck, sometimes growing quickly, Swelling in the neck, Pain in the
front of the neck, sometimes going up to the ears, Hoarseness or other voice changes that do not go
away, Trouble swallowing, Trouble breathing, A constant cough that is not due to a cold.

C. Estrogen/ Progesterone R in Breast cancer→ Has prognostic and therapeutic value. Termed luminal
cancers. ER will regulate PR

D. HER-2/neu (ERBB2) in Breast cancer→ Common with patients with Li-Fraumeni. Difficult to treat.

E. ER negative, HER-2 neg (Triple negative) in Breast cancer→ More common in African American
Women, bad prognosis

Breast Histology: Terminal Duct Lobule Unit→ common place of origin of the tumor because it’s the
part that is most likely to respond to the hormones

-Dual lined structure: Luminal cells + myoepithelial cells

F. Ki-67 → Present at low levels in quiescent cells but is increased in proliferating cells. Its reactivity is a
specific nuclear marker for tumor cell proliferation

How do I report these findings?

-Cytokeratin (+) → Poorly differentiated Epithelial Malignancy or Carcinoma

-CD20 (+) → High grade Lymphoma


Keyla Galloza Acevedo

a. B-Hcg: Hydatidiform mole (Pre-malignant, Grape like structure, Gestational Trophoblastic


tumor, No fetus in ultrasound, To treat I need to evacuate the uterus contents and monitor the
hormone levels) , Choriocarcinoma (Very aggressive tumor but responds well to treatment),
Testicular tumor (Seminoma, Common in Young adults and older)
-B-Hcg is used in commercial pregnancy test kits so woman will think she is pregnant
b. Calcitonin: Medullary thyroid carcinoma, Converted to Amyloid
c. Catecholamines: Pheochromocytoma, VMA in urine
d. AFP: Hepatocellular carcinoma, Testicular cancer (Yolk sac, usually seen in pediatric group), Bad
prognosis
e. CEA: Colon cancer, if it’s detected at time of diagnosis, it will be important during follow up.
Sudden increase in CEA may mean metastases or recurrence
f. NSE: Small cell carcinoma, Neuroblastoma (Neuroendocrine properties)
g. CA-125: Ovarian cancer, usually seen in epithelial tumor, Can be silent so might not have
symptoms. Women may present with vague abdominal pain, small abdominal mass in
ultrasound, should look for CA-125 in blood sample

Microscopic changes: Benign Tumor Well differentiated Poorly


malignant tumor differentiated
Malignant tumor
Pleomorphism (variation in - + +++
size and shape of cells and
nuclei)
Hyperchromatism (Dense + + +
staining nuclei)
Keyla Galloza Acevedo

-Does not help


differentiate malignant
from benign tumor
Nuclear cytoplasmic ratio Normal Abnormal Abnormal (+++)
(N/C)
-Normal-1:4 or 1:6
Malignant-1:1
Mitosis Normal Abnormal Abnormal (+++)
-Malignant: Tripolar,
Quadripolar, Stellate
-Benign: Bipolar
Loss of polarity: N/A + +
-Seen in epithelial lined
structures only (Cervix, stomach)
-Epithelial cells dividing in all
directions
-Some changes are notes as
dysplasia
Differentiation + + -
Invasion - + +

Other microscopic changes:


Tumor Giant cells Tumor cells having 2 or more nuclei
-Common in Sarcomas
-NOT macrophages (Not the same as Giant
Langhans cells seen in TB)
Tumor Necrosis Seen in rapidly growing tumors
-Ischemic areas in central areas
-Ischemia can also be seen following
chemo/radiation (this is a good sign)
-This is the reason that we usually take Biopsies
from the growing edge of the tumor and not from
the center (ischemic necrosis and no viable cells)
Dysplasia: Disordered growth of epithelium
-Usually happens in metaplastic epithelium (Ej.
Barrets Esophagus, Resp. tract in smokers,
Parakeratosis (Psoriasis), Cervical Intraepithelial
neoplasia (CIN))
-Not always progresses to malignancy
-Mild to moderate dysplasias are reversible

a. Mild→ lower 1/3


b. Moderate→ Middle 2/3
c. Severe/ Carcinoma in situ→ Full thickness
(Pre-invasive lesion)
-As long as basement membrane is intact, it is
considered a dysplasia and not a cancer
Keyla Galloza Acevedo

-Common site for dysplasia→ Squamo-


columnar junction/ Transformation zone of
the cervix → CIN

Follicular Thyroid Adenoma Follicular Thyroid


Adenocarcinoma
Capsular breach - +
Vascular invation - +

Histology:

Neoplasias Part 2:
Benign Tumors: Exception: Hemangioma
-Do not infiltrate, invade or
metastasize -Benign but does not have a
-Remain localized and fibrous capsule
developed a fibrous tissue -Characterized by increased
capsule number of normal or abnormal
BV filled with blood
-This makes it difficult to
determine the excision margins
during surgery, therefore lead
to recurrence
-Use India Ink to stain the
margins of the specimen

Follicular Adenoma of the


thyroid:
Keyla Galloza Acevedo

Malignant Tumors:
-Infiltrate and destroy Invasive Ductal Carcinoma of
surrounding tissue the Breast (IDC)
-Poorly demarcated
-Slowly expanding malignant
tumors might develop a
pseudocapsule but would
show invasion on microscopy Renal Cell Carcinoma:
-Pseudocapsule + pushing
margin but on microscopy
shows invasion
-Appears to be benign but it is
not

Carcinoma of head of the


pancreas
-Infiltration of ampulla of Vater,
duodenal wall and common bile duct
-No demarcation
-Resection is very difficult
-Symptoms: Pruritis (Itching),
Excoriations (scratch marks), Light
colored stools, Dark urine,
Deep/Obstructive Jaundice

Metastasis: Exception: Gliomas and Basal


-Spread of tumors that are Cell carcinomas
physically discontinuous from -They do not metastasize
primary tumor -Gliomas: No lympahtics
-Lymphatics are a common -Basal cell carcinoma: Just
route for metastasis causes an ulcer
-Usually seen as multiple
nodules
Glioma

(BCC)
Keyla Galloza Acevedo

Lung metastasis:
-Canon ball appearance
-Ej. Deposits from a primary
breast carcinoma

Liver metastasis:
-Multiple nodules

Pathways of 1. Seeding of body Peritoneal Cavity


spread: cavities and Ej. Ovaries,
-Routes by surfaces Appendix
which tumor -Malignant cells -Mucin secreting
cells exfoliate from surfaces adenocarcinoma →
and implant/invade pseudomyxoma
disseminate tissue in body cavities
peritonei (Jelly Belly)
(Pleura, Pericardium,
Peritoneum) -Mucin secreting tumors
usually have a better
prognosis
Subarachnoid
space:
Ej. Acute
Lymphoblastic
Leukemia (ALL),

Glioblastoma
multiforme (GM)

(ALL)
2. Lymphatic Breast Carcinoma:
Spread: -Tumor cells on
Keyla Galloza Acevedo

-Common pathway outer quadrant of


for initial spread of breast can move to
carcinomas and the Delto-pectoral
sarcomas LN and eventually
-Follows the reach the Axillary
natural route of LN
lymphatic drainage -Treatment usually
-Sentinel LN: First requires a
LN that receives mastectomy. In
lymph from the case of a radical
primary tumor mastectomy, you -Possible Primary Tumors:
-This LN helps in might have to a. Stomach, Pancreas, Colon
the surgical remove the axillary
management of LN and this may -Patient with Supraclavicular edema
breast carcinoma cause swelling of (Supraclavicular LN/ Virchow LN)→
(If its neg. for the patients arm Suspect malignancy (Gastric, Prostatic,
tumor cells, I don’t (Lymphedema) Testicular)
need to clear the -Use Technetium-99
axillary LN) or blue dye to see
the malignant LN

3.Hematogenous Common sites:


Spread: 1.Liver: GIT,
-Seen in carcinoma of
Carcinomas and pancreas, breast
Sarcomas 2. Lungs: bone
-Veins are commonly cancers, gastric
involved (thin carcinoma, breast
walled) 3.Vertebral: thyroid Tumor emboli inside a BV
or prostate
carcinoma
(Paravertebral
venous/ Batson’s
plexus has no
valves)
4. Brain: Breast,
Lung
5. Renal vein: Renal
CC
4.Unusual Krukenberg tumor:
pathways of Gastric carcinoma
spread: (signet ring)
-Linitus plastic or
Leather bottle
stomach
-Usually Gastric
carcinoma is not
Keyla Galloza Acevedo

common in young
women
-Tumor shows
bilateral deposits in
surface of ovaries
-Only seen in
females

Perineural invasion
(Not really a route
of spread):
-Malignancies:
Pancreas, Prostate,
Salivary glands
-Pancoast tumor →
Horner syndrome
(Interruption of the
sympathetic nerve
supply to the eye)
-Miosis (constricted
pupil), partial
pstosis and
anhidrosis

BPH and Prostatic cancer: Prostatic cancer metastasis:


-BPH is usually seen in the Peri-urethral/ Inner -Tumor cells from the prostatic cancer detach and
zone. It causes compression of the prostatic invade the prostatic plexus of veins until it
urethra; therefore pt. will have retention of urine. eventually reaches the Vertebral Bastons’ plexus
Due to the straining to get the urine out, the → Bone metastasis
bladder will eventually hypertrophy.
-During digital rectal examination (men > 40 yrs), -Pt. may come with back pain but no prostatic
in case of BPH, the prostate will feel smooth and symptoms, symptoms can be felt when it has
enlarged already metastasized to the bone
-In case malignancy, the prostate feels rough and
nodular. -Elderly pt. with a back ache, investigate for
-Prostate cancers are usually seen in the prostatic malignancy
Peripheral/Outer zone and it does not cause -First I do a blood sample and test for PSA and
compression of the prostatic urethra. then a do the DRE. I don’t do the DRE first because
I would be stimulating the prostate and might get
a false (+)
Keyla Galloza Acevedo

Acquired Pre-disposing a. Asbestosis: Mesothelioma


conditions: (pleura)-slow growing
1. Chronic Inflammation malignancy
and Cancer
2. Precursor lesions and b.Inflammatory Bowel
cancer Disease/Chrons disease/
Ulcerative colitis: Colorectal
Chronic Inflammation: Carcinoma (usually seen in
(a)
young age group)
-Increase # of cells at the site of
chronic inflammation, this c. Barrets Esophagus:
increases the pool of tissue stem Esophageal adenocarcinoma
cells (susceptible to (Distal 1/3 esophagus) following
transformation)→ ROS Intestinal Metaplasia
-You will also see inflammatory
mediators that may promote cell d.Chronic Hepatitis (B/C): (b)
survival Hepatocellular carcinoma
-In CI both repair and metaplasia
may contribute to cancer e.Chronic osteomyelitis:
Cutaneous squamous cell
carcinoma
-Bone infection causes increase
of inflammatory cells and
edema, this causes pressure
within bone cavity to increase,
will compress BV, cause ischemia
and eventually the bone will (c)
break and form a sinus (blind
opening), now the surface of the
skin is exposed and will develop
a chronic irritation that can
eventually progress to skin
cancer

f.Chronic cervicitis (HPV (d and r)


Infection): Cervical carcinoma

g.Bronchial mucosa: Squamous


metaplasia carcinoma

h. Dysplastic nevus/mole:
Malignant melanoma

i.Shistosomiasis bladder:
Squamous cell carcinoma of
Keyla Galloza Acevedo

bladder

j. Chronic atrophic gastritis (H.


pylori): Gastric adenocarcinoma

k. Endometrial hyperplasia (HRT


in Post-Menopausal women):
Adenocarcinoma
(e)
l. Leukoplakia oral cavity:
Squamous cell carcinoma
-Risk factor: Chewing tobacco
-White plaque lesions in oral
cavity

m. Solar keratosis: Cutaneous


Sq. cell carcinoma
-Parakeratosis: (f)
Hyperchromatism, Loss of
polarity, Pleomorphism
-Abnormal keratin (has nuclei).
Normal keratin has no nuclei and
has a basket weave appearance.

n. Myelodysplastic syndrome:
AML

o. Villous adenoma rectum: (g)


Adenocarcinoma of colon
-Broad base villous, Shrub like
-More common in the Rectal
Sigmoid colon

p. Complete hydatidiform mole:


Choriocarcinoma
-HM looks like a bunch of grapes
-Common in Southeast Asia
(Cambodia, Vietnam) (h)
-High B-Hcg, no fetus in
ultrasound
-Gestational tumor, proliferation
of the Psytotrophoblast and
Syncyiotrophoblast
-Obstetric history: Height of
uterus does not correspond to
last menstrual cycle (LMC) (i)
-Histology will show Villous Shistosomas in bladder tissue
hyperplasia with
Keyla Galloza Acevedo

Synctiotrophoblast

q. Scar tissue in lung:


Adenocarcinoma of lung

r. Regenerative nodules in
cirrhosis: Hepatocellular (j)
carcinoma

(k)

(l)

(m)

(n)
Keyla Galloza Acevedo

(o)

(p)

(q)

IM=Intestinal metaplasia IEN= Intestinal Epithelial Neoplasia GC= Gastric carcinoma


Keyla Galloza Acevedo

Neoplasias Part 3:
1.Hereditary Breast Cancer BRCA 1/ BRCA 2. Carriers of these
mutations are also at risk of Prostate
and Pancreatic cancers
BRCA 1 (alone): Increased risk of Ovarian Cancer.
Familiar Cancers Poorly differentiated Breast Cancer.
Ej. Colon, Breast, Ovary, Brain, TNBC (Triple (-) Breast cancer: ER(-),
Melanoma PR(-), HER-2(-))→ This is a bad
prognosis
-Early age onset, tumors BRCA 2 (alone): (+) ER Breast Cancer→ tumors with
arising in 2 or more close estrogen receptors have a better
relatives, multiple/bilateral prognosis and can be treated by anti-
tumors estrogen drugs (Ej. Tamoxifen)
Glioblastoma multiforme: Highly
8 Steps in Cancer 1.Mutation in Growth Factors aggressive Brain malignancy
(Autocrine Loop) -Mutated protooncogene = SIS
-Synthesizes its own GF that -Secretes PDGF + PDGFR
acts on itself → Gastric Carcinoma: FGF3 + FGFR3
autonomously replicates
ERBB1: Lung adenocarcinoma (MC
cancer of lung base on histology).
-Normally is a EGFR
-Encodes for EGF
-Point mutation
2. Mutation in Growth Factor ERBB2 (HER-2/Neu): Breast
Receptors (Point mutation, Carcinoma
Gene Rearrangement, Over -Amplification mutation
Expression) c-KIT: Gastro Intestinal Stromal Tumor
-Constitutive GF independent (GIST)→ Tumor in the wall of the
Tyrosine Kinase activity bowel that projects towards the
serosal surface
A. Self-sufficiency in growth -Mesenchymal tumor of the GIT
signals (mutations in Proto- -Arise from Interstitial cells of Cajal→
oncogenes) GIT pacemaker, connected to the
myenteric plexus so important for the
propulsion of food (found btw.
Mucosa and Serosa)
-Gain of Function mutation (Point
mutation)
RET: Papillary Carcinoma of the
Thyroid
-Psammoma bodies

EML4-ALK: Lung adenocarcinoma


(non-small cell lung cancer)
-ALK=Tyrosine Kinase
Keyla Galloza Acevedo

-Gene rearrangement mutation


RAS mutation: MAPK, PI3K/AKT
-90% Pancreatic adenocarcinoma,
Cholangiocarcinoma
-50% Colon, Endometrial and Thyroid
cancer
-MAPK= Increase MYC + D cycline →
Cell cycle progression
-PI3K/AKT + mTOR → Pro-growth
metabolism + Increased protein
synthesis
3. Mutation in Signal BRAF mutation: Hairy cell leukemia,
Transduction (Tumor Melanoma
independent of GF continues -Constitutive activation of MEK
dividing = Constitutive pathway independent of extracellular
Activation) factors → Excessive cell proliferation
-Receptor Tyrosine Kinase and survival
Signaling Pathway PI3K mutations: Breast Carcinoma,
Endometrial Carcinoma
-Activates AKT
-Frequently mutated with PTEN
(Tumor Suppressor Gene)
-Mutated PTEN can lead to both
Endometrial Hyperplasia and
Carcinoma
BCR-ABL: Chronic Myeloid Leukemia
(CML)
-Translocation Chr. (9;22)→
-Non Receptor Tyrosine Philadelphia chromosome
Kinase Jak2 mutation (JAK-STAT):
Polycythemia vera, Essential
thrombocytosis (body makes too
much platelets), Primary
Myelofibrosis (Increase fibrous tissue
in Bone marrow, low normal blood
cells)
c-MYC: Burkitt Lymphoma (B-cells) ,
Breast Carcinoma
-EBV → Malignant BC
-Translocation Chr. (8;14)→
repositioning of MYC to Ig heavy chain
4. Mutations in Nuclear promoter in Chr. 14
Transcriptors (Transcription N-MYC: Neuroblastoma
Factors) -Commonly seen in infants
Can originate from: Adrenal glands,
posterior mediastinum, neck, pelvis,
sympathetic ganglion
Keyla Galloza Acevedo

-N-MYC amplification is a poor


prognosis indicator
-Tumor is made up of immature
ganglionic cells. As the child grows,
the immature ganglion starts showing
maturity (better prognosis) and can
eventually decrease in size or
completely disappear
5. Cyclins/CDK’s Cyclin D1: Mantle Cell lymphoma
-CDK’s→ gain of function -Usually detected in stage 4 (very bad
mutation prognosis)
-CDK Inhibitors→ loss of -t(11;14)
function mutation -Translocation/gain of function
mutations

CDKN2A (p16): Familial Melanoma


- Germline Loss of function

CDKN2A (p16): Pancreatic


adenocarcinoma, Glioblastoma,
Esophageal carcinoma (PEG)
-Somatic Loss of function
Rb (13q14):
-Normally regulates G1-S phase
transition in the cell cycle
-Germline mutation: Familiar
Retinoblastoma
-Higher chance to develop
Osteosarcoma
-Somatic mutation: Glioblastoma,
Small cell carcinoma of the lung,
Breast Cancer
P53:
Mutation in Tumor -Guardian of the genome, regulates
Suppressor Genes (Rb, p53, cell cycle progression, DNA repair,
APC) cellular senescence, Apoptosis
-Need 2 mutated copies in -Most frequently mutated gene in
order to have a loss of human cancers
function mutation -Loss of function mutation at
Chr.17p13.1
-Somatic mutations: most common
-Germline mutation: Li-Fraumeni
syndrome→ Greater chance of
developing a malignant tumor by the
age of 50, sarcomas, breast cancer,
B. Insensitivity to Growth leukemia, brain tumors, carcinomas of
Inhibitory Signals the adrenal gland
Keyla Galloza Acevedo

-Patient presents with broad range of


tumors along his life span, suspect Li-
fraumeni

Don’t confuse with MEN2: Also has a


broad range of malignancies but these
are of endocrine origin

APC (5q21): Familial Adenomatous


Polyposis (Adenomatous Polyposis
Coli)
-Component of the WNT signaling
pathway, normally keeps B-Catenin
quite to avoid excess cell proliferation
-Germline Loss of function
-Formation of thousands of
adenomatous polyps in the colon
during teens or early twenties
-Diagnosis requires more than 100
polyps
-Malignant transformation can lead to
colon cancer
This mutation can include additional
features:
A. Gardner syndrome: FAP+
Osteomas (mandible skull)+
Fibromatosis + Epidermal
inclusion cysts
B. Turcot syndrome: FAP+ CNS
tumors (Medulloblastoma,
Glioblastoma)
Colon Adenocarcinoma:
-MC malignancy in the US
-Risk factor: Low fiber diet, Intestinal
microbiota
-Common mutations: APC/B-catenin
(Chr. Instability is a hallmark), KRAS,
SMAD4 (TGF-B), TP53, Microsatellite
instability
-CF: Melena, Stool (+) for occult blood
(Guaiac test), apple core radiology
-The cancer could eventually reach
the bladder because bowel contents
can start to leak, a fistula can appear
btw. Bladder and bowel that could
eventually lead to presence of fecal
matter in urine
-Ulceroproliferative (growing edge)
Keyla Galloza Acevedo

E-cadherins: Esophageal, colonic,


breast, ovarian and prostate cancer
-Germline loss of function: CDH1→
Familial gastric carcinoma
-Normally anchor cells together
-B-catenin binds to tails of E-cadherin,
when it’s not functioning, B-catenin is
set free and you have increase in cell
proliferation
-Loss of cell-cell contact
-Loss of E-cadherin is needed for
invasion and metastasis
-Early loss of E-cadherin = More
aggressive cancer
-Could see cells in ‘Indian file’=Lobular
carcinoma of the breast
TGF-B: Colonic adenocarcinoma,
Gastric adenocarcinoma, Endometrial
adenocarcinoma
-Loss of function mutations
-Is very versatile, can: Induce cell
proliferation, Fibrogenic GF, Inhibit
epithelial proliferation (turn on anti-
proliferative genes)
-Regulates SMAD-4
SMAD-4: Pancreatic adenocarcinoma
-Sx: Obstructive Jaundice, Itching
(Excoriations), Palpable gallbladder
(Courvoisier’s Law)
-Normally regulated by TGF-B
-Inactivation leads to increase cell
proliferation by increase expression of
CDKN2A
KRAS (MC),
CDKN2A,SMAD4,TP53,BRCA2(Familial
Pattern): Carcinoma of pancreas
(Intraductal)
-4th leading cause of death in US
-RF: Cigarette smoking, high fat diet,
DM, Chronic pancreatitis
-Tend to be Desmoplastic tumor
-Can affect Ampula of Vater
PTEN: Cowden Syndrome→ Benign
follicular appendage skin tumors, and
breast carcinoma
-Tricoepithelioma
-PTEN is inactivated in both
endometrial hyperplasia and
Keyla Galloza Acevedo

carcinoma
-Normally acts as a tumor suppressor
by applying brakes of pro-
survival/pro-growth PI3K/AKT
NF-1
Cont. of Tumor Suppressors -Neurofibromatosis type 1: benign
neurofibromatosis, café-au-lait, Lisch
nodules (hamartomas in iris),
pheocromocytoma
- Neurofibromatosis type 2:
>Germline mutation→ Benign
bilateral Schwannoma of Acoustic
nerve (Nodular mass around the
Acoustic nerve, deep seeded tumor
that may cause death due to
compression of brain structures)
von Hippel-Lindau (VHL): Hereditary
renal cancers (~50yrs which is
uncommon because usually appear
later in life), Pheochromocytoma,
Hemangioblastoma of CNS, Retinal
angioma, Renal cyst
-Germline mutations in Chr. 33
-Normally VHL is an ubiquitin ligase
that regulates the Hypoxic Inducing
Factor (HIF).
-In the presence of O2, HIF binds to
VHL= no effect
-In a hypoxic state the HIF is
stimulated and will increase vascular
growth (VEGF, PDGF)
WT1: Wilms Tumor (Pediatric tumor)
-Chr. 11p13
-Involved in renal and gonadal
differentiation
-Mother feels a mass on child’s
stomach
Patched (PTCH): Gorlin syndrome
(Nevoid Basal Cell Carcinoma)
C. Altered Metabolism Warburg Effect Even in the presence of O2, cancers
cells increase their glucose uptake and
convert it to lactate (fermentation) =
Aerobic glycolysis
-Provides rapidly dividing tumor cells
with metabolic intermediates that are
needed for the synthesis of
cellular components
Keyla Galloza Acevedo

D. Evasion of Apoptosis Bcl-2 (anti-apoptotic factor): Follicular


lymphoma
-t(14:18)→ over-expression of Bcl-2→
anti-apoptotic properties
E. Limitless Replicative Telomerase Tumors up-regulate Telomerase,
Potential which prevents telomere from
shortening, therefore no apoptosis is
triggered

F. Sustained Pro-angiogenic factors Tumors cells make new blood vessels,


Angiogenesis from existing BV, to use as a source of
O2, nutrients and a route to remove
waste products
-Endothelial cells: IGF, PDGF
-Angiogenesis promoters: VEGF, bFGF
-I can block these BV to cause tumor
death→ Embolization of the BV to
choke the tumor
G. Ability to invade and Local invasion and Distant Metastatic cascade:
metastasize spread (Starts with a 1. Invasion of ECM
Metastatic Clone) -Metastatic clone→ Down-regulation
of E-cadherins, Proteolytic enzymes +
MMP’s, Integrins + laminins +
collagen, ECM molecules (migration of
tumor cells)
-MMP are important in tissue
remodeling
2. Vascular dissemination +
Homing of tumor cells
-Within BV tumor cells are clumped
together = tumor emboli
-Tumor surround themselves by
platelets to avoid being detected by
the NK cells
-CD44 on T cells is used by tumor cells
to reach certain tissues ej. Liver, Lung
3. Colonization

-Major cases of cancer-related


morbidity and mortality

Anatomic location for metastasis:


a. Prostatic adenocarcinoma→
Bone
b. Bronchogenic carcinoma→
Adrenals, Brain
c. Neuroblastoma→ Liver, Bones
Keyla Galloza Acevedo

H. Ability to evade the Tumor immunity Normally, antigens found in tumor can
host immune elicit an immune response and cell
response mediated immunity is the main anti-
tumor mechanism (Cytotoxic TC, NK
cells, Macrophages)
-But tumors cells can still evade the
immune response by selective
outgrowth of antigen-negative
variants (keep switching antigens)

FAP HNPCC
Genetic abnormality: APC, Chr. 5 MLH1, MSH2
Age: Polyps in teen years, carcinoma Cancer by age 45
of colon in the next 10 yrs
Morphology: >100 adenomatous polyps No polyp, limited mostly to
cecum and proximal colon
Associated disease: Gardner and Turcot ---

Neoplasias Part 4:
Genomic Instability: HNPCC (Lynch syndrome):
Familial carcinoma of the
-DNA repair defects colon (cecum and proximal
colon) evident by the age of 45
a. Mismatch repair -No pre-existing polyp
(HNPCC) -Affected person has only one
intact copy of the gene
b. NER (Xeroderma -Don’t confuse with FAP
pigmentosa) Mismatch Repair -Germline mutation: MLH1 +
MSH2
c. Recombination repair -Mutations happen in
(Ataxia telangiectasia) microsatellites (unstable)
-Sporadic cancers: Epigenetic
silencing of MLH1
-Colorectal carcinoma genes:
a. Oncogenic mediators: MSI,
MLH1, CIN, KRAS, BRAF, PTEN,
PIK3CA, EGFR, COX-2
b. Tumor suppressors: APC, B-
catenin, TP53, BAX, SMAD4,
TGFBR2, TGF-B, 15-PGDH

Xeroderma pigmentosum:
Keyla Galloza Acevedo

Squamous cell carcinoma


(Keratin pearls), Basal cell
carcinoma, Malignant
Nucleotide Excision Repair (NER) melanoma
-AR
-Wavelengths of UV-B rays are
more dangerous, as it
damages the DNA and creates
Pyrimidine dimers
-Water inside our cells
undergo hydrolysis and the
body generates ROS (Lipid
peroxidation, DNA mutations,
Cross-linking of proteins)

Covalent DNA cross links and


double-stranded DNA breaks
Homologous Recombination Repair Factors are normally repaired by
homologous recombination
a. Bloom syndrome -Failure to repair increases the
b. Ataxia Telangiectasia risk of cancer
c. Familial Breast Cancer a. BRCA1→Familial Breast and
Ovarian cancer
b. BRCA2→ Familial Breast and
Ovarian cancer
c. TP53→ Li-Fraumeni
(Sarcoma, Leukemia, Brain
tumors)-ER and HER (+)
d. PTEN→ Thyroid,
Endometrium
Carcinogenic Agents and 1. Chemical carcinogenesis 1. Direct (Carcinogens):
their Cellular Interactions Do not require no
a. Direct→ Cancer metabolic conversion
1. Chemical b. Indirect→ P450→ Cancer to be carcinogenic
carcinogenesis Ej. Nitrogen mustard
(Direct and -Weak carcinogens
Indirect) Drugs:
2. Radiant energy Cyclophosphamide,
3. Oncogenic viruses chlorambucil,
and microbes busulfan, melphalan
(Need to follow up
Initiation and Promotion: patients when treating
them with these
a. Initiation: drugs)
exposure of cells -This drugs could lead
to a sufficient to Myelodisplastic
dose of a syndrome (MDS)
carcinogenic -Ej. Used
Keyla Galloza Acevedo

agent. Initiated cyclophosphamide to


cell is altered, treat Hodgkins
making it Lymphoma→ Cured
potentially HL but drugs caused
capable of giving changes in Bone
rise to a tumor Marrow which led to
-Initiation alone, dysplastic cells MDS
however, is not (pre-cancer) which can
sufficient for eventually lead to AML
tumor formation (Poor-prognosis)

b. Promoters: can 2. Indirect (Pro-


induce tumors to arise carcinogens):
from initiated cells, but -Require metabolic
they are non-tumorigenic conversion to produce
by themselves the carcinogen
-Most are metabolized
by P-450
-Genes that encode
these enzymes are
polymorphic
Ej. CYP1A1 (Enzyme product of
P-450)
-This enzyme metabolizes
polycyclic aromatic carbons to
its active forms (carcinogen)-
Benzopyrene (Found in cig.
smoke)
-Individuals with a highly
inducible CYP1A1: 10% of the
population (Caucasian)
-Associated with smoking
related Lung carcinoma

A. Polycyclic aromatic
hydrocarbons:
-Benzopyrene (most potent
carcinogen)
-Ingredients of tobacco smoke,
and in broiling of animal
fat/fish
-Smoking→ Lung cancer
-Ingestion of broiled meat→
Bladder cancer
Keyla Galloza Acevedo

C. Aromatic Amines:
-Beta naphthylamine (aniline
dye, rubber/Tire industry)
-This amine is inhaled, is
detoxified by the liver , then
the product is taken to the
bladder and the enzyme Beta
glucuronidase converts it back
to its carcinogenic form →
Transitional cell carcinoma
-Field effect: Many tumors
growing in one area (multiple
papillary lesions in bladder)
D. Aflatoxin B1:
-Mycotoxin from some strains
of Asperguillus flavus
-Common in moist areas. It
contaminates stored grains
-Can lead to Hepatocellular
carcinoma
-It’s a well differentiated
tumor (Trabecular pattern +
Bile synthesis by tumor cells)
Trabecular pattern
E. Asbestosis:
-Occupational related disease
(Inhalation of asbestos)
-Has 2 patterns:
a. Serpentine→ gets stuck in
the airway passages, does not
invade lower Resp. tract
b. Linear: Is aero- dynamic and
reaches the lower Respiratory
tract → Malignant
Mesothelioma (Nodular
Pleural Thickening)
-Can involve the diaphragm
and you can see a beaded
appearance in needle
aspiration of the pleural
effusion
-Ferruginous bodies:
Macrophages try to degrade
the fibers but can’t and
eventually become frustrated
macrophages and leave behind
a trail of Hemosiderin + fibers
(FB)
Keyla Galloza Acevedo

F. Vinyl chloride (PVC):


-Occupational related
malignancy to the plastic
industry
-Angiosarcoma of the Liver
(Vascular channels can be
seen)
-Other carcinogens that can
lead to Angiosarcoma of liver
are:
Vascular channels a. Arsenic: Very deadly, rodent
poison can get absorbed
through the hand and
eventually cause cancer
b. Thorotrast: Had been used
as a contrast agent in
radiology
G. Nitrites:
-Nitrates and nitrites are
sometimes used during meat
processing
-Bacteria can convert these
Nitrates and Nitrites into
Nitrosamines (carcinogenic)
-Carcinoma of the Stomach
H. Diethylstilbestrol (DES):
-Used earlier for threatened
abortion
1. Clear cell adenocarcinoma
of vagina and cervix→ seen in
daughters of women who took
DES during pregnancy
2. Breast carcinoma→ women
who took DES during
pregnancy
A. Ultraviolet rays: Squamous
cell carcinoma, Basal cell
carcinoma, Melanoma of the
2.Radiant Energy skin
-UVB (280-320nm): 10:00am-
3:00pm → Danger (Most
important for skin cancer)
Keyla Galloza Acevedo

B. Ionizing radiation:
-Risk of Leukemia in children
exposed to radiation
-Papillary carcinoma of the
thyroid
>Psammoma bodies seen in
Dystrophic calcifications

3. Microbial 1. Human T-Cell


Carcinogenesis Leukemia Virus Type 1
(Oncogenic viruses): -Retrovirus
1. Human T-Cell -IVDA and STD (Sexual, Blood,
Leukemia Breast feeding)
2. Papilloma virus -Long latency 40-60yrs
(HPV) -TC Leukemia/Lymphoma
3. Epstein Barr virus -Tropism for CD4 cells
(EBV) -Tax gene: promotes growth
4. Hepatitis B virus signaling and survival +
(HBV) genomic instability
5. Kaposi sarcoma >Activates Long Terminal
virus (KSHV) Repeats (LTR) and Cyclins
>Interferes with PI3K/AKT and
activates NF-kB (promotes cell
survival en proliferation)
-TC malignancies will always
show skin involvement (skin
lesions)
-Resistant to chemotherapy
2. Human Papilloma
Virus (HPV):
-Causes mot cervical cancers,
as well as in vagina, vulva,
penis, anus, rectum and
oropharynx/larynx
-HPV1→ verruca plantaris
(plantar warts)
-HPV 6/11→ Condyloma
acuminate (Benign anogenital
warts)
-HPV 16/18 → Cervical
dysplasia/cancer
-E6→ Up regulates MDM2 and
promotes P53 degradation. It
also up-regulates expression of
telomerase
-E7→ Hyperphosphorilates Rb,
which causes it to release E2F,
Keyla Galloza Acevedo

now you have cell cycle


progression from G1 to S and it
inactivates CDKI
-HPV infects immature Sq. cells
during repair of epithelial
breaks but needs mature Sq.
cells to replicate
-Koliocytic atypia→
Characterizes HPV infection
(Danger)
Koliocyte change >Best seen in Cytology
-Cervix Biopsy: Invasive Well-
differentiated Sq. cell
carcinoma

Cervix Biopsy
3. Epstein Barr Virus
(EBV):
-Usually transmitted via saliva
Assoc. Malignancies:
a. Burkitt lymphoma
(t(8;14), c-MYC)
b. BC lymphoma
(immunosuppressed)
(a) c. Hodgkin lymphoma
d. Nasopharyngeal
carcinoma
-Majority of the times it is a
latent infection
-Infects BC by binding CD21
and expresses Viral Proteins:
1. LMP-1 (Deadly):
a. Constitutively activates CD
40→ Receives signals from hTC
and then make BC proliferate
(b)
b. Activates Nf-kB and
JAK/STAT
c. Block apoptosis
2. EBNA-2:
a. Promotes BC activation and
replication (Lymphoma may
Keyla Galloza Acevedo

arise if these cells acquire


mutation: t(8,14))
b. Viral cytokine (vIL-10) →
stops macrophages from
activating the TC
3. EBNA-1: Binds EBV genome
to the host cell chromosome

(c)

(d)

4. Hepatitis B
-Hepatocellular carcinoma
-Common in Far East and
Africa
-Chronic viral injury: leads
to increase proliferation of
hepatocytes (GF,
cytokines, chemokine’s,
ROS)→ Hepatocytes
become mutagenic
-Activation of NF-kB (block
apoptosis)
-HBx→ activates
transcription factors and
inactivates p53
5. Helicobacter pylori
(Cag A = oncogenic
strain)
-Can lead to Chronic Gastritis
(High gastrin)→ Gastric
Atrophy→ Intestinal
Metaplasia (goblet cells) →
Adenocarcinoma or BC
lymphoma (Maltoma)
-Linked to: Gastric carcinoma
(atrophy + intestinal
Keyla Galloza Acevedo

metaplasia) and Lymphomas


(Chronic antigenic stimulation
+ inflammation)
-Can have 2 scenarios:
a. Antral/ pyloric gastritis +
Duodenal ulcer
b. Multifocal Chronic Gastritis
6. Kaposi sarcoma
(KSHV/HHV8):
-Vascular neoplasm associated
with AIDS (Aggressive tumor)
-Mostly STD
-Spindle cell tumor
-May involve skin, oral
mucosa, LN, Visceral organs
-Mostly involves lower
extremities, head and neck
region

Clinical aspects of 1. Cancer Cachexia


Neoplasia: -Anorexia and weight loss
-Equal loss of fat and lean muscle (Elevated
1. Cancer Cachexia BMR)
2. Paraneoplastic -Usually seen in stage 4 cancer→ Both host
syndrome and tumor cells are producing cytokines
3. Grading and (TNF, IL-1, INF gamma) that interferes with
Staging the metabolism
4. Lab diagnosis of
cancer
5. Tumor markers
2. Paraneoplastic a.Cushing syndrome:
syndromes (Resemble endocrine -ACTH
problems because the tumor cells -Pt. with a cigarette smoking
mimic endocrine hormones- history
neuroendocrine tumors)
b.SIADH:
a. Cushing syndrome: -ADH
b. SIADH: -Watch out for Hyponatremia
c. Hypercalcemia:
d. Carcinoid Syndrome (ej. Carcinoid c. Hypercalcemia:
tumor commonly located in submucosa -PTHrP
of intestine-appendix, rectum and -Not due to Metastatic Bone
terminal ileum) Cancer
e. Polycythemia
f. Myasthenia d.Carcinoid Syndrome:
g. Eaton-Lambert Syndrome -Serotonin, Bradykinin
Keyla Galloza Acevedo

h. Acanthosis nigricans (Blushing + Diarrhea)


i. Hyperthrophic osteoarthropathy and -MC in the GIT
clubbing fingers -Serotonin (Derived from
j. Venous thrombosis Tryptophan) increases when
consuming cheese and coffee
-Serotonin increase the levels
of Fibrogenic cytokines (Go to
the R side of the heart and
cause Fibrotic damage to the
Tricuspid valve)
-High 5’-HIAA in urine
-Metastasis to Liver (Carcinoid
disease)
-Tricuspid Stenosis

e.Polycythemia:
-Erythropoietin
-Well Differentiated RCC→
High Erythropoietin→ High
RBC

f. Myasthenia: Autoimmune
attack against the Ach R
-Weakness

g. E-L syndrome: Problems in


VG Ca+ channels → no release
of Ach
-Symptoms decrease during
exercise

h. Acanthosis nigricans:
-Appearance of dark patches
(darkening of skin folds)
-Higher than normal insulin
levels in the blood

i.HO + clubbing fingers:

j.Venous Thrombosis:
-Pancreatic adenocarcinoma
Malignant glands are
producing mucin (intra and
extra). The extracellular mucin
eventually reaches the
bloodstream and interacts
with clotting factors→
Increase risk of thrombosis
Keyla Galloza Acevedo

(Migrating
Thrombophlebitis in popliteal
area)
-Trousseaus phenomenon:
Hypocalcaemia, Carpopedal
spasms, twitching of facial
muscles

3.Grading (Pathologist) A. Grading: histological determination and B.Staging:


and Staging (Clinicians): refers to the degree of cellular differentiation -Use TNM (Tumor size, LN,
-Based on pleomorphism, cellularity, necrosis, Metastasis)
invasion, and # of mitosis -More important than Grade
-Tumors can be graded from I to IV
-Well differentiated tumors (Morphology and Ej. T1=<2cm, T2=2-4cm, T3=4-
Function) = Grade 1 6cm, T4= 6>
-Poorly differentiated malignant tumors= grade
3 or 4 Ej. N1 = LN involvement, N0=
No LN, M0= no Metas., M1 =
(+) Metas.

Ej. T3 N0 M0 → Pt has a 4-6cm


tumor, no LN involvement and
no signs of metastasis
Keyla Galloza Acevedo

Small Cell Carcinoma of the Lung (SCCL) -High risk in cigarette smokers
-Rapidly growing malignancies (likely to present
spread at time of diagnosis)
-No known precursor lesion
-Loss of tumor suppressors (p53, Rb)
-Most arise from major bronchi or periphery
(Exception because normally Central tumor are
Keyla Galloza Acevedo

associated with smoking while tumors in periphery


are normally associated with adenocarcinoma and
non-smokers)
-May synthesize ectopic ACTH and ADH
-Positive for chromogranin and CD56 (general
marker for neuroendocrine tumors)
-Seen in paraneoplastic syndromes (Cushing,
SIADH, L-E)
-Additional symptoms: Compression of the SVC
(SVC syndrome), Horner syndrome, Acanthosis
nigricans
Small cells with scant cytoplasm, high mitotic rate, -May metastasize to: Liver, Bone, Adrenal, Brain
dense core of neurosecretory granules

Paraneoplastic Syndrome: Hypercalcemia Bone Metastases


-Osteolysis induced by cancer: Primary bone
-Production of calcemic humoral substances by tumor (Ej. Multiple Myeloma) or metastatic to
extraosseous neoplasms (PTHrP) bone from any primary lesion
-Seen in Sq. cell carcinoma of lung and Breast -Not paraneoplastic
Carcinoma -Hypercalcemia due to primary or secondary
-No bone metastasis involvement of the skeleton by tumor (High
-To differentiate between Bone Metastasis and osteoclastic activity and hyperthrophy)
Paraneoplastic I need to do a Bone Scan -Frequently seen as complications of cancer
-In paraneoplastic syndrome you won’t see any a. Osteolytic bone disease: more aggressive and
bone lesions more common (Moth-eaten appearance on Bone
Scan)
b. Osteoblastic bone disease: slower course.
Production of abnormal bone tissue. Commonly
seen in Prostatic cancer

(a)
Keyla Galloza Acevedo

(b)

Pancoast’s Tumor: Horner Syndrome


-Superior sulcus tumor arise in the apex of the lung
and invade adjacent structures producing the
Pancoast tumors

Horner syndrome:
-Lesion in the central or peripheral sympathetic
nervous system
-Miosis (constricted pupil), mild ptosis, anhidrosis
Anemias
Iron deficiency anemia Anemia of chronic disease Sideroblastic anemia Hemolytic Anemia Beta thalesemia Alpha thalasemia
Classification Microcytic Hypochromic Microcytic hypochromic Microcytic hypochromic Microcytic Anemia Microcytic Hypochromic (rushed erythropoeis causes improper hemoglobinzation) Microcytic Anemia
Hb/ Hct Reduced Severely low
MCV, MCH, MCHC All reduced (<80) Reduced MCV Reduced MCV MCV reduced Deceased MCV
Smear Normal cellularity (Bone marrow) Nucleated RBCs
 -Elevated reticulocyte count

Anisocytosis (RBCs are unequal in size)
 -Irongutted mitochondria is seen as a ring of

 -Target cells (Decreased Hb= decreased volume, some hb in middle of cell)


 sideroblasts (15-100%) Iron gutted mitochondria 
 Increased reticulocytes (Increased cel turn over)- Polychromataphils—> -Fragemented RBCS


 -NULCEATED RBCS

Polikocytosis (Pencil cells)
 -Dimorphic Red cells (Normocytic + Microcytic)
 Supravital stains

 

Hypochromic (increased central pallor)


 - Pappenheimer bodies (iron residues in RBCs)

Thin rim of hemoglobininization 
 


Target cell
 -Nucleated RBCS

Wide RDW
Image Nothing unusual (May show some Microcytic, hypochomic) Supravital stain showing reticulocytes Looks a lot like IDA, use lab results to distinguish (electrophoresis)

Lab diagnosis/ -Bone marrow exhibits hyper plasticity. Low O2 carrying Infection/Cancer will increases production inflammatory cytokines (TNF, -Pathologic iron deposits in erythroblast mitochondria. Defect in heme -RBC lifespan ~ 120 days. Older RBCs are susceptible to phagiocytsosis in the -1 or both of the beta globin genes doesn't work (Inherited autosomal recessive)
 -Reduced synthesis of Alpha chains

synthesis within mitochondria. Iron is brought into mitchodondria to form heme. reticulendothelail system (spleen, liver, BM). RBCs when broken release Hb. 
 

Pathogenesis capacity activated HIF-a to stimulate a EPO—-> erythropoiesis increased
 IL-1, IL-6, IFN-y). IL-6
will act on the liver to promote Globing and heme are split from each other. Globin is sent to the protein pool. homzyogous- Beta thalesimia major (severe)
 


 If something goes wrong in heme development—>
synthesis of hepicidin. This will prevent release of iron into Porythyin ring of heme is cleaved to produce biliverdin which reduced to 
 Hydrops fetalis= 4 defecting a genes leads to alpha
-Microcytic maturation in BM (Buzz word for IDA)
 iron remains trapped in mitochondria—> increased iron bilirubin. Bilirubin is passed in plasma and forms complex with Albumin—> Heterozygous- Beta thalesmia minor (asymtomatic anemia)

blood. The inflammatorycytokines will also act on the transported to liver.
 
 chain tetramers (Bart bodies). Very high affinity for O2

 stores.

kidney to decrease synthesis of EPO and thus RBC 
 In β thalassemia, the synthesis of normal α globin chains from the unaffected α globin genes continues as but no delivery.
-Loss of stainable iron in macrophages, demonstrated by 

May be because of absence of protophryin or lack of ALA synthases or Lack -Shortened RBC life span causes BM hyperplasia (BMH). Anemia occurs if normal, resulting in the accumulation within the erythroid precursors of excess unmatched α globin.
Prussian blues staining. (0 or +1)
 production will fall.
 BMH cannot compensate for RBC destruction. Hemolytic disorders 


 of ferrchlesatase or any enzyme involved in heme synthesis 
 The free α globin chains are not able to form viable tetramers and instead precipitate in the red cell

 


 precursors in the bone marrow forming inclusion bodies. These α chain inclusions can be
-Low serum Iron
 Highly stainable iron in macrophages Erythroid hyperplasia
 Intravascular hemolysis- At first the Hb that is liberated from RBC lysis is demonstrated by both light and electron microscopy in the erythroid precursors in the bone marrow as well

-Increased TIBC
 demonstrated by Prussian blue. Bc of increased 
 captured by Hapatoglobin and taken to the liver. However haptoglobin as in the peripheral red cells following splenectomy. They are responsible for the extensive intramedullary
-Increased hemosiderin 
 has limited synthesis. 
 destruction of the erythroid precursors and hence the ineffective erythropoiesis that underlies all β

 iron stores
 
 
 thalassemias. 

-Low serum Ferritin 
 
 After a few days Hapatoglobin is deleted and Hb breaks down further into 


 
 EXPRESSION OF MITOCHONDRIAL FERRITIN
-
Meth hb, The ferriheme component is bound by hemopexin and taken to Anemia in β thalassemia thus results from a combination of ineffective erythropoiesis, peripheral hemolysis,
-% saturation of transferrin is below 33
 


(dead give away)
 the liver. 
 and an overall reduction in hemoglobin synthesis. The severity of disease in β thalassemia correlates well
-TIBC is decreased
 
 with the degree of imbalance between α and non-α globin chains and the size of the free α chain pool.

-Increased Free Erythrocyte Protoporphyrin . Cannot form

-Serum ferritin is increased (ARP)

-TIBC reduced
 After a while Hemopexin levels become depleted. Once both hemopexin and Thus, factors that reduce the degree of chain imbalance and the magnitude of α chain excess in the red cell
precursors will have an impact on the phenotype

Tetrapyrole ring bc there is no central Fe… Therefore precursors increase in -Serum ferritin increased
 haptoglobin levels are depleted. Hb escapes to Kidneys where it causes

 

circulation hemoglobininuria. Hb escapes into plasma causing hemoglobinemia
-Serum Iron is reduced -Serum iron increased
 (Plasma turns pinkish red). In the PCT the Hb is reabsorbed In cells and increased gastrointestinal iron absorption lead to iron overload in the body. Iron overload impairs the

 immune system, placing patients at greater risk of infection and illness.

broken down—-> leads to hemosinderiuria. 


 



Erythropoeitic porphyria-
- Erythropoietic Extravascular- RBCs are tagged with IgG and destroyed in the spleen. -Anoxia stimulates increased EPO—> increased Erythropoeis 

protoporphyria is a form of porphyria, which varies in severity and can be very Haptoglobin will be slightly lowered because some Hb escapes int circulation. 

Hemopexin will be normal
 

painful. It arises from a deficiency in the enzyme 
 -Elevated serum bilirubin (RBC lysis) 

ferrochelatase , leading to abnormally high levels of 
 

protoporphyrin in the red blood cells (erythrocytes), Bone marrow Hyperplasia (increased reticulocytes -Decreased/normal TIBC

plasma, skin, and liver in circulation)
 -Increased serum ferritin


 -Increased serum iron

LDH increased (High cell turn over)
 


 HbA2 and HBf are increased (Non-beta chain structures) HBa is
Extra medullary Hematopoesis absent


Clinical features - Fatigue, Light headiness, palpitation, pounding in ears, decreased work -Fatugue and pallor
 - Hyperbilirubinemia (depending on amount of RBCs broken down) and -Pregnancies will terminate very early (Fetal distress in 3rd
-Manifestation may not be apparent until complete switch from fetal
performance
 
 jaundice (not reliable)
 trimester).


 -Iron overload
 
 to adult Hb (6 months)
 

-Menstrual abnormalities 
 
 -Decreased levels of haptoglobin (especially intravascular hemolysis) 
 
 Fetal distress- Higher heart beat or absence 


 -Patient may or may not be B6 responsive 
 
 -Tissue anoxia
 


 -Hemopexin is low in intravscular hemolysis (Very important to distinguish 
 -Edema of face and abdominal distension. Thoracic cavity is
- Pica - a compulsive eating disorder in which people eat nonfood items. Lead posion- 
 between Intravscular and extravascular) 
 -Iron overload may lead to congestive hearty failure (RBC concentrated transfusion to resolve this)
 filled by the pericardial sac distended by effusion and
Dirt, clay, and flaking paint are the most common items eaten
 
 
 cardiomegaly; massive hepatomegaly. Caused by extra

—> Ice example coming on exam 
 Coarse stippling, Pb inhibits Pyramidine 5’ nucleitidase thus preventing the -Hemoglobinemia—> Plasma Hb increases causes plasma to turn pink/ medullary hematopoiesis 

-Marked pallor, Jaundice= Lemon yellow


 break down of RNA in reticulocytes. 
 red
 


 -Toxemia of pregnancy

 

-Pallor, glostitis
 Lead line (plumbism). 

- Disorders causing secondary hemochromatosis (e.g., thalassemia) are characterized by ineffective
Hemoglobinuria—> Renal threshold for Hb reabsorption is exceeded, pink erythropoiesis leading to increased duodenal uptake of iron


 
 color to urine on examination 
 

- Angular chelilitis, Koilonychia (spoon shape nails)
 Increased density in epiphysis 
 Splenomegaly (by 3 years)


 
 

-Impaired growth in children, hair loss is girls
 Cholelithiasis-> Supersaturation of bile with bilirubin -Growth retardation—> linear growth, mandible stays same size (Malo-occlusion of teeth)


 

(Pigmented gall stones)

Plummer-Vinson Syndrome—> Microcytic
- 

-Bilirubin gall stones bc of hemolysis (Splenectomy may help)


hypochromic anemia, Atrophic glosstis, -Skeletal poor growth (especially linear growth… they will be shorter),
Enlarged skull (All skull bones except mandible… Chipmunk)

-Crew cut Skull X ray (inner and outer plates are separated by increased erythropoiesis)

Esophageal webbing = Premalignant lesion 
 

- Minor- Will be asymptomatic but will become evident during pregnancy

Risk factors -Dietary lack of iron




 -Chronic microbial infection
 -X

linked (ALA synthetase-2)
 Arab, Mediteranean, Black, Indian

-Increase need for iron (Growth, pregnancy)




 
 -Myelodysplastic syndrome (MDS)- a group of cancers in which immature
blood cells in the bone marrow do not mature, so do not become healthy blood
-Chronic blood loss conditions ( ie. Ulceric carcinomas)


 -Chronic Immune disorders 
 cells.


-Hook worm



 -Chronic alcoholism 



Any condition that tilts the iron balance negative
-Neoplasms -Isoniazid, INH


-B6 deficiency—> cofactor for ALA synthase


-Lead poison 


Table 1

B12/Folic acid deficiency Anemia of renal failure Alcoholism

Classification Megaloblatic (Changes in BM) Non-Megaloblastic (No BM maturation) Non-Megaloblastic (No BM maturation)

Hb/ Hct Reduced

MCV, MCH, MCHC MCV Exceeds 120

Smear Macrocytic change
 -Macrocytes are round rather than the typical oavlocytes.
 -Macrocytes are round rather than the typical oavlocytes.

-Presence of ovalocytes 
 
 

-Absenceof hypersegmented neutrophils and -Absence of hypersegmented neutrophils

- Howell-Jelly Bodies—> remnants appear as 

inclusion bodies
 Pancytopenia.
 -Pancyopenia not seen

 

-Absence of reticulocytes (underproduction)
 -Pancyopenia not seen


 

Hyper-segmented neutrophils (+5 lobes) -Macrocytic red cells= Burr cells
(earliest morphological change)
Image Peripheral Blood Smear 









Lab diagnosis/ Ineffective erythropoiesis. Ineffective interaction -anemia may not manifest
 -Indirect effect—> alcohol can produce megaloblastic anemia due to
Pathogenesis between Folate and B12 is responsible for megaloblastic 
 nutritional causes

anemia. Lack of B12/Folate will delay DNA maturation -Damage to cells that make erythropoietintin 

(Thymine) therefore delay in replication. Hemoglobinzation -Direct effect—> alcohol can act on BM and precursor cells to
therefore suppression of RBC synthesis and produce “non-megelaomblastic” anemia (antibodies against
not affected because it is an RNA process that uses Uracil
instead of DNA (N/Y Asynchrony—-> leads to ineffective shortened RBC life span.
 acetaldehyde)
erythropoetitis)
 


 -qualitative defects in platelets= prolonged bleeding times

-Low CBC count (Lekopenia, Thrombocytopenia) bc of 

ineffective erythropoeies. Delay in maturation
 (Treated with Darbepoetin alfa and epoetin)

Vitamin B12 Pernicious anemia: Progressive destruction
and eventual loss of parietal cells, leads to loss of IF.
Blocking autoantibodies present in gastric juice can bind
B12 preventing IF from acting on it.


BM finding: Increased cellularity because of delayed
maturation and increased destruction. Megoblastic
maturation. Lack of chromatin clumping (does not
condense). Abnormally large Ganulolcytes and platelets.


Megaloblastic marrow


CYTOPLASM SHOWS REGULAR HEMOGLOBINZATION


Biochemical testing- Serum Bilirubin mild increase bc of
RBC lysing. Serum B12 decreased therefore HC increased
and methylmalonic acid increased (ONLY SEEN IN B12
Def). Antiboides to IF present in most cases.


Gastric biopsy- Chronic atrophic gastritis is commonly
seen. Lymphocytes will destroy parietal cells. May result in
total achloryida 


Folic acid Pernicious anemia: 


Woman who gave birth to babies with neural-tube
defects (hasnt closed after 28 days) has low
serum levels of Folate. AFP will be elevated in
neural tube defect for pregnant woman
(Maternal AFP)


Clinical features B12




-Lemon tint anemia, Mild icterus, atrophic glosstits


Neurological: Peripheral neuropathy (Stalking parathesia and
numbness)


Subacute combined spinal degeneration: DCML (loss of
vibration, Propioception). + Romberg . May seen lateral
column lesion 


-Cerebral manifestations- Megaloblastic madness


Folate

-Neural tube defects (doesnt close in 28 days)-
Spina Bifid Occulta, Meninhgocele,
Myelomenigocele

Anencephaly
Risk factors For B12

-Deficient in food

-Pernicious anemia

-Gastrectomy, Ileum resection, gastric bypass

-Acid blocking drugs

-Chronic pancreatitis

-Bacteria, Parasites

-Vegan diet 


For folate

Methotrexate, 5-Flourocil


Etiology B12: N. Europe (woman)

Megaloblastic marrow
Table 1

Aplastic anemia Hereditary Spherocytosis PNH Sickle cell anemia G6PD deficiency PK deficiency

Classification Corrected reticulocyte count <3% Corrected reticulocyte count >3% Corrected reticulocyte count >3% Corrected reticulocyte count >3% Corrected reticulocyte count >3% Corrected reticulocyte count >3%

Hb/ Hct Decrease Hb Decrease Hb Decrease Hb Decrease Hb Decrease Hb

MCV, MCH, MCHC Normal MCV Normal MCV Normal MCV Normal MCV Normal MCV

Smear Spherocytes—> RBCs absent central pallor 
 10-15% sickled—> young RBCs undergo sickling, most Reticulocytes increased


 are usually lysed… thats why % is so low.
 Nucleated RBCs

Reticulocyte increases
 
 


 Acanthocytes with surf spicules (spur cells)
Howell-Jolly bodies may be present

Image Doubt it

Lab diagnosis/ -Reduction in the number of hematopoietic stem/ -Spherical And less deformable
 -Intravascular hemolysis—> Decreased Haptoglobin and -Point mutation at 6th position of the B globin chain leads -Intravascular hemolysis 
 -Hemolyitic anemia due to Gyloclyitic enzyme def

Pathogenesis Progenitor cells
 
 Hemopexin, Hemoglobinemia. 
 to replacement of a glutamate residue with valine residue.
 
 


 -Vulnerable to splenic sequestration and distortion 
 
 
 -G6PD reduces NADP to NADPH. NADPH provide - PKD result in impaired glucose utilization

-Leads to inability to make mature rbcs
 
 -Somatic mutation of PIGA—> Damage to GPI—> CD55, HTZ= 40-45% of the Hb in Hbs form
 reduction for ROS via Gluthione reductase enzyme. 
 - Decreased Lactate production


 -Mutation in Spectrin or ankyrin—> Loss of RBC membrane and CD59, C8 binding proteins 
 
 
 

Extrinsic cause- Immune mediated suppression of increased permeability to sodium
 
 Homo= 90-95% Hb in Hbs form
 
 -Increase in Glycolytic intermediates before PK (2,3 DPG)

marrow progenitors. Stem cells antigenically altered by 
 Reduced Iron stores
 HbA-Absent
 -Oxidant exposure results in oxidation of Sulfhdyryl group 

exposure to toxin. Provokes cellular immune response -Decrease in membrane stability and membrane loss. Decrease 
 HbF= greater the 5%
 on Hb chain. Forms a membrane bound inclsuion—> -Dimished capacity to generate ATP (failure of NA pump)

which kill hemotpoetic progenitors
surface to volume ratio. Splenic trapping—> erythrostasis. Hyperplastic bone marrow
 
 Heinz body (bite cells)
 

Phagocytosis causes extravascular hemolysis
 
 2% sodium Metabisulfate will test for it
 
 -Extravascular hemolysis

Intrinsic cause- Stem cell abnormality. Chemotherapy 
 Sucrose Hemolysis test +

 Oxygen stress—> Anti-malarials, Infections (Viral hepatitis, 

treatment for one malignancy (Hodgkins) leads to Osmotic Fragility test: Spherocytes rupture easily HAM test + When deoxygenated, HbS undergoes polymerization. Pneumonia, Typhoid fever), Fava beans (favism) Increase S. Bilirubin
Malignancy (MDS+AML) Aggregate HbS molecules form needle like fibres within
red cells.

-Chronic hemolysis

-Microvascular obstruction

-Tissue damage


Polyemrized RBC damage membrane—> Ca influx leads
to K efflux with water. Cells become irreversibly sickled

Clinical features Jaundice, Splenomegaly, Gall stones
 Nocturnal Hemoglonuria- resp acidosis during night Vaso-oclusive crises- Sickle red cells higher than normal -Features of acute hemolysis Anemia


 time augments complement attachment to cell. Red urine levels of adhesion molecules and are therefore sticky
 

Aplastic crisis- Associated with acute parvovirus infection, in the morning.
 
 Bilirubin

Virus red cell progenitors. Worsening anemia= Reduced 
 -Mediators release from granulocytes during inflammation 

Reticulocytes
 
 up regulates expression of adhesion molecules on Aplastic crisis

 Chronic Hemolysis- 
 endothelial cells—-> stagnation of red cells in micro-
Hemolytic crisis- Increased splenic destruction of sphreocytes. 
 circulation 

Enlarged tender spleen. Worsening anemia, increased jaundiced Iron deficiency 
 

and darkening of urine.
 Bleeding (Thrombocyopinia)
 Hand and foot syndrome- Hand and feet appear swollen

 
 and painful. Micro-infarction of carpal and tarsal bones. 

Colethilailis- Pigment of gall stones (Hyperbilirubinmia) Risk of thrombosis—> platelets release aggregating 

facotors—> hepatic vein thrombosis leads to Budd Osteomyelitis is common—> Salmonella Paratyphi

Chairii syndrome 
 


 Acute chest syndrome- Acute chest pain following lung
May develop AML infection. Pulmonary blood becomes sluggish, associated
with increased mortality.


CNS- Seizures/Strokes due to hypoxia..May lead to
hemiparesis


Retinopathy- Blindness


Severe infections- Autosplenectomy


Sequestration Crises- Massive sequestration of sickled 

cells in spleen. Rapid splenic enlargement 


Aplastic crisis- Transient bone marrow failure—>
Parvovirus B19. Worsening anemia and decrease
Reticulocyte count 


Colelithiasis- Hyperbilirubinimia 


Chronic leg ulcers, Renal (Hematuria, Damage to vas recta
decrease osmolarity).


Fat embolism is possible

Risk factors Anticancer drugs, Benzene

Etiology
RBC Disorders Quiz
Q1) A 21 year old woman brings her 2 year old male baby with
complains of poor feeding. The child was born after a normal full-
term pregnancy to a vegan mother exclusively breastfed. P/E: pallor,
mild scleral icterus, body weight, length and head circumference were
below the expected parameters. Also noted were enlarged liver and
spleen. Labs: Hb 9.7 gm% (N 14 to 17 gm%), total white cell count
7,000 cells/mm3 (N 4,000 to 11,000 cells/mm3), MCV of 130 fL (N 80
to 100 fL). Which one of the following additional features may be
found in this child?
A) Elevated levels of serum methylmalonic acid
B) Micro infarction of ends of long bones
C) Liver biopsy would show intense golden brown particles
D) Presence of heterozygous α-thalassemia trait
E) CT brain shows diffuse areas of demyelination
Q2) A 3 year old Caucasian boy is brought by his parents to the clinic
because they think he looks "pale" and "weak". On examination the
boy is lethargic and has pale conjunctivae. Blood analysis denotes
megaloblastic anemia. Blood biochemistries reveal increased
homocysteine levels. There are no other abnormalities. The most
likely cause for this boy's condition is:
A) Vitamin B12 deficiency
B) 5 –aminoleulinic acid deficiency
C) Glucose 6 phosphate dehydrogenase deficiency
D) Myeloperoxidase deficiency
E) Glutathione deficiency
Q3) A 22 year old man is evaluated in the emergency department
because of the sudden onset of right hemiparesis of four hours`
duration. P/E: febrile, pallor, scleral icterus, slurred speech, no
alcohol on breath, left-sided hemiparesis, BP 110/70 mmHg, pulse is
100/min, respiration rate is 24/min., and temperature is 39.1 C. Labs:
Hb 8 gm%i (N 14 to 18 gm%), raised total serum bilirubin, peripheral
blood smear is shown. Which of the following is the most likely
diagnosis?
A) Hereditary spherocytosis
B) Iron deficiency anemia
C) Sickle cell anemia
D) Thalassemia minor
E) G6PD deficiency
Q4) A 60 year old man comes to his physician with complaints of
easy fatigability and palpitations for the last six months. P/E: pallor of
skin & mucous membranes. No evidence of cardiac or respiratory
failure. Labs: Hb 8.4 gm%, MCV 75 fL (N 80 to 100 fL), total white cell
count 9,000 cells/mm3 (N 4,000 to 11,000 cells/mm3), total platelet
count 380,000 cells/mm3 (N 150,000 to 400,000 cells/mm3). Serum
chemistry: reduced serum ferritin and normal serum bilirubin.
Peripheral smear shows small erythrocytes with marked variation in
size. Which one of the following is the most appropriate next step?
A) Prussian blue stain of bone marrow
B) Coomb’s test for anti- rbc antibodies
C) Hb electrophoresis
D) Test for occult blood in stool
E) Therapeutic trial with folic acid
Q5) A 38 year old female with a three-year history of rheumatoid
arthritis currently on immunosuppressive therapy and including
corticosteroids complains of extreme tiredness. She has a CBC that
shows Hb 10 gm% Hct 30%, MCV 88 fL (N 80 to 100 fL), WBC count
6500 cells/mm3, and platelet count 137,000 cells/mm3, reticulocyte
count is 2.1%, reduced serum iron, increased serum ferritin, and
increased total iron binding capacity (TIBC). Which of the following is
the likely diagnosis?
A) Sideroblastic anemia
B) Beta-thalassemia trait
C) Anemia of chronic disease
D) Vitamin B12 deficiency
E) Iron deficiency anemia

Q6) A 68 year old man on chemotherapy for carcinoma stomach


presents with a 5 day history of fever, productive cough, and chest
pain. These symptoms were noticed a week after completing his 3rd
cycle of chemotherapy. P/E: febrile, vitals within normal limits, rales in
both lung bases. Labs: Hb 12 gm% (N 14 to 17 gm%), total white cell
count 1,900 cells/mm3 (N 4,000 to 11,000 cells/mm3), absolute
neutrophil count 450 cells/mm3 ( N ≥ 1,500 cells/mm3). The lab picture
improved on day five of starting Filgrastim. Which one of the following
is likely to explain the blood picture in this case?
A) Bone marrow stem cell depression
B) Increased splenic destruction
C) Myelodysplastic syndrome
D) Presence of circulating immune complexes
E) Tuberculosis of bone marrow
Q7) A 7 year old boy with a known diagnosis of sickle cell disease
presents with fever, malaise and anorexia since oneweek. P/E:
appears ill, body temperature 38.6˚C, pallor, scleral icterus, chest
clear, mild hepatomegaly, and no joint or neurological abnormalities.
Labs: Hb 5 gm% (N 14 to 17 gm%), total white cell count 2,800
cells/mm3 (N 4,000 to 11,000 cells/mm3), reticulocyte count below
normal, and peripheral blood smear shows 35% sickled red cells.
Which one of the following best explains these findings?
A) Infections by bacteria, Mycoplasma pneumonia
B) Hyperactive splenic destruction of sickled red cells
C) Renal hypoperfusion with resultant decrease in EPO
D) Radiology of long bones would reveal osteomyelitis
E) Detection of Parvovirus B19 in serological tests
Q8) A previously healthy 30 year old man complains of weakness,
dizziness, abdominal pain and passing coffee colored urine since two
days. There is no history of travel or being on any medication. P/E
vitals and system evaluation were within normal limits. Labs: Hb 11.4
gm% (N 14 to 17 gm%), total white cell count 4,500 cells/mm3 (N
4,000 to 11,000 cells/mm3), peripheral smear shows few
polychromatophils, urine analysis revealed presence of hemoglobin.
Flow cytometry of blood revealed that 67% of red cells lacked CD55.
Which one of the following complications is likely this disease?
A) Inability to concentrate urine
B) Heart failure due to iron deposition
C) Bone marrow shows dyserythropoeisis
D) Development of acute myelogenous leukemia
E) Raynaud’s phenomenon
Q9) A 28 year old man complains of loss of weight, weakness, and
episodes of abdominal pain associated with diarrhea since several
months. Labs: Hb 10.6 gm% (N 14 to 17 gm%), hematocrit 30%
(normal 37% to 47%), mean corpuscular volume 132 fl (N 80 to 100
fl), mean corpuscular hemoglobin volume 25 pg/cell (N 27 pg/cell to
32 pg/cell), mean corpuscular hemoglobin concentration 30% (N
32% to 36%). Stool samples obtained for ova and cyst examination
were sent to the clinical path lab. Also, blood was sent for culture. A
wet mount showed the presence of operculated eggs, broader than
long segments, and scolex was observed. The blood culture was
reported as negative. Which one of the following features is likely to
be noticed in this case?
A) Pappenheimer bodies in red cells
B) Elevated anti parietal antibodies
C) Lack of response to 256 Hz tuning fork test
D) Microcytic maturation of red cells in marrow
E) Difficulty in swallowing, esophageal webs
Q10) Which one of the following lab features is characteristic in
intravascular hemolysis?
A) Thrombocytopenia
B) Neutropenia
C) Increased conjugated bilirubin
D) Depressed erythropoiesis
E) Decreased hemopexin
Q11) A 20-year-old woman seen at the mother and child rural clinic
show the following findings:
Body weight: 61.2 kg (135 Ib).
Blood pressure: 110/70 mm Hg.
Mildly enlarged spleen
Laboratory studies:
Hemoglobin: 11 gm%
Hematocrit: 30%
Total white cell count: 10,000 cells/mm3
Total platelet count: 230,000 cells/mm3
Reticulocyte count: 2.5%
Peripheral blood smear: occasional target cells
Serum haptoglobin: undetectable
Serum lactate dehydrogenase: 645 U/L (increased)
Urine: hemosiderinuria
HAM test: negative
What one of the following is the most likely diagnosis?
A) Beta thalassemia major
B) Aplastic anemia
C) Aplastic anemia
D) Paroxysmal nocturnal hemoglobinuria
E) Acute myeloid leukemia
Q12) A 57 year old homeless man is seen in the walk in clinic for
extreme lethargy and loss of appetite since three months. The patient
mentions chronic alcoholism and smoking weed. P/E: afebrile, pallor,
palpable liver, and presence of minimal fluid in abdomen. Labs: Hb
11 gm% (N 14 to 17 gm%), total white cell count 8,000 cells/mm3 (N
4,000 to 11,000 cells/mm3), elevated liver enzymes, MCV 76 fl (N 80
to 100 fl) increased serum iron, increased serum ferritin, and
decreased TIBC. Which one of the following is the likely diagnosis?
A) Sideroblastic anemia
B) Iron deficiency, chronic
C) Vitamin B12 deficiency
D) Sickle cell disease, trait
E) Aplastic anemia

NOTE: This Quiz is for students in Pathology 1. Please do not


distribute this Quiz to others. Kindly honor this request.
Dr. Alfred Roy, MD
Professor of Pathology
UMHS

1. A 7. E
2. A 8. D
3. C 9. C
4. D 10. E
5. C 11. A
6. A 12. A
Quiz 3

WINTER 2021
Home / My courses / Pathology I / Block Three / Quiz 3. Winter 2021.

Started on Monday, 22 March 2021, 5:07 PM


State Finished
Completed on Monday, 22 March 2021, 5:11 PM
Time taken 3 mins 43 secs
Marks 10.00/10.00
Grade 100.00 out of 100.00

Question 1 A 65-year-old man complains of increasing fatiguability. Patient tells you that he never misses out
Correct on his weekend tennis with his friends, but in the past one month he is unable to play due to
Mark 1.00 out tiredness.
of 1.00
Past medical history is significant for successful treatment of Stage I colonic adenocarcinoma, and
herniorrhaphy.
Labs:
Hb 12 gm% (N 14 to 17 gm%)
Total white cell count: 102,000 cells/mm3 (N 4,000 to 11,000 cells/mm3)
Total platelet count: 180,000 cells/mm3 (N 150,000 to 400,000 cells/mm3)
Urine analysis: normal
LDH: raised
Peripheral blood smear shown:

Which one of the following test would be most appropriate in this case?

Select one:
a. Liver biopsy with Prussian blue

b. Direct Coomb test !

c. Thick blood smear for malarial parasites

d. Bone marrow biopsy

e. CSF for proteins


Your answer is correct.
The correct answer is: Direct Coomb test
Correct
Marks for this submission: 1.00/1.00.

Question 2
A 30-year-old man has had a progressively worsening productive cough for one month. On physical
Correct
examination, a few small non-tender lymph nodes are palpable in the axillae, and the tip of the
Mark 1.00 out spleen is not palpable. Laboratory studies show Hgb 10.2 g/dl, Hct 31.1%, MCV 90 fL, WBC count
of 1.00
67,000/microliter, and platelet count 36,000/microliter. Microscopic examination of his bone
marrow aspirate shows 26% of blasts (with Auer rods) and promyelocytes together. Which of the
following is the most likely molecular abnormality seen in the tumor cells?

Select one:
a. t(8:14)

b. t(9:22)

c. Deletion 5q

d. t(15:17) !

e. Inv(16)

Your answer is correct.


The correct answer is: t(15:17)
Correct
Marks for this submission: 1.00/1.00.

Question 3
A 15-year-old boy, born of the non-consanguineous marriage, 2nd child of his parents, right-
Correct
handed, resident of California, 10th-grade student, symptomatic since 4 months, presented with
Mark 1.00 out complaints of fever with chills on and off and bilateral inguinal and axillary lymphadenopathy. He
of 1.00
had no significant past or family history. He was admitted to a hospital where lymph node biopsy
was suggestive of reactive lymphadenitis. Investigations revealed a total WBC count
2530/ microliter and platelet count of 138,000/microliter. Lepto IgM, Dengue IgM, HAV IgM, HEV
IgM, HCV Antibody negative, USG Abdomen- Normal, urine Culture/ sensitivity - negative, total
Bilirubin- 2.68mg%, direct Bilirubin-2mg%,ALT 663U/L,AST 210U/L. The patient was suspected to
have acute leukemia. And bone marrow aspiration was done findings are given.
His condition got deteriorated. Further lab investigations show elevated blood levels of Interferon-
gamma, TNF-alpha, IL-6, IL-12, IL-2 receptor. Which of the following is true in this patient?
Select one:
a. Leukemia

b. Lymphoma

c. Macrophage Activation syndrome !

d. Bacterial infection

e. Fungal infection

Your answer is correct.


The correct answer is: Macrophage Activation syndrome
Correct
Marks for this submission: 1.00/1.00.

Question 4
A 17-year-old adolescent has had malaise for the past 3 weeks. He has a mild sore throat on
Correct
physical examination, as well as tender axillary and inguinal lymphadenopathy. A CBC shows Hgb
Mark 1.00 out 14.0 g/dL, Hct 42.2%, MCV 90 fL, platelet count 301,300/ microliter, and WBC count 8120/
of 1.00
microliter. The peripheral blood smear findings are given. Which of the following is the most likely
cause of illness in this patient?

Select one:
a. Hematological Malignancy

b. Tuberculosis

c. Infectious Mononucleosis !

d. Lymphoma

e. Acute Appendicitis

Your answer is correct.


The correct answer is: Infectious Mononucleosis
Correct
Marks for this submission: 1.00/1.00.

Question 5 A 30-year-old woman complains of progressive weakness. Patient mentions that she has been
Question 5 A 30-year-old woman complains of progressive weakness. Patient mentions that she has been
Correct feeling extremely tired for the past 3 months. She also complains of occasional abdominal
Mark 1.00 out cramping pain and passage of dark colored urine in the morning.
of 1.00
Medical history is significant for appendectomy done during school days, and cholecystectomy
three years ago.
P/E: pallor, mild icterus, vitals are normal, abdomen is soft and no visceromegaly.
Labs:
Hb: 12 gm% (N 14 to 17 gm%)
Total white cell count: 3,800 cells/mm3 (N 4,000 to 11,000 cells/mm3)
Total platelet count: 120,000 cells/mm3 (N 150,000 to 400,000 cells/mm3)
LDH: elevated
Haptoglobin: mild reduction
DAT: negative
A likely complication in this patient includes

Select one:
a. Adrenocortical insufficiency

b. MALTOMA

c. Carcinoma colon

d. Venous thrombosis !

e. Disseminated Intravascular Coagulation

Your answer is correct.


The correct answer is: Venous thrombosis
Correct
Marks for this submission: 1.00/1.00.

Question 6
A 6-year-old male with a 5 day history of vomiting and diarrhea was transferred to your medical
Correct
center from a rural hospital. Several other family members have been hospitalized. All were eating
Mark 1.00 out ice cream made with raw eggs from their farm. The patient brought to emergency with a high fever
of 1.00
and in shock. He had no lymphadenopathy or hepato - splenomegaly. His CBC showed a Hgb 13.1
gm/dl, Hct 40%, WBC 22,000/µl with 40% neutrophils, 26% bands, 10% Myelocytes, 0% blasts,
20% lymphocytes, 4% Monocytes. Platelet count was 135,000/µl. Which of the following is the
best lab test to differentiate this condition from leukemia in this patient?

Select one:
a. Marked Leukocytosis

b. Shift to left

c. High LAP score

d. Absence of Philadelphia chromosome !

e. Absolute Basophilia

Your answer is correct.


The correct answer is: Absence of Philadelphia chromosome
The correct answer is: Absence of Philadelphia chromosome
Correct
Marks for this submission: 1.00/1.00.

Question 7
A 42-year-old man has had dragging sensation of the left upper abdomen for the past 4 weeks. On
Correct
physical examination, his temperature is 37.8 C. Laboratory studies show a Hgb of 12.2 g/dL, Hct
Mark 1.00 out 37.1%, MCV 92 fL, platelet count 243,000/ microliter, and WBC count 125,000/ microliter. The WBC
of 1.00
differential count shows 82 neutrophils, 8 bands, 3 Metamyelocytes, 1 myelocyte, 4%
lymphocytes, and 1% Monocytes. The leukocyte alkaline phosphatase (LAP) score is 0. On
examination, his spleen is markedly enlarged. Which of the following molecular abnormality seen in
this man?

Select one:
a. Inv(16)

b. t(15:17)

c. t(9:22) !

d. Deletion 5q

e. t(8:14)

Your answer is correct.


The correct answer is: t(9:22)
Correct
Marks for this submission: 1.00/1.00.

Question 8
A clinical study is performed of subjects from age of 50 to 80 years who were documented by
Correct
laboratory testing to have myeloproliferative disorders. The medical records of these subjects are
Mark 1.00 out analyzed and the disease conditions documented in these subjects recorded. A subset of these
of 1.00
patients had developed both acute myeloid and lymphoid Leukemias 3 to 5 years after diagnosis of
myeloproliferative disorders.
Which of the following is most appropriate about the tumor cell in this subset of patients?

Select one:
a. Pluripotent stem cell !

b. Histiocyte

c. Myeloid stem cell

d. Lymphoid stem cell

e. Committed cell

Your answer is correct.


The correct answer is: Pluripotent stem cell
Correct
Marks for this submission: 1.00/1.00.
Question 9
A 64-year-old has fatigue and bleeding from nose intermittently for the past two years. On physical
Correct
examination, the spleen can be felt below the left costal margin. An abdominal CT scan reveals
Mark 1.00 out massive (estimated 4000-gram size) splenomegaly. Laboratory studies show Hgb 20.2 g/dl, Hct
of 1.00
61.1%, MCV 90 fL, WBC count 17,000/ microliter, and platelet count 476,000/ microliter. Which of
the following is appropriate in Primary Polycythemia?

Select one:
a. Pluripotent tumor cell

b. 20% myeloblasts in the bone marrow

c. Highest risk of acute leukemic transformation

d. JAK-2 mutations !

e. High Blood Erythropoietin levels

Your answer is correct.


The correct answer is: JAK-2 mutations
Correct
Marks for this submission: 1.00/1.00.

Question 10 A 76-year-old man tells you that he two episodes of black out. Patient tells you that he had been
Correct experiencing progressive weakness for the past 6 months, and had 2 episodes of fainting recently.
Mark 1.00 out Medical history is significant for Type II Diabetes, and Hypertension. Patient is on beta blockers,
of 1.00
and metformin for diabetes.
P/E: pallor, BP 180/140 mmHg, weak and delayed distal pulse, palpable systolic thrill over the
bifurcation of the carotids, and murmur in second right intercostal space.
Investigations:
Oxygen saturation at room temperature: 94%
CBC: normal
MCV: 94 fL (N 80 to 100 fL)
Urine: blood detected
LDH: elevated
Reticulocyte count; increased
An additional finding in this case would be

Select one:
a. Occlusion of left anterior descending coronary artery

b. Positive DAT

c. Bombay blood group

d. Schistocytes in blood smear !

e. Varicose veins

Your answer is correct.


The correct answer is: Schistocytes in blood smear
Correct
Marks for this submission: 1.00/1.00.

◀︎ WBC disorders - recording3/22/2021 Jump to... WBC disorders Neoplastic part-3 ▶︎


Bleeding Disorder General Information Incidence Pathogenesis Lab Investigations Clinical Features

Bleeding disorders *seen in *systemic Platelet count: normal • Immune


caused by vessel wall childhood & hypersensitivity disease disorder
abnormalities adolescence of unknown cause Bleeding time: normal
• Children
Picture 1: *frequently *characterized by Coagulation work-up:
Perivascular following widespread normal • Follows
infiltrates forming a upper inflammatory reaction *Activated PTT infection
collar around respiratory of the capillaries & small
subcutaneous blood tract infection vessels Skin biopsy: • Vasculitis
vessels - IgA, C3, fibrin
*purpuric rash (ALL deposits • Purpura
Cuff of a short sleeve: OVER THE BODY), Renal biopsy:
Henoch- around the blood abdominal colic, - IgA, C3 deposits
Schönleinpurpura vessel polyarthralgia, acute • GI bleed
glomerulonephritits Immunofluorescence
Inflammatory cells • Glomerulone
but NO lumen: here *deposition of immune Picture 2: phritis
you find fibrinoid complexes (IgA) Skin biopsy: Pt. with
necrosis (completely acute GI bleed: occur
occluded the small glomerulonephritisà because mucosal
vessels) *Appendicitis seen!!!**** note upper dermal vessels might rupture
*Hematuria can be blood vessels with a
Characterized by present collar of inflammatory
immune deposition infiltrate. Blue arrow ***Buttocks and
into endothelium ****Rash is a consistent shows a blood vessel Lower limbs!***
tissue finding!!!***** obliterated by
inflammation
&fibrinoid necrosis

Thrombocytopenia

Bleeding related to reduced platelet number

*decreased production of platelets (aplastic anemia, leukemias)

*decreased platelet survival (immunologic or non-immunologic)

*sequestration (hypersplenism)

*dilutional (massive transfusions)

Immune 1.Primary or *seen in *formation of anti platelet *Thrombocytopenia *petechial
Thrombocytopenic Idiopathic children, antibodies (varying degrees) hemorrhages on skin,
purpura : *2 clinical sub usually epistaxis
types: Acute or following *self limited, resolves *bleeding time:
Chronic viral infection spontaneously within 6 prolonged
*Acute is months
usually self- *coagulation study:
Acute Immune limiting, seen *steroid therapy indicated, normal
Thrombocytopenic in children if, thrombocytopenia is
purpura severe
*Chronic shows *If platelets don’t
recurring *Leukemias: improve: PROBLEMS!
bleeding over *Bone marrow
months & infiltration
years *Hypersplenism
*SLE

2.Secondary
most common
*Drug induced: most
causing aplastic
anemia
*young to *formation of ‘auto Lab investigations: *easy bruising,
middle aged antibodies’ menorrhagia, melena,
women Platelet count: bleed from mucus
*these are directed against decreased membranes
*vary from ‘platelet membrane
mild to severe glycoproteins’ (IgG Ab Bleeding time: *subconjunctival &
retinal hemorrhages
directed against Gp IIb-IIIa, prolonged
maybe seen
80% of patients)

Partial
*complications: sub
*anti-platelet antibodies Thromboplastin
arachnoid
Chronic Immune act as opsonins, that Time: normal
hemorrhages,
Thrombocytopenic
intracerebral
Purpura *are recognized by IgG Fc Bone marrow: bone
hemorrhages
receptors on phagocytes marrow is indicated, to

rule out other causes
*transplacental
*results in platelet for thrombocytopenia
transfer of IgG may
destruction, mostly in
produce transient
spleen *slight increase in
thrombocytopenia in
megakaryocytes
infants


*corticosteroids &
splenectomy

*immunosuppressive
therapy
Microangiopathic hemolytic anemia

*mostly *deficiency of enzyme Hb: reduced Pentad:
women ADAMTS 13 (Cleaves v 1. fever
WF)à excess of v WF à White cell count:
increases platelet leukocytosis 2. thrombocytopenia
adhesion in areas of
endothelial injury Platelet count: 3. renal failure
reduced
*platelets are involved in 4. microangiopathic
thrombus formation, Bleeding time: hemolytic anemia
A) Thrombotic resulting in prolonged
thrombocytopenic thrombocytopenia 5. neurological
purpura PBS: schistocytes, low symptoms
platelets
Normal scenario: *Polychromatophils* *patient with
endothelial damage is *Nucleated red cells thrombocytopenia
followed by release of microangiopathic
ultra large vWF factor ADAMTS 13 assay: hemolytic anemia
multimers. From the not reliable (procedure
Wiebel-Palade bodies in is not sophisticated)
the endothelium. These
are normally cleaved by
the action of ADAMTS13
enzyme.


Shiga like toxins: Stx1 *children *infectious Hb: reduced *Triad:
& Stx2 gastroenteritis caused 1. acute renal failure
*epidemic , by E.coli strain 0157:H7 Platelet count: 2. thrombocytopenia
summer reduced 3. hemolytic anemia
months, under *shiga like toxin is (thrombocytopenia)
cooked red elaborated which binds *infants and young
meat, and damages endothelial Peripheral blood children
unpasteurized cells in glomerulus…. smear: schistocytes
milk, cytokines & initiating *bloody diarrhea,
contaminated platelet activation & Bleeding time: vomiting, fever,
water, aggregation (thrombotic prolonged hypertension
environment)
B) Hemolytic S. creatinine: raised *plasmapheresis
uremic syndrome Remember: HUS may
look like DIC. But, there is Urine analysis:
no consumption of proteinuria
coagulation factors in HUS Stool examination:
EHEC O157:H7

Bleeding disorders related to defective platelet function:

A) Defective adhesion: Bernard-Soulier syndrome
B) Defective aggregation: Glanzmann’s thromboasthenia
Defective platelet secretion: Storage pool defects

*rare autosomal *severe bleeding CBC:
A) Defective recessive, tendency due to Total platelet count:
adhesion consanguinity decreased (moderate
*defective adhesion of to severe)
*thrombocytopenia, platelets to
giant platelets, & a subendothelium, PBS: giant platelets
failure to bind to GPIb
ligands, more *due to decreased Bleeding time:
importantly to von binding to vWF prolonged
Bernard-Soulier Willebrand factor &
syndrome thrombin *deficiency of Coagulation work-up:
membrane Gp 1b and normal (NOT
associated Gp IX affected)

Platelet aggregation:
abnormal study
(*some complexes are
affected)

Clinical Features:
- epistaxis,
ecchymoses,
menorrhagia,
B) Defective *rare, familial, *platelet membrane *total platelet count:
aggregation autosomal recessive lacks: Gp IIb-IIIa normal*
consanguineous
marriage *Gp IIb-IIIa is a receptor PBS: normal looking
for fibrinogen platelets

Glanzmann’s *Bleeding time:
thromboasthenia prolonged*

Platelet aggregation:
abnormal study

Coagulation work-up:
normal


Bleeding disorders Drugs: Uremia:
(Platelet function) *aspirin causes platelet reduction in ADP
aggregation defect induced platelet
A) von Willebrand aggregation
disease *inhibits COX pathway,
which blocks synthesis Infections
B)Hemophilia A of thrombaxane A2 - in the setting of DIC
(TXA2)
C)Hemophilia B
Role of vWF: *inherited, *characterized by: *platelet count: *spontaneous bleed
-synthesized by autosomal normal from mucous
endothelial cells and dominant 1) defect of platelet membrane: epistaxis
megakaryocytes disease function leading to *bleeding time:
prolonged bleeding prolonged (this is due *bleed from trivial
Functions: *clinically to a platelet adhesion wounds,
1.serves as critical resembles 2) coagulation defect defect) menorrhagia, dental
bridge between Hemophilia, due to deficiency of procedures, minor
circulating platelets milder factor VIII activity in *PT: normal surgeries
and injured blood disease plasma
vessel wall *PTT: increased

A) von Willebrand 2.serves as a carrier *vWF activity:
Disease for factor VIII in reduced
plasma
*Factor VIII activity:
*once in circulation reduced
factor VIII, it binds to
vWF (made by * Factor VIII activity is
endothelial cells) the most sensitive

*platelet adhesion
molecule

*vWF stabilizes
factor VIII, &
promotes adhesion
of platelets to
endothelium

*X linked recessive Lab investigations: *concentration of
trait clotting factor in plasma
Platelet count: ranges from
*females are normal complete deficiency to
asymptomatic 30% of normal
carriers Bleeding time:
normal *activity below 1%
*transmits the is associated with
abnormal X Prothrombin time: severe disease
chromosome to 50% normal
*easy bruising,
of male offspring
hemorrhage following
Partial trauma or operative
*no family history in Thromboplastin procedures
B) Hemophilia A 30% Time: prolonged
*‘spontaneous’
Factor VIII levels: hemorrhages to
*clinically reduced knee joints
Hemophilia A & B are detection of female (hemarthroses)
indistinguishable carriers
*chronic hemophilic
arthritis
(soft tissue hematomas &
hemarthroses)

*hematuria

*Bleed into CNS
(most serious
*petechiae are
absent
*clinically similar to
*decrease in Factor Hemophilia A
IX activity
C) Hemophilia B
*X linked recessive

*Factor IX is reduced

*complex systemic *coagulation may be Platelet count: Morphology:
thrombohemorrhagic initiated through: decreased thrombi seen in
disorder extrinsic (obstetric various organs
complications, Prothrombin time - Brain
*disseminated malignancy)or intrinsic (PT): prolonged - Heart
intravascular clotting pathway (gram neg - Lungs
causes a hemostatic septicemia) Partial - Kidneys
defect, resulting from thromboplastin time
Disseminated utilization of the *result in formation of (PTT): prolonged DIC may be
Intravascular coagulation factors & thrombin, which converts associated with:
Coagulation platelets fibrinogen to fibrin Thrombin time-
(consumption prolonged 1. Waterhouse-
coagulopathy) *widespread deposition of Friderichsen
fibrin in Fibrinogen levels: syndrome
Etiology: microcirculationà reduced
* NOT a primary ischemic insult 2. Sheehan
disorder D- Dimers: increased postpartum
*fibrin in microcirculation: pituitary
*secondary to various microangiopathic necrosis
causes hemolytic anemia
Fibrinogen
*obstetric *platelets and degradation
complications coagulation factors are products: increased
(abruptio placentae, consumed
amniotic fluid Peripheral blood
embolism, abortion, *platelets damaged and smear: fragmented
intra uterine death) used in microthrombi rbc’s

*infections (sepsis) *release of plasmin to
clear fibrin (fibrin Treatment:
*neoplasms (mucin degradation) - treat underlying
secreting cause
adenocarcinoma) *factors are also used by - volume
plasminà hemostatic replacement
*massive tissue injury failure - correction of
hypotension
*snake bite (viper) and
oxygenation
- blood
component
therapy
- heparin (some
cases)


serum us plasma
clottingfa
Bpg

ie In MyeloidLeukemia
I Anemias
Polycythemia

b
caused
s 5h0
septic a

Neutrophil
corticosteroid
carbamazepine

Hematopoiesis
Hemolytic'Anem Anemia
Hypoplasia
HighRisk FungalInfections
µµmi aplastic Candida Aspergillus
Ttype1 INF Tcellproliferation

leucocytosis
LAPLeukemoidalkalinephosphatase

of
h in
near
µµµ to

Malignant
Lymphomas
Hodgkin's
NHL
systemicinflammatory response syndrome
produce
onto
proteins
CLL
GrowthSignaling
ApoptoticCell
Death NaiveB Cells
CD19
CDs CD20 peripheralsmear
OF mutations B le smudgecells
selfrenewal oftumorcells markers
TargiledRuptured
kinaseactivity
Tyrosine
kits Nffppl.HN AWBC N100000
1314AM
MAPK
arburgMetabolism
acutemyeloid
leukemia

Histiocyte Tissuecell immaturedendriticcell

DNAlesions
chromosomalbreaks
Mycagtranslocations

NAlesions 4instability
Bloom's
sdirected panwn.isAnemia
Telengectasia
CL 6 ataxia
hebreaks
factor
unscription

Glutensensitive

Koogeenniicc
Kiimusseess

Lill
laitTeenleukemialymphoma
carcinogen Benzenecompounds
Kitty
king
0dg
NKeenLymphomas
AHHH118
emlymphoma

lamination
yionGastric
133 Cell
lymphoma

u patients

nsensitiveEnteropathy Chromosomal Abnormalities


asiimplants Deletions 5979,209
Teenlymphoma
Monosomy 547
Trisomy 8

Ossofgenecoding
bosomal protein

iRI.fi ve it cellular
erythropoiesis
MY
period µpµµsP

imiled in
lerlytransfusion
Antibiotics
loud
omg stem cell
transplant

mmmunnoommodulator
DNAmethylationof

T
C hypercellularmarrow hemopoieticprogenitorcells marrowFibrosis
insecondaryhemopienisites cytopeniasin
blood
peripheral

philadelphiach22

TReticulin

CML
Adults
40years
insidiousonset
nemiasensation
agging abdomen
eetsideor
cinfarcts
lenomegaly spun
Abdomen
H
RABLtranslocations DrugResistance
BCRABLmutations

leukemia

myeloid
translocation
Acute
peurRO
AN 15
ATRA
d
putra
µµoicA
Mutations
in TK JAK
proliferationin
erythroidcellline

function
REPOReactivesecondary
EPO
µyBt In Hct

MHCt 41311
Extra Medullary Hematopoiesis

Mutation
pit
gµcz tb
EP0qRB

res
padded
poison

gyppposis
20
trials
gAK
clinical
Tumorcells
lackJAks
mutations

Mutated Thrombopietin independent

TX Gentle Chemotherapy
Neoplastic
megakaryocytes

Factors
biogenicGrowth
1GFPPDGF
x marrow
blastsin
bro
Deposits
acollagen
fibrosis
myelo 460years
Anemia
splenomegaly
tweight
sweats secondary
night Govt
hyperurecemia
Leucocytosis

Extra t
edu
mappoeis
µ

00
Sio
ai

serine
threonine
kinase

nemia Histiocytes
hrombocytopenia
rections
ansformtoLangerhan
Cell
sarcoma
cells
withanaplastictumor
1PML AML M3
presentswith Dic

HMM
retinoic
d
Aeur H
fever
is r r
ri

Gum Hypertrophy

R
igen
eptor
ene
AGR
arrangements
onocional
rearrangement Rs cell Reed
queDNAsequence

I
Leukemia
NNW

2 gone
3 nympho'M

4 gypped
5
µH
6

POMP
lymphoma 8

3
4

56
Teens
myth

c to
a
NOTCH't
I
I Down's
v
Mw
ai
syndrome

oiiEE.it
un
R
gymphoblasts

stain't

fat

814
Burkitt's Cmyc
lesion
jawmacrophages
RapidTinsice
EBY

is
yeawMPH
Tumorsuppressor Genes
gµµe't
9 Deletions 13 11 17
µ rt trisomy129

disruptednucleioftumourcells

PanBCD 1920,23 5
x IgM D

BTK BTKdetective inKlinkedAgammaglobulinemia

DiffuseLarge node spleen


lymph
BceuLymphoma mass
additional mutations

BMinfiltrates

Tumorcellexpress CD19
CD20
CD10
SurfaceIg
BCL6
CD19,20 10
Chromatin
Clumped Nuclear
2 5Nucleoli
RoyalBlueCytoplasm

Treatment Hsetransplantation
inhibitors
Proteasome

chromosome 11

lymphadenopathy
CentroblastsAbsent BlastoidVariant
paid cens
intermediatesize
chromatin
open
4Mitosis
extranodal poorprognosis

Activates Nk Kp

Stomach

SHH Thyroid Salivarygland

Bone Marrow
Langerhan
cellHistiocytosis Valine
to
Fibrosis Glutamate
Drytap
TRAPH
spleen whilepulplooks redbeefy Bone
tract marrow
Liver hairycellsinfiltrateportal Aspirate
Monocytopeniak

MY
MENS
Waldenstrom
MGUS
monoclonal
Gammopathy

MSpike IgM
many
hyperviscosity
blurry vision

CRAB

µfindings
earlystageofMM

1m44
it hypercalcemiamultipleosteolyticlesions
defects
A nd
p activates
renalfailure acquiredimmune
µ lesions
oyeonasts
lytic Ch14932 cyclinDi Dzabnormalities
pan
causing
B
Tubular proti
Epithelium
gone

gem
levels
11g
recurrent
Bacterial
infections

but I immunity
toxicto tubular
cells
epithelial

419M

ProteasomeInhibitors

kFractures 0 Hypercalcemia

Strom's

macrogiobulinemia
Walden
lesions

one
www.oyiiot
µ
com

IgM
Abnormal Platelets

1 Nodular sclerosis
2 mixedcellularity
Rich
pelebst'M 3 Lymphocyte Depleted
fever 4 Lymphocyte predominance
5 Lymphocyte
node
painless
lymph
enlargement

off EBYexpresses LMP1 latentmembrane


7rominent protein 1
Nucleoli
marginal
mantle
Burkitt
HTLV
TCcontain
proviruses
ischemic necrosis
cytoplasmic granules
large azurophilic
EBV episomes
All tumorcellshavetumor
cell lackCD21
but EBx infected
Markers 1 cells
CD3 TCR NK16,56
Keyla Galloza Acevedo

RBC Disorders Block 3

Immune Hemolytic Anemia’s

Remember that Mast Cells are derived from the Myeloid Progenitor cell

Bone Marrow Aspirate Bone Marrow Biopsy

-In an aspiration you don’t see Bony Trabeculae or In a biopsy you see Bony trabeculae en Empty
Empty Spaces Spaces (Addipose tissue + Hematopoietic
Aspiration is usually taken from Sternum (only in Elements)
children >12yrs) or Iliac crest -Can be done in R Iliac Crest or Anterior Surface of
-No sternum aspiration in children <12yrs because the Tibia (no specific age)
needle could puncture heart -You can give general anesthesia or local
Keyla Galloza Acevedo

-I can confirm Immune Thrombocytopenic purpura anesthesia and it might take around 10min
via needle aspiration -With a biopsy I can identify the Marrow activity
(Ratio of hematopoietic elements to fat cells/
Cellurity):
a. Normocellular (50% Hematopoietic cells), 1:1
ratio
b. Hypercellular (>75% Hematopoietic cells) ej.
Hemolytic anemias, Leukemias
c. Hypocellular (<25% Hematopoietic cells)
ej. Aplastic anemia

1. What is it? Bone marrow is the spongy


tissue inside some of your bones, such
as your hip and thigh bones. It contains
stem cells. The stem cells can develop
into the red blood cells that carry
oxygen through your body, the white
blood cells that fight infections, and
the platelets that help with blood
clotting
2. Location: It is located in the medullary
cavities (centers) of certain large
bones.
-The bone marrow undergoes changes
with age. The most readily apparent
change is a decline in marrow cellularity.
The percentage of marrow space occupied
by hematopoietic tissue goes from 40–60%
in young adults to 20–40% in older people,
with the remaining space being taken up
by fat (yellow marrow)
-Adults: Red marrow is found mainly in the
flat bones, such as the hip bone, sternum
(breast) bone, skull, ribs, vertebrae, and
shoulder blades, as well as in the
Bone Marrow metaphyseal and epiphyseal ends of the
long bones, such as the femur, tibia, and
humerus, where the bone is cancellous or
spongy
-Children <18yrs: BM is found all around
the bone
3. Composition: Bone marrow contains
two types of stem cells: hemopoietic
(which can produce blood cells) and
Keyla Galloza Acevedo

stromal (which can produce fat,


cartilage and bone).
-65% : Granulocytes and their precursors
-25% : Erythroid precursors
-10% : Lymphocytes and Monocytes

-Myeloid: Erythroid ratio (3:1):


>Myeloid-Myeloctes, Metamyelocytes &
Granulocytes
>Erythroid-Polychromatophilic (Reticulocytes)
and Orthochromic normoblast

Clinical Significance of M:E :


a. Increased: Myeloid Leukemia
b. Decreased: Anemia’s, Polycythemia
(Polycythemia vera could progress to
Acute Myeloid leukemia )
4. How to obtain it: The bone marrow
aspiration is usually done first. The
doctor makes a small incision, then
inserts a hollow needle through the
bone and into the bone marrow. Using
a syringe attached to the needle, the
doctor withdraws a sample of the
liquid portion of the bone marrow. You
may feel a brief sharp pain or stinging

-A bone marrow biopsy involves removing a


small sample of the bone marrow inside your
bones for testing. The biopsy is done using a
small needle inserted into the bone.

-EDTA (ethylenediamine tetraacetic acid): has


been recommended as the anticoagulant of
choice for hematological testing because it
allows the best preservation of cellular
components and morphology of blood cells.

-Immunophenotyping: A process that uses


antibodies to identify cells based on the types
of antigens or markers on the surface of the
cells. This process is used in basic research and
to help diagnose diseases, such as specific
types of leukemia and lymphoma.
Keyla Galloza Acevedo

5. Why is it tested: Bone marrow tests


check to see if your bone marrow is
working correctly and making normal
amounts of blood cells. The tests can
help diagnose and monitor various
bone marrow disorders, blood
disorders, and certain types of cancer.

a.Serum or plasma: 55%


Peripheral Blood
b.Buffy coat: 1%
-Contains Platelets and WBC
-Best site to check for Hemoparasites

c.Hematocrit (Packed cell Volume/ PCV) or


Red Cell Volume: 44%
-Supravital stain is used to stain for
Reticulocytes
-Reticulocyte count will tell us if the BM is
healthy or damaged. Normally the RC is
around for 3 days (2 days in the BM and 1 day
in peripheral blood and then mature to a RBC)
-In hemolytic anemia’s, RC are usually
increased because the BM is trying to
compensate
-It’s important to look at the Red Cell Mass in
order to identify Hemoconcentration and
Hemodilution
-44-44% in Males
-38-42% in Females
-Hbx3 =15mg/dl Htc=15x3=45%
-Hb/3 = TRBC
-Commonly used stains: Giemsa or Leishman

1. Difference between serum and


plasma:
a.Serum: (-) Fibrinogen (clotting factors
are consumed) Plasma – CF
b.Plasma: (+) Fibrinogen (Clotting factors
-RBC (Erythrocytes) are preserved) Serum + CF
-WBC (Leukocytes)
-Platelets (Thrombocytes) 2. How to know the functional status of
the marrow by looking at the
Keyla Galloza Acevedo

peripheral blood?
- A blood smear is considered abnormal when
there's an abnormality in the size, shape,
color, or number of cells in your blood.
Abnormal results may vary depending on the
type of blood cell affected

<80 MCV = Microcytic

>80 MCV = Macrocytic

-Anemia of Chronic disease


could be both micro and
normocytic
Keyla Galloza Acevedo

Migration of stem cells:


a. Early months: Yolk sac
b. Third month: Liver
c. Fourth month: Bone Marrow
d. By birth: Marrow is the sole source of
blood cells
-Up to puberty, entire skeleton: Marrow is Red
and Active
-By age 18: Marrow is limited to vertebrae,
ribs, sternum, skull, pelvis and proximal
Hematopoiesis epiphyseal regions of the humerus and femur
>Remaining marrow yellow, fatty and inactive
-Adults: 50% marrow space is active , other
50% is inactive

Clinical significance:
a. Premature infant: Hematopoiesis in
liver (rarely spleen, thymus and LN)
b. Extramedullary Hematopoiesis (at
least 2 cell lines): Abnormal in the full
term infant
c. Stem Cell dysfunction:
-Marrow failure (Aplastic anemia
Hypocellular BM)
-Hematopoietic neoplasms
(Leukemias Hypercellular BM)

d. Marrow Infiltrating Disorders


(diseases distort the architecture)
Lack of space in the BM
1. Metastatic cancer (Breast cancer in
women and prostate cancer in
men)
2. Granulomatous disease (TB,
Sarcoidosis, Cat Scratch, Fungal
infections, Lymphogranuloma
venerum, Leprosy, Syphilis)
3. Abnormal release of immature
precursors into the peripheral
blood
-Leukoerythroblastosis: anemia due to
destruction or crowding out of hematopoietic
tissues in the bone marrow by a space-
occupying lesion; reduction in normal marrow
Keyla Galloza Acevedo

cells induces release of immature


hematopoietic cells, especially nucleated
erythrocytes, into the bloodstream.
-Myelophthisic anemia: normocytic,
normochromic anemia that occurs when
normal marrow space is infiltrated and
replaced by nonhematopoietic or abnormal
cells. Causes include tumors, granulomatous
disorders, lipid storage diseases, and primary
myelofibrosis.
Keyla Galloza Acevedo

Pathology Anemia:
1. Causes: Blood loss, Hemolytic,
Low Erythropoiesis, Low B12,
Low Folate, Low Iron
2. Low Hb & PCV/Htc
Hemolytic Anemia
1. Causes: Membrane defect (HS,
PNH), Enzyme defect (G6PD),
Defective Hb Synthesis (HbS,
Thal.), Immune (IHA), Trauma
2. High Reticulocytes
3. High Indirect Bilirubin
4. High LDH
5. Low Haptoglobin
-Normal RBC: 1/3 Central pallor 6. (-) Coomb test
Keyla Galloza Acevedo

7. Erythroid Hyperplasia: High


numbers of erythroid precursors
(normoblast)
8. Extramedullary Hematopoiesis
9. Reticulocytosis
10. Schistocytes (Fragment RBC ej.
Helmet shape – can be seen in
pt. with G6PD def., HS, Burns
and snake bites)

In chronic cases of Hemolysis:


a. Hemosiderosis: Due to the
excessive breakdown of RBC
within the BV, this causes iron
released from the RBC to
accumulates
b. Cholelithiasis (pigment
gallstones): Due to the excessive
RBC breakdown, liberated
hemoglobin is broken down,
and its heme component is
eventually, degraded into
bilirubin by the liver
- 5 F's: Fat (overweight), Forty (age
near or above 40), female, fertile
(premenopausal- increased
estrogen is thought to increase
cholesterol levels in bile and
decrease gallbladder contractions),
and fair (gallstones more common
in Caucasians).

To diagnose Immunehemolytic Anemia we use


Coombs antiglobulin test
Keyla Galloza Acevedo

1. Warm Antibody Immunohemolytic Anemia


-MC form of IHA
-50% of cases are idiopathic (primary)
-Most causative antibodies are of the IgG
class
-Antibodies against Rh blood group antigens
-Antigens- penicillin and cephalosporins &
Quinidine, alpha-methyldopa (Used to treat
HTN)
Reticulocytes (Stains: New Methylene
2. Cold Agglutinin Immunohemolytic Anemia Blue, Brilliant Crisail Blue)
-IgM
-Causes: -In an Hemolytic anemia peripheral
>Acute Certain infectious disorders blood smear you can see: Acanthocytes
(Mycoplasma, Infectious Mononucleosis, (Spur cells-PKD, Liver Failure),
CMG, Influenza virus, HIV) Spherocytes (HS), Target cells
>Chronic Certain lymphoid neoplasms or (Thalassemias), Echinocytes (Burr cells-
idiopathic Anemia of Renal Failure)
-Symptoms: pallor, cyanosis of body parts
(Fingers, Toes and Ears) exposed to below 30 Immune Hemolytic Anemia:
C temp. (Raynaud phenomenon) Autoimmune hemolytic anemia (AIHA),
or immune hemolytic anemia, happens
3. Cold Hemolysin Hemolytic Anemia when the immune system does not
(Paroxysmal Cold Hemoglobinuria) work properly. It mistakes red blood
-Intravascular hemolysis with hemoglobinuria cells for unwanted substances and
after exposure to cold temperatures attacks them, causing them to die early.
-Complement dependent This leaves a person without enough
-IgG binds to P blood group antigen on the red blood cells
red cell surface at low temp. 1. (+) Coomb test
-Donath-Landsteiner antibody (These Based on cause:
antibodies form and destroy red blood cells a. Hereditary: intrinsic defects
when the body is exposed to cold b. Acquired: extrinsic factors (ej.
temperatures) Autoantibodies)
-Associated with: Mycoplasma pneumonia, Based on site of Lysis:
measles, mumps and ill-defined viral and flu a. Intravascular:
symptoms -Hemoglobinemia
-First recognized with syphilis -Hemoglobinuria
-Jaundice
-Hemosidenuria
-Low Haptoglobin is characteristic of
Intravascular Hemolysis (Double check
because for Block 2 Dr. Roy said
Haptoglobin is low in both Extra and
Intra and Hemopexin was low only in
Keyla Galloza Acevedo

Intra)
b. Extravascular:
-Jaundice
-Splenomegaly

2.WBC Non-Neoplastic Disorders


Normal WBC -Total leukocyte count: 4,000-11,000 cells/cubic
mm
-Differential leukocyte count:
a. Neutrophile/Polymorphous (50-60%)
b. Lymphocytes (20-40%) TC & BC (2:1)
-TC (CD4 &CD8 2:1)
c. Monocytes (7-10%)
d. Eosinophils (4-6%)
e. Basophils (0.5%)

Basic Functions of Leukocytes: Major component


of the body's defenses against disease. Leukocytes
protect the body against invading microorganisms
Referent cell: Cell that is used for comparison and body cells with mutated DNA, and they clean
Keyla Galloza Acevedo

of the rest of the cells up debris.


-Lymphocytes are our referent cell (8
microns) a. Neutrophils: First line of defense, pus cells
-RBC size 7 microns b. Lymphocytes: Important for Humoral and
-Platelets 1-3 microns
CMI (TC & BC)
-Monocyte 16 microns (Largest cell in the
>GAMED Ig
peripheral blood smear)
c. Eosinophils: Important in parasitic
If the RBC is bigger than the lymphocyte = infestations
Macrocytic > High in Chronic Myelogenous Leukemia
>Secretes Histamine when IgE binds to its R
If the platelet is bigger than the Lymphocyte (Degranulation)
= Giant platelet/ Megakaryocytic fragments
(Means there is a Thrombocytopenia and the Normal platelet count: 150-450k
BM is trying to compensate) <150= Thrombocytopenia
<100 = Moderate Thrombocytopenia
Giant cell in peripheral blood <10k = Severe Thrombocytopenia (Medical
Monocyte Emergency) Need to give platelet transfusions
Giant cell in BM Megakaryocyte

Cell life span (days):


-RBC = 120
-Platelets = 7-10
-Transfused platelets = 1-3
-Leukocytes = 13-20

-MHCH: >35% in Hereditary Spherocytosis (only


condition)

-Increase RDW : a. Iron def. anemia (>17%)


b. Thalassemia (15-17%)

A. Neutropenia- MC and important


-Low circulating neutrophil count (High risk of
infections ej. Constant throat infections and
pneumonias
Normal neutrophil count = 6,000
<1500 = Neutropenia
1000= worrisome
Keyla Galloza Acevedo

500 = serious infections


100 = life threatening bacterial and fungal
infections (Agranulocytosis) Patient can die
within hours to days

Causes:
a. Low or ineffective production
1. Aplastic Anemia (marrow failure) &
Infiltrative marrow disorders (No space for
BM to produce blood cells) Damage to
HSC
2.Drugs: Attack on granulocyte CFU
3.Vit. Def (B12, Folic acid), MDS (ej.
Cyclophosphamide) Ineffective Hematopoiesis
(BM won’t allow the abnormal cells to leave the
BM)
>In MDS blood cells are okay in # but they are not
properly differentiated/ matured
4.Inherited disorders: Kostmann syndrome
(Defective granulocyte differentiation)
Non-Neoplastic Abnormalities
b. Increase removal from circulation
Penias (Leukopenia) Low WBC 1. Immunological (idiopathic)
2. SLE, Drugs
3. Increase use due to infections
(Bacterial, Fungal, Rickttsial)
4. Hypersplenism Clinical pathological
condition : Massive splenomegaly
(>150g)
-Blood cells are trapped in the spleen (cytopenia)
and BM thinks it need to produce more cells
Hyperplasia
-Treatments: Splenectomy (Hematology reverts
back to normal)
-Conditions that cause Splenomegaly:
a. CML
b. Storage disorders ej. Gauchers
c. Malaria (Tropical splenomegaly)
d. Hereditary Spherocytosis

c. Drug toxicity (MCC)


1. Dose related: Alkyalting agents (anti-
cancer), Ant. Metabolites (anti-
autoimmune)
Keyla Galloza Acevedo

2. Idiosynchratic: Chlorpromazine (BM


suppression), Sulfonamides
(Antibiotic), Thiouracil (anti-thyroid)
3. Idiopathic- Auto Antibodies
4. LGL Leukemia: severe Neutropenia in
the background of monoclonal
proliferation of Large Granular
Lymphocytes (NK cells) suppression
of granulocytic progenitors

Morphology:
a. Hypercellular marrow (seen if there is
peripheral destruction or ineffective
hematopoiesis)
b. Hypocellular marrow (seen if there is
Myelo-suppresion)
c. Necrotizing, Ulcerative lesion (oral cavity)
d. Agranulocytic angina: acute infection
characterized by sore throat, fever, fatigue
e. Deep fungal infections (Candida,
Asperguillus)
f. Botryomycosis (exuberant bacterial
growth)

S&S: related to infection, malaise, fever, chills,


Agranulocytosis (severe neutropenia) life
threatening infections (ej. Pneumonia or
Septicemia) Pt. can die within hrs to days

Treatment:
1. Broad Spectrum antibiotics
2. G-CSF (Given after immune-suppressive
therapy)

B. Lymphopenia

Causes:
a. Congenital (syndromes)
b. HIV/AIDS (Low CD4)
c. Corticosteroides, Cytotoxic drug
d. Cyslosporin A (Immunosupressive)
selectively attacks CD4
e. Auto-immune disorders
f. Malnutrition (MCC)
Keyla Galloza Acevedo

g. Severe Acute Viral Infection (High Type I


INF TC proliferation migration to LN
and adhere to endothelium
Lymphopenia)
Causes:
1. Infections/ Inflammatory conditions (ej.
Acute appendicitis or pyelonephritis)
Leukocytosis (High WBC) Pathogenesis:
1. Cell storage pool BM, Thymus,
circulation, peripheral tissues
2. Rate of release into circulation
3. Marginal pool (adherent to vascular
endothelium)
4. Rate of extravasation
5. Maintained by cytokines, GF and Adhesion
molecules
6. Maintained through: Commitment (ej. IL-5
need for eosinophils), proliferation,
differentiation, extravasation

Morphology:
In severe infection/Inflammations (ej.
Kawasaki’s/Takayasu Vasculitis that usually
affects young children):
-In Hodgkins Lymphoma the 1.Changes seen in Neutrophiles:
primary cells you see on the a. Toxic granulations (abnormal thick/
peripheral blood smear are coarse azurophilic grannules)-come from
MPO/MPX
Eosinophils
b. Dohle bodies (Basophilic remnants of ER)
-MDS Normal # of cells but
immature
-MPD Increase normal of cells

a.
Keyla Galloza Acevedo

b.

2.Changes seen Lymphocytes:


a. Reactive transformed lymphocyte (Bigger
lymphocyte with abnormal nuclei, can be
confused with a malignant lymphocyte)
-Need to differentiate a Leukemoid reaction
(inflammation) from Leukemia (CML)

a.

A.Neutrophilia:
a. Immediate: Release from marrow storage
pool by IL-1 & TNF-a
b. Prolonged: IL-1 & TNF-a act on the
macrophages, marrow stromal cells, T cells
G-CSF Granulocytopoiesis
Keyla Galloza Acevedo

Hypersegmented neutrophils

B.Eosinophilia:
a. IL-5 (Eotoxin)

-What parasites cause peripheral eosinophilia? Nematodes/Roundworms

-4 conditions that cause eosinophilia? Nematodes, Drugs, Allergies (Bronchial Asthma,


Rhinitis), Hodgkins Lympphoma
Keyla Galloza Acevedo

Giant Platelet Lymphocyte

-Fluffy contents -Scant cytoplasm


-Nucleus lacks Definition -Well defined nucleus

CML Leukemoid Reaction


Most important test to differentiate is the
Cytogenetic test (Karyotyping) t(9;22) Seen in severe inflammation ej. Infectious
-For specific cases we could use FISH or PCR mononucleosis or Kawasaki’s/Takayasu
-LAP= Leukocyte alkaline phosphatase
-NAP=Neutrophil alkaline phosphatase
Keyla Galloza Acevedo

Alkaline phosphatases come from:


1. Neutrophiles
2. Blast
3. Bone (Osteoclast =Acid phosphatase,
Osteoblast = Alkaline phosphatase)

-80% cases transform into AML


-20% cases transform into ALL

20 or >20% Blast (immature tumor cells in the


BM) = Acute Leukemia
-Irregular nuclei that looks pushed to
periphery (nucleus is moving and taking
shape of the cytoplasm)

A. Acute Nonspecific Lymphadenitis

Causes: Microbes, Cell debris, Foreign material

Lymph node Disorders Sites:


a. Generalized: More than 2 groups of
A. Non-neoplastic or Inflammatory LN, common in children with Viral and
1. Acute nonspecific Lymphadenitis Bacterial infections
2. Chronic nonspecific Lymphadenitis b. Localized: Cervical (infections in oral
3. Hemophagocytic Lymphocyto cavity), Axillary/Inguinal (limb
Histiocytosis (HLH) infections), Mesenteric (acute
appendicitis)
Morphology:
a. Large lymphoid follicles with
prominent germinal centers
(Zonation)
Keyla Galloza Acevedo

-Zonation: Tells us Rx hyperplasia is happening


ej. Lymphadenitis (we need to differentiate it
from Lymphoma)

b. Tingible body macrophages


Normal Lymph node Histology (Macrophages with undigested
bacterial or necrotic material)

Clinical:
-Enlarged and tender LN, Heal with scar

B.Chronic Nonspecific Lymphadenitis


Types:
1. Follicular Hyperplasia:
-Humoral response: RA, Toxoplasma, HIV
-Germinal centers with secondary follicles
-Tingible body macrophages
-Mantle zone surrounding the sec. follicles
-Zonation ej. Acute/Chronic Lymphadenitis
(DDx. Follicular Lymphoma)
Keyla Galloza Acevedo

2. Marginal Zone Hyperplasia


-Common in Toxoplasma and HIV
-Outside the Mantle zone
-Monocytoid B cells (memory cells) found in
the marginal zone

3. Para-cortical Hyperplasia
-Cellular response
-Dilantin Rx. (Phenytoin is used to prevent
and control seizures (also called an
anticonvulsant or antiepileptic drug). It works
by reducing the spread of seizure activity in
the brain)
> Association of Dilantin (hydantoin) therapy
and lymphadenopathy in the form of
"pseudolymphoma" or malignant lymphoma
> Documented lymphadenopathy developed 1
week to 30 years (median, 5 years) after the
start of Dilantin
Keyla Galloza Acevedo

-Viral (IM), Viral Vaccines, HIV


-Large T immunoblast in Para-cortical areas

4. Sinus Histiocytes
-Prominent lymphatic sinusoids
-Due to immune response to tumors or their
products (Commonly seen with Breast or
Gastrointestinal carcinomas)

Ej. In case of Breast cancer removal during a


Mastectomy, to check if we need to perform
axillary clearance, we need to look at, at least,
20LN to see which ones are showing tumor
infiltration. If from 16 LN isolated, 8 are
showing infiltration we say 8/16. The
remaining 8, if enlarged, can be separated.
The 8 infiltrated LN will show Sinus
Histiocytosis
Keyla Galloza Acevedo

C.Hemophagocytic Lymphocyto Histiocytosis


(Macrophage Activating Syndrome)

-First arrow: BM precursor


-Second arrow: Hemophagocyte
(Macrophage engulfing RBC)

-Reactive condition (not a neoplasm)


Increase in Macrophages + CD8 + CTL
(Diagnostic)
-MC trigger = EBV infection

Types:
1. Familial (Young age)
2. Sporadic (Any age)
Keyla Galloza Acevedo

Pathogenesis:
-Systemic activation of Macrophages and
CTL’s
>Release large amounts of inflammatory
mediators (IFN gamma, TNFa, IL-6, IL-12,
Soluble IL-2R)
>IL-6: Very important marker because
predicts the outcome of a viral infections ej.
COVID-19
>This cytokine storm: Suppresses
Hematopoiesis (severe cytopenias), SIRS,
Shock
-The Macrophages phagocytose both
progenitor cells in BM and formed elements in
the periphery, leading to cytopenia

-Macrophage engulfing a Band cell and an


Eosinophil

-Mutations formation/deployment of toxic


granules in CTL and NK cells

Clinical (MC presentation):


1. Acute febrile illness
2. Splenomegaly
3. Hepatomegaly
4. Hemophagocytosis in BM
5. Anemia
6. Thrombocytopenia
7. Very high plasma Ferritin and IL-2R
8. Abnormal LFT (High total bilirubin,
Alkaline phosphatase, GGT) and TGL
Hepatitis
9. Abnormal coagulation profile (PT,
Keyla Galloza Acevedo

PPT, Fibrinogen) DIC

Prognosis:
a. Untreated: Die of shock/MOF in 2
months (familial)
b. Treated: Can develop 2 problems
-Renal failure (adults)
-Growth retardation, Mental
retardation (Children)
Treatment:
1. HSC transplantation (familial) 50%
survival
2. Immunosuppressive Rx (ej.
Methotrexate)
3. Mild chemotherapy (Sporadic forms)

Lymphadenitis Lymphoma

-Zonation (Pale, dark, pale zones) -No pale and dark areas (No Zonation)
-No capsular infiltration -Capsular infiltration
-Irregular size follicles -Follicles look similar in size
Keyla Galloza Acevedo

3.WBC Neoplastic Disorders Part 1


Neoplastic Disorders 1. Lymphoid Neoplasms
-New purposless sudden growth, Invasive, Not a. BC (CD19,20,21)
well circumscribed -ej. Follicular BC lymphoma, Burkitts
-Burkitts infects CD21
Pathogenesis: b. TC (CD4,7,8,28)
A. Chromosomal translocations (MC -CD28 is used as a 2 signal for TC
abnormality): activation
a. Genetic abnormalities/mutations c. NK/ Large granular Lymphocytes
-Loss of function (Ej. Acute (CD16,56)
promyelocytic leukemia/ APML -CD16 ADCC
t(15,17) increase proliferation -CD56 Unknown function
but arrests in differentiation) -Important for innate immunity
>This tumor has Retinoic Acid R so
can be treated with ATRA Morphology:
>Tumor cells can change from -Resembles one of the stages of development
Promyelocyte to Myelocyte of normal counter parts (Tumor cells arrested
>Blast + Promyelocyte >20 %= in a certain developmental stage)
APML -Helps in diagnosis and classification
-Gain of function
b. Genomic aberrations (oncoprotein) 2. Myeloid Neoplasm
-Activates growth signaling a. Acute Myeloid Leukemia (AML)
pathways (Ej. CML t(9;22) activates TK immature/blast
Increase proliferation of both Myeloid and -There are 8 subtypes (0-7)
Lymphoid (Remember that CML tumor cell is -Acute means rapidly developing % of Blast in
not commited, its pluripotent which is a the BM
unique feature of CML) - >20% Blast (immature cells)
-Inhibits apoptosis (Ej. Follicular BC -If we find <20% Blast in the PB, we need to
lymphoma (Non-Hodgkins) t(14;18) check the BM (RULE)
Increase BCL-2 Buttock shape nuclei tumor -Even if we find >20% Blast in the PB we still
cells Granules that acts like need to check the BM to confirm
Keyla Galloza Acevedo

Thromboplastin - catalyzes the conversion of b. Myelodysplastic syndromes (MDS)


prothrombin to thrombin dysplastic/disorganized
>Slow growing tumor (GFT <10%) = Doesn’t Morphology:
respond that well to Chemo -BM: Ineffecitive Hematopoiesis (Hypocellular)
>Chemotherapy is cell cycle specific so it -PB: Corresponding Cytopenias (Ej. Anemia,
works better on rapidly dividing cells) Leukpenia, Thrombocytopenia)

c.Genetic Aberrations (Antigen R c. Myeloproliferative disorders (MPD)


gene rearrangements & diversification proliferation of mature cells
Oncogenes) Morphology:
-Ag activates BC Enters Germinal -BM: Hypercellular Can end in
center Increase expression of AID DNA Myelofibrosis of the BM
modifying enzyme Ig gene modification -PB: Philias/Cytosis
Cell switching (production of a different -Hypercellularity is a common feature of: CML,
subclass of Ig) in 2 ways: Essential Thrombocytemia/cytosis,
1. Intragenic recombinations Polycythemia vera
2. Replacement of “C” gene segment of
IgH (Ig class switching) 3. Histiocytoses (Group of disorders)
-Somatic Hypermutation: Point -Histiocyte (tissue cell); Immature dentritic cell
mutations of IG genes Increase Ab affinity to (Langerhans)
Ag MYC activation in GC BC Lymphoma)
> AID cause lesions in DNA Chromosomal Myelodysplastic Syndromes(MDS)
breaks MYC/Ig translocations 1. Refractory Anemia
>AID misdirected BCL-6 (TF) gene breaks 2. RA + Ringed Sideroblast (RARS)
BC lymphoma 3. RA + Excess Blast (RAEB)
>Aberrant recombination of V(D)J 4. RA + Excess Blast in transformation to
recombinase abnormal joining of other Myelocytes
genes to Ag R Precursors TC tumors 5. Chronic myelomonocytic leukemia
(CMML)
B. Inherited Gene Factors -These are all Pre-maignant/Pre-leukemia
a. Genomic instability: Bloom syndrome, conditions but not everyone
Fanconi anemia, Ataxia telangectasia, -Refractory anemia: Not responding to
Xeroderma pigmentosa Hematenics (Iron, B12, Folic) even after 6
b. Increase susceptibility to childhood months Rx, the Hb is not improved and the
Leukemias: Down syndrome (Trisomy anemia cannot be corrected
21), NF1 (Neurofibromatosis/ Von -These are a group of clonal Stem cell
Recklinghasen) disorders (Myeloid progenitor cells)
C. Oncogenic Viruses
a. HTLV-1: Adult TC Characterized:
Leukemia/Lymphoma -Tropical a. Maturation defect: Ineffective
Spastic Paraplesia) Hematopoiesis (Cells are destroyed
b. EBV: Burkitts, Hodgkins, NK cell because they are immature and
Lymphoma, Nasopharyngeal dysplastic)
carcinoma (100% associated with b. Peripheral Blood cytopenias
Keyla Galloza Acevedo

EBV = Temporal association) c. High risk of AML (70%)


c. KSHV/HHV8: BC lymphoma + d. In early stages BM can be
malignant pleural effusion Hypercellular and in later stages
(Temporal association) hypocellular
d. HIV: BC lymphoma
D. Chronic Inflammation: Types:
a. H. pylori: Chronic Gastritis a. Primary or Idiopathic (unkown
Gastric BC Lymphoma (Maltoma) in cause) >50yrs, gradual onset, High
antrum and duodenum If I treat risk of AML
with antibiotics during the early b. RT or Drugs: after 2-8 Rx, Higher risk of
stages, the Maltoma will regress AML (Treatment related MDS is
b. Gluten sensitive worse)
Enteropathy/Celiac disease:
Sensitivity to Gliadin Budding of Pathogenesis:
intestinal villi Malabsorption 1. Epigenetic factors: similar mutations of
TC Lymphoma genes in AML (DNA methylations,
c. Breast implants: Foreign body Histone modifications)
irritates the IS TC Lymphoma 2. RNA splicing: mutations of 3’ end
d. HIV: TC dysregulation BC 3. Transcription factors: mutations of TF
Hyperplasia GC BC lymphoma of normal myelopoiesis
(particulary in pt. that are infected Cytogenetic abnormalities:
with EBV and KSHV) 1. Chromosomal abnormalities
E. Iatrogenic (Due to treatment or a. Deletions (5q, 7q, 20q)
induced by Dr.) b. Monosomy ( 5 & 7)
a. Radiotherapy, Chemotherapy c. Trisomy (8)
Mutagenic effects on 2. Others: Loss of function mutation of
hematopoietic and lymphoid TSG (p53)
tissues Myeloid and Lymphoid 3. MYC oncogene mutations
tumors a. Slight excess very common in
>Takes 1-10yrs to develop a 2nd neoplasms
malignancy after exposure b. MYC oncoprotein in Chr. 8 that
>Thyroid and Gonads are very sensitive normally stimulates cell
to radiation proliferation
>WBC are the most sensitive to 4. 5q deletion
radiation while RBC and platelets are a. Loss of gene coding ribosomal
the least sensitive protein RPS14 Ineffective
F. Smoking (Benzo/Benzyoparine- erythropoiesis (anemia in PB)
carcinogen): Morphology:
a. Increase risk of AML 2x compared to 1. BM: usually Hypercellular + abnormal
non-smockers differentiation of all 3 cell lines
(pancytopenia)
a. Erythroid:
- Ring sideroblast
-Megaloblastoid maturation
Keyla Galloza Acevedo

-Budding nucleated cells

Ring Sideroblast

b. Granulocytes:
-Neutrophiles (Hypogranular-2nd,
Azurophilic Toxic granules-MPO,
Dohle bodies)
>Dohle bodies are commonly seen
in Kawasaki (vascular inflammatory
condition in children)

Dohle bodies (RE remnants)

-Pseudo Pegler: Huet neutrophils


(two nuclear segments-Dumbell
shape) or segmented nuclei

c. Megakaryocyte: Uni or
Multilobulated Pawn ball like
Keyla Galloza Acevedo

d. Myeloblast: <20% of marrow


cellularity (>20% in AML)

2. Peripheral blood:
a. Cytopenias (Pancytopenia)
-MC Leukopenia is Neutropenia (Recurrent throat
infections and pneumonias)
b. Pseudo Pelger-Huet neutrphils
c. Giant platelets (Megakaryocitic
fragments)
d. Macrocytes
e. Monocytosis
f. Myeloblast (<10%)
Clinical features:
1. Common in elderly (70yrs)
2. 50% Asymptomatic
3. In symptomatic: Cytopenias
complications (Ej. Infections, Bleeding,
Anemia)
Prognosis:
1. Primary MDS:
a. Worst prognosis (High blast count
and multiple chromosomal
aberrations)
b. Median survival
-Best prognostic group ( 5 yrs)
-Worst prognostic group (9-
24months)
c. Progression to AML (10-40%)
2. Treament related MDS (Worst)
a. Median survival (4-8 months)
b. Higher Risk of progression to AML
c. Severe cytopenias (commonly lead
to death-MCC)
d. Incubation period to develop cancer
Keyla Galloza Acevedo

after treatment is 2-8yrs

Treatment:
1. Limited in elderly (Antibiotics and
blood transfusions if symptomatic)
2. Young people (Stem cell
transplantation offers cure)
3. DNA methylation Inhibitors
4. Pt. with isolated mutations (5q)
respond better to the
Immunomodulator Thalidomide
(used to treat morning sickness in
pregnant females)
-Baby developed seal like limbs
(Phocomelia) after treatment

Irregular lobulated nucleated RBC (Typical of


MDS)
Keyla Galloza Acevedo

-MDS also have <20%, but you also see cytopenia and hypocellularity. In CML you see
High Lymphocytes and Hypercellularity

-Auer rods are Granules of Myeloperoxidase arranged as Rod Shaped (Diagnostic)

AML ALL
Keyla Galloza Acevedo

4. WBC Neoplastic Disorders Part 2


1. Chronic Myeologenous Leukemia
Myeloproliferative Disorders (MPD) (CML)
Molecular Mechanisms
>90% of cases: t(9;22) Philadelphia
1. Chronic myelogenous leukemia (CML)
chromosome
2. Polycythemia vera (PV)
-BCR (22), ABL (9) BCR-ABL (can be seen by
3. Essential Thrombocythemia (ET)
karyotyping) has TK activity
Normal pathway:
-GF/ligand binds with its R on the cell
Activation of TK by dimerization and
autophosphorylation Activated TK signals
pathways (RAS, MAP) by phosphorylation
Act on hemopoietic progenitor cells leads to
proliferation and survival

Mutations in TK/Signal pathways


-Hemopoietic progenitor cells become GF
independent lead to uncontrolled
proliferation of mature or differentiated cells
(mainly granulocytes, megakaryocytes and
lymphoid cells if pluripotent cells are affected In <5% pt. with BCR-ABL have to be diagnosed
in case of CML) by PCR or FISH

MDS (Immature, Undifferentiated cells ) vs Tumor cell: Pleuripotent Stem Cell (AML or
MPS (Mature, Differentiated non-functional ALL)
cells) Morphology:
1. BM:
Common features of MPD a. Low or normal Erythroid cells
Stages: b. Sea Blue Histiocytes (Macrophages
a. Proliferative Stage: Hypercellular BM with green blue cytoplasm seen in
and Philias/cytosis in PB CML or Neiman picks)
b. Phase of Extramedullary c. High Reticulin (is a type of fiber in
hematopoiesis: Homing of connective tissue composed of type
III collagen secreted by reticular
hematopoietic progenitor cells in
cells)
secondary hemopoietic sites (Spleen,
Liver, LN)
c. Spent/burnt out phase: Marrow
fibrosis (Deposition of reticular
fiber/Type 3 collagen) with
cytopenias
d. Transformation phase: Acute
Leukemias
Keyla Galloza Acevedo

Unique features of certain MPD


-Specific TK mutations help in diagnosis and
targeted therapy
Ej. Systemic Mastosis (abnormal # of mast
cells in the body) KIT TK mutation
Ej.CML BCR-ABL fusion gene (100%)
Ej. PV JAK2 point mutation (>95%)
Sea Blue Histiocyte
2. PB:
a. Marked leukocytosis (TWBC:
100,000)
b. High # of mature Neutrophiles and
its precursors (Myelocytes,
Metamyelocytes, Band forms)

c. High # of Eosinophils and Basophils


(Absolute Basophilia unique
feature of CML)
d. Myeloblast <10% (<20% in BM)
e. High platelets

3. Organs:
a. Spleen: Mild splenomegaly in early
stages due to congestion. Marked
splenomegaly in later stages (2.5-
3.0 Kg)
-Sub-capsular infarcts (Severe
abdominal pain Medical
emergency)
-Normal weight of the spleen is 150
grams
Keyla Galloza Acevedo

b. Liver: Hepatomegaly
c. LN: Lymphadenopathy

Some things to know for the Step:


1. In blunt trauma to the abdomen, the
MC organ damaged is the spleen
2. In a stab wound to abdomen, the MC
organ damaged is the Small Intestine
Clinical features:
1. Age: Common in adults >40yrs (Juvenile
CML can be seen in teens)
2. Onset: insidious (proceeding in a
gradual, subtle way, but with harmful
effects)
3. Anemia: mild to moderate (easy
fatigability, weakness, weight loss)
4. Dragging sensation of left side of
abdomen (Due to marked
splenomegaly)
5. Acute abdomen: Due to splenic infarcts
Lab findings:
1. Karyotyping/PCR demonstrates BCR-
ABL translocations in >95% cases
Course of Disease:
1. Median survival of 3 years
2. 50% develop Accelerated phase (# of
myeloid precursors in BM is increasing
so there is no space to synthesize blood
cells) Worsening Anemia, Low
platelets, High Basophilia
3. If we don’t treat the CML in the next 6-
12 months it could progress to a Blast
crisis 70% AML or 30% ALL
Keyla Galloza Acevedo

IKAROS Transcription factor that regulates


differentiation of hemopoietic progenitors
-Mutation in IKAROS is seen in Lymphoid Blast
Crisis (ALL)
-CML most commonly develops into AML but if
they develop into ALL, those patients will have
IKAROS mutation

How is CML different from other MPD?


-In CML mutations lead to activation of TK
signaling pathways which lead to stronger
proliferation signals
-In other MPD (PV,ET and Myelofribrosis)
mutations in JAK signals are different and
relatively weaker

Treatment:
1. BCR-ABL inhibitors (1st Generation)
-Remission (disappearance) seen in >90% of
cases
-Act on mutated cells and suppresses BCR-
ABL Decreased risk of accelerated CML and
Blast crisis
-Limitations: Can’t eliminate CML tumor/stem
cells and Drug resistance (2nd and 3rd
generation BCR-ABL inhibitors overcome these
issues)
B. HSC transplantation
-Indications: young patients and early stages of
disease (cures 75% of cases)
-Not effective in: Accelerated phases, Blast
crisis and Elderly patients
>First I need to burn the BM to thin the cells
and then induce the stem cells but during this
procedure the pt. could develop cytopenia
which would cause risk of infections and these
certain pt. wouldn’t tolerate it

2. Polycythemia Vera (PV)


Tumor cell: Myeloid Progenitor cell (Can’t
develop into a Lymphoid Leukemia)

Molecular mechanism:
-Point mutations in TK
Keyla Galloza Acevedo

-JAK (activating or gain of function)


Proliferation of all 3 cell lines but specially
Erythroid (RBC)

Types:
1. Primary PV
2. Secondary/Reactive PV
-To differentiate them I need to test blood
levels of Erythropoietin (EPO)
-EPO is also produced by Kidney (80%) and
Liver (20%)
Primary PV
-Low EPO in blood
Clinical features are due to:
1. High Htc Hyper viscosity (Slow
circulation primarily in veins cloths
thrombi below the knee DVT
Pulmonary embolism)
2. Abnormal platelet function (high # but
they are not working)
Bleeding/Thrombi Cerebrovascular
infarcts

Pathogenesis:
-In 97% of cases: Point mutation in JAK2
(Phenylalanine Valine)
-In 30% of cases: 2 mutated copies of JAK2
>Tends to behave like BCR-ABL
>High WBC
>Pruritis (sever itching, Histamine)
>Rapid progression to Spent/Fibrotic phase
(Bad prognosis)

Morphology:
1. BM: Hypercellular in early stages.
Hypocellular in late stages due to
Myelofibrosis (Inc. Reticulin fibers)
2. PB: Inc. in all 3 cell lines but higher inc.
in RBC mass
3. Organomegaly: mild in early stages due
to congestion. Moderate in later stages
due to Extramedullary Hematopoiesis
(EMH-Spleen, Liver, LN)
Keyla Galloza Acevedo

Clinical Features:
1. Increase RBC mass
2. High Htc, Inc. Blood volume
3. Low venous pressure distended
vessels
4. Plethora (excess blood)
5. Cyanosis (sluggish blood flow)
6. Headaches, Dizziness, HTN, GI
symptoms
7. Pruritis and Peptic ulcers (Histamine
from Basophils)
8. Hyperurecemia + 2nd Gout (Big Toe-
Monosodium urate crystals)
9. Bleeding/clotting episodes (abnormal
blood flow and platelet dysfunction
DVC,MI, Strokes, Budd-Chiari (Hepatic
VT + abdominal pain + ascites), Bowel
infarctions (Portal and mesenteric VT),
Epistaxis and gum bleeds (early
findings)

Histamine + H2 = Inc. gastric acid secretion


Histamine + H1 = allergic rx

Labs:
1. Hb% : 14-28 gm/dL
2. Htc >60% (may be low following chronic
bleeding)
3. WBC: 12k to 50k (normal 4k-11k)
4. Platelets: 500k (normal: 100k-150k)
with Giant platelets (functionally
abnormal)
Clinical course:
-If untreated die in months from bleeds and
cloths
-Only Phlebotomy improves life by 10yrs
(process of making a puncture in a vein usually
in the arm, with a cannula, for the purpose of
drawing blood)
-JAK2 Inhibitors

Prognosis:
-Spent/fibrotic phase develops despite Rx in
15-20% of cases by 10yrs
Keyla Galloza Acevedo

-AML transformation in 2% cases (Tumor cells


lack JAK mutations)

3.Essential Thrombocytosis (ET)

Pathogenesis:
a. 50% cases: JAK2 point mutations
b. 5-10% of cases: MPL mutations (TK
activated by Thrombopoietin)
c. 40-45%: Calreticulin mutation
(multifunctional soluble protein that
binds Ca2+ ions (a second messenger in
signal transduction), rendering it
inactive)
-JAK2 mutations and Calreticulin are
mutually exclusive (Both mutations will not
be seen in a single pt)
Clinical:
a. Increase in platelets (functionally
abnormal) Bleeding/Thrombotic
events (like PV)
-No polycythemia or myelofibrosis
-Need to rule out conditions that cause
reactive Thrombocytosis (CML,IDA)

90% of cases of PV have JAK2 mutations


50% of cases of ET have JAK2 mutations

Molecular Mechanism:

MK= Megakaryocyte
No polycythemia but Inc. in abnormal
Megakaryocytes

Morphology:
1. BM: Mild hypercellularity (Inc. number
Keyla Galloza Acevedo

in Megakaryocyte with abnormal


morphology)
2. PB: Inc. # of functionally abnormal
platelets, Giant platelets, Mild
leukocytosis
3. Organomegaly: EMH

Clinical Course:
a. Age >60 (sometimes in young)
b. Bleeding/Clotting complications
c. Spent phase and AML transformation
(uncommon)
d. Erythromelalgia (Throbbing and
burning hands and feet due to
occlusion of small arterioles)
-Can be triggered by an inc. in
temperature (ej. Exercise)
-Also seen in PV
-Seems similar to Sickle cell anemia but
in SSA you usually see ulcers,
autosplenectomy and obstruction
inside the vessel wall
e. Indolent course like PV with long
term remissions
f. Median survival: 10-15yrs
g. Severe Thrombotic events are seen in
pt. with very high PLTS counts & Homozygous
JAK2 mutations

Treatment: Gentle Chemotherapy

4. Myelofibrosis (MF)
Most characteristic feature: Obliterative Bone
Marrow Fibrosis
-Excess of collagen from Non-neoplastic
fibroblast accumulate in the BM and cause
obstruction so cells are obligated to migrate
(EMH)

Molecular mechanism:
a. JAK2 mutation (50-60%, MC)
b. MPL mutations (1-5%)
c. Calreticulin mutations (40-45%)
Keyla Galloza Acevedo

Pathogenesis:

Morphology:
1. BM:
a. Early phase: Hypercellular +
abnormal Megakaryocytes
(enlarged, dysplastic, clustered)
b. Progressive phase: Hypocellular +
Fibrosis + cloud like Megakaryocytes
c. Late phase: Osteosclerosis of the
BM (looks like Spent phase of other
MPD) + homing of hematopietic
cells to EMH sites abnormal RBC
Anemia
2. PB:
a. Leukoerythroblastosis (anemia due
to destruction or crowding out of
hematopoietic tissues in the bone
marrow by a space-occupying
lesion; reduction in normal marrow
cells induces release of immature
hematopoietic cells, especially
nucleated erythrocytes, into the
bloodstream)
b. Tear drop RBC (Dacrocytes:
damaged during birth)
Keyla Galloza Acevedo

c. Basophilia (also seen in PV)


d. Large/Giant Platelets

3. Spleen:
a. Markedly enlarged (4kg)
b. EMH starts in sinusoids and later
extends to cords
c. Cut section look like CML
d. Sub-capsular infarcts (ME)
4. Liver: Enlarged (EMH)
5. LN: Enlarged (EMH)

Clinical Features:
a. Age >60yrs
b. Less common than PV, ET
c. Anemia and Splenomegaly (Dragging of
the L side)-same as CML
d. Loss of weight, night sweats (Inc.
metabolism due to the EMH)
e. Hyperurecemia (Inc. cell turnover)
2nd Gout

-MF, CML, PV have similar features so watch


out
>PV Spent phase: 10yrs, Erythroid
>MF Spent phase: 3yrs, Fibrosis

Lab:
1. Moderate to severe anemia
2. Leukoerythroblastosis
3. Leukocytosis in early stage and
Leukopenia in later stage
4. Initially normal PLTS then
Thrombocytopenia
5. Bone Marrow Evaluation (Both BM
Keyla Galloza Acevedo

Aspiration and Biopsy) BEST AND


MUST INECTIGATION FOR DIAGNOSIS
-Dry Tap
Treatment:
1. Primary MF is difficult to treat (Median
survival is 3-5yrs)
2. HSC transplantation in young and fit
patients (Best option)
3. JAK2 inhibitors
Complications:
a. Infections
b. Thombotic/bleeding episodes
c. Transformation to AML (5-20%)-
Highest of all 3 MPD)
d. AML can develop in EMH sites

Myelofibrosis (Silver stain to see Reticular


fibers)

5.WBC Neoplastic Disorders- Histiocytosis

Langerhans Cell Histiocytosis Palpable LN in Breast cancer:


1. LN metastasis
2. Reactive Histiocytosis
-Group of disorders
-Characteristic feature: Increased dentritic -Need to biopsy LN to differentiate and
cells/macrophages stage (TNM)
-Range from malignant Histiocytic
lymphomas to benign reactive Histiocytosis A.Multifocal Multisystem LCH (Letterer-Siwe
disease)
Keyla Galloza Acevedo

-Not the same as Langhan cells (Epitheloid 1. Age <2yrs


macrophages seen in granulomas) 2. Present with Seborrhic cutaneous
lesions on trunk and scalp (due to LC
infiltration)
3. Hepatosplenomegaly,
Lymphadenopathy, Pulmonary lesions
4. Destructive osteolytic lesions in late
stage
Pathogenesis: BM: Infiltrated with LC
a. Oncogenic mutations affecting BRAF PB: Myelopthesic Anemia (Anemia of
(Valine-Glutamic acid) infiltrative disorder), Thrombocytopenia,
b. BRAF is a Serine/Threonine Kinase otitis media, mastoiditis (bacteria from
located in the signal transduction middle ear infiltrate mastoid air cells)
pathways downstream of RAS -OM: ASOM or CSOM pus can oose out
c. BRAF is also mutated in Hairy Cell and puncture the tympanic membrane
Leukemia leading to Conductive deafness
Morphology: 7. Transform to Langerhan Cell Sarcoma
1. Abundant, vacuolated cytoplasm with anaplastic tumor cells (Highly
2. Vesicular nuclei, nuclear aggressive and fatal if untreated)
grooving/nuclear folds >Treatment: Intensive chemotherapy (5yrs
3. Birbeck granules, cytoplasmic tennis survival)
racket-like (pentalaminar tubules of
Langerin protein) B. Unifocal and Multifocal Unisystem
4. Tumor cells express HLADR, S-100 and (Eosinophilic Granuloma):
CD-1a 1. Langerhans cell with polymorphous
populations of Eosinophils (most
prominent), lymphocytes, plasma cells,
neutrophils
2. Location: Medullary cavities of
calvarium, ribs, femur
-Unisystem: any of the following (skin, lungs,
stomach)

-Unifocal: ej. affects Bones (children and


adults), asymptomatic or with pathological
fractures, spontaneous healing or cure by local
excision or radiation

C.Multifocal Unisystem (Hand Schiller-


Christian disease):
1. Affect young children
2. Multiple bone destructing masses
expand into soft tissues
3. Affect pituitary stalk of hypothalamus
Keyla Galloza Acevedo

in 50% of patients, present with


Diabtetes Insipidus
4. Triad: DI, Bone defects (calvaria),
exophthalmos
5. Prognosis: spontaneous remission or
mild chemotherapy

D.Pulmonary LCH (special type)


1. Adult smokers (regress after stop
smoking)
2. 60% are reactive proliferations
3. 40% present with BRAF mutations

How LC of normal cutaneous distribution


home into storage sites?
a. Normal epidermal Langerhan cells
express CCR-6 (chemokine R)
b. Neoplastic Langerhans cells express
CCR-6 and CCR-7 (migrate to tissues
which express these chemokines)

6. Neoplastic WBC Disorders- Lymphoid Neoplasms 1

Precursor B and T cell General Properties:


Neoplasms A.Diagnosis of Lymphoid neoplasms (BC,TC,NK):
1. Peripheral BC Neoplasm 1. Biopsy and histopathologic examination(HPE) is a
2. Follicular lymphoma must
3. Diffuse large BC
Lymphoma 2. Antigen receptor gene (AgR) rearrangements
4. Burkitts lymphoma (takes place before neoplastic transformation of
5. Plasma cell neoplasms lymphoid cells-All the tumor cells will have same
and related disorders AgR gene sequence(monoclonal) - But in infections
6. Mantle cell lymphoma /inflammations lymphocytes express different
7. Marginal zone lymphoma AgR(polyclonal))
8. Hairy Cell Leukemia
-Every AgR rearrangement leads to unique DNA
Keyla Galloza Acevedo

Peripheral TC, NK Neoplasms sequence and used as monoclonal marker (helps to


1. Peripheral TC lymphoma, identify sub clones of residual tumor cells after
Unspecified treatment)
2. Anaplastic large cell
lymphoma (ALK +) 3. Tumor cells resemble one of the stages of their
3. Adult T cell normal differentiated cells (B, T cells) helps in
leukemia/lymphoma identifying specific features of Leukemias and
4. Mycosis Fungoidosis/ lymphomas
Sezary syndrome
5. Large granular B. Lymphoid neoplasm can cause:
lymphocytic leukemia a. Loss of immunity infections
6. Extra nodal NK/T cell b. Loss self – tolerance Autoimmune diseases
lymphoma -People with inherited /acquired immunodeficiency
disorders have increased risk of lymphoid
Definitions and classifications neoplasms (EBV induced B cell type)
1) Leukemia - Neoplasms
involves bone marrow C. Neoplastic B/T cells home in to their normal
and peripheral blood anatomical sites
2) Lymphoma involves- 1. Ex – follicular lymphomas germinal
Tissue masses (lymph centers of lymph nodes, Cutaneous cell
node/nodal and or lymphomas to skin
extra-nodal) 2. Adhesion molecules and cytokine receptors
3) Leukemia/Lymphoma guide the homing of the cells
describes tissue 3. Neoplastic cells (lymphoid) widely
distribution at the time distributed by the time of diagnosis with
of diagnosis Exceptions:
4) Lymphomas- >Hodgkin’s – contiguous spread
Hodgkin’s Vs NHL >Marginal zone Lymphomas limited to
(Hodgkin's has special site of inflammation
pathological features and
totally different from Precursor B & T cell neoplasms
Non Hodgkin's A. Acute Lymphoblastic leukemia/lymphoma(
Lymphomas- NHL) ALL)
5) Plasma cell neoplasms
mainly arise from bone 1. Neoplastic cell-Lymphoblast (Pre B or Pre
marrow T cell)
6) Site/location-2/3rds of 2. 85% are B-ALL present in young-
NHL and all Hodgkin's childhood or adult Leukemias
cause lymph anode 3. T-ALL seen in adolescent males
enlargement (1/3 of NHL
involve extra-nodal sites-
Skin, stomach, brain)
7) plasma cell neoplasms
Keyla Galloza Acevedo

(multiple myeloma)
presents with bone
destruction lead to pain
and fractures
8) Clinical features are
related to lymphoid
neoplasms are due to -(i)
Tumor secreted proteins
4. Most common cancer of children of USA
-Light chains (BJP-Bence
(less than 15 years age group)
Jones Proteins) in
5. Most common in males and whites(3:1)
multiple myeloma
6. B ALL has highest incidence in less than 3
(ii)cytokines,chemokines
years of age (related to predominance of
secreted by Hodgkin‘s
B cell population in young age group)
and peripheral T cell
7. T ALL is seen in adolescence (with Peak
lymphoma cause fever,
growth of Thymus in this age group)
night sweats
Pathogenesis:
9) Classification(WHO)-is
a. B,T cell developments need expression and
based on molecular
function of transcription factors
diagnostic
b. In 70% of cases chromosomal aberrations
tools(morphologic,
modify these transcription factors
Immunophenotype,
> For T-ALL: Gain of function mutations are
genotype, clinical
seen in NOTCH1
findings)
>For B-ALL: loss of function mutations of
genes PAX-5, E2A,EBF, t(12:21) affect ETV
Lymphoid Neoplasms are
6,RUNX-1
classified into:
>ETV,RUNX-1 required in development of
a. Precursor B cell-
very early hemopoietic precursors
Immature B cells
c. Different mutations - Affect differentiation,
b. Peripheral B cells-Mature
cause maturation arrest, make precursor cells
B cell
behave like stem cells
c. Precursor T cell-
d. Single gene mutations are not sufficient (at
Immature T cell
least less than 10 mutations are needed)
d. Peripheral T/NK-mature
e. MC (>90%) chromosomal change in ALL is
cells
Hyperploidy (more than 50 chromosomes)
e. Hodgkin’s lymphoma
f. Hyperploidy/hypoploidy are only seen in B
with RS cell (Reed
ALL
Sternberg)
g.B,T- ALLs have different translocations,
pathogenesis distinct in each type

Morphology:
a. Marrow: Hypercellular, ↑Lymphoblasts,
↑mitotic rate
b. Thymic mass is seen in > 50% of T ALL, along
Keyla Galloza Acevedo

with splenomegaly and lymphadenopathy


c. Starry sky appearance due to macrophages
engulfing apoptotic cells
Morphology of Myeloblasts and
lymphoblast are very important(given
in table)
Staining (lymphoblast MPO “- ve”,
PAS +ve)

Immunophenotyping:
a. 95% of cases of Pre B, Pre T Lymphoblasts are
TdT Positive (Terminal deoxynucleotide
Transferase)
b. TdT expressed in only in Pre B, Pre T
Lymphoblasts

Clinical Features:
a. ALL, AML are clinically similar with ↑ blasts
in the bone marrow (which suppress
hematopoiesis anemia,
thrombocytopenia)
b. Have abrupt stormy onset, can cause mass
effect (MC in T ALL) lead to bone pains,
lymphadenopathy and Hepatosplenomegaly,
testicular enlargement, compress large
vessels, Air ways of mediastenum
c. CNS symptoms (due to meningial spread
Headache, vomiting, nerve palsies
Prognosis:
1. Paediatric ALL- Excellent prognosis with
chemotherapy (remissions in 95%, cure in
85% of cases)
2. ALL is still leading cause of death in children
- why?

3. Most predictive value is molecular


detection of residual disease
4. B-ALL are associated with BCR - ABL
Keyla Galloza Acevedo

mutations with strong tyrosine kinase activity


of 190KDa (CML – 210 KDa)
5. Treatment with BCR-ABL Kinase inhibitors
6. Adults with ALL have poor results (due to
molecular Pathogenesis and poor tolerance
to treatment)

B. Chronic lymphocytic Leukemia (CLL)/


Small lymphocytic lymphoma (SLL)

1. Difference between CLL and SLL is -number of


peripheral blood Lymphoblasts
2. For CLL absolute lymphocytic count is >
5,000/microlitre (normal count?)
3. Both CLL and SLL are less common in Japan
and Asia
4. CLL is MC leukemia in adults of western
world
>Age more than 60 years and twice as
common in males
5. SLL is 4% of all NHL

Pathogenesis:

1. Chromosomal Translocations are rare


2. Most common genetic abnormalities are
deletion of chromosomes 13,11,17 and
Trisomy 12q
3. Tumor suppressor genes - miR15A miR161
are on chromosome 13
4. Tumor cell - Post germinal centre memory B
cell (Naive B cell in some cases)
5. DNA sequencing helps to find
Hypermutations of Ig gene (which has better
clinical course)
6. Cases with unmutated IG genes are
aggressive
7. Tumor cells are confined to proliferating
germinal centres
8. Stromal cells in the tumor stimulate NK-ƙB (
transcription factor)
9. B cell receptor(membrane bound Ig)
stimulates cascade of kinases - BrutonTK-BTK
which
Keyla Galloza Acevedo

lead to CLL tumor cell growth and survival


>BTK inhibitors are used in treatment of CLL
>BTK defective in X linked
Agammaglobulinemia

Morphology:
a. Lymph Node- Architecture is effaced; tumour
cells are 6-12 microns, small activated
lymphocytes
>Activated Lymphocytes are larger and form
proliferation centre, which are pathogenic of
CLL/SLL
b. Peripheral blood shows disruption of nuclei
of tumour cells, called smudge cells

c. Bone Marrow: Infiltrated with tumour cells


d. Spleen: Tumour infiltrates are seen in red
and white pulp
e. Liver: Infiltration is mainly in Portal tract

Immunophenotyping:
-Tumour cells express Pan B: CD-19, 20, 23, 5 and
low level expression of IgM and IgD
Keyla Galloza Acevedo

Clinical Features:
a. Commonly asymptomatic
b. If symptomatic, they present with easy
fatigability, weight loss, anorexia
c. Generalised lymphadenopathy,
Hepatosplenomegaly
d. White cell count varies (In CLL- more than
200,000/ microlitre, in SLL marrow
involvement leads to Leukemia)
e. Small, monoclonal Ig spike may be seen
f. Both CLL and SLL affect immune function
result in
>Hypogammaglobulinemia which leads to
increased risk of bacterial infection
>Immune Hemolytic Anemia and
Thrombocytopenia
Clinical course:
a. Median survival is 4-6 years
b. Survival is more than 10 years in patients
with minimal tumour burden

Poor prognostic features:


1. Deletions of 11q, 17q
2. Absence of somatic Hypermutations
3. ZAP-70 expression (leads to increased signals
of Ig receptor)
4. NOTCH- 1 mutations
5. High tumour burden

Treatment:
1. Asymptomatic patients: Only observation
2. Symptomatic:
>Chemotherapy with Antibodies against CD-
20
Keyla Galloza Acevedo

>HSC transplantation in young patients


> BTK inhibitors

Most important factor: CLL/SLL transform into


diffuse large B-cell lymphoma in 5-10% of cases,
called Richter syndrome

>Richter syndrome diagnosed by rapidly growing


mass in lymph node and spleen due to additional
mutations,
survival is < one year.

C. Follicular Lymphoma
-MC form of indolent NHL in USA (uncommon in
Europe and rare in Asia)
-Common in middle age, and Male: Female – 1:1

Pathogenesis:

1. Tumor cell – Germinal center B cell


2. Associated with translocations of BCL2 –
t(14:18), BCL2 on 14 and IgH on 18
>BCL2 of Bcl2 family is anti-
mutations leads to survival of tumors cells
(tumor grows by survival rather than
proliferation of cells , Low growth fraction,
less sensitive to chemotherapy)
3.
MLL2 g
abnormal function of Histone methyl
transferase)

Morphology:

1. Nodular or nodulo - diffuse pattern,


2. LN architecture is preserved.
3. Two population cells are present
>Centrocytes – small with scant cytoplasm
with nuclear cleaving
>Centroblasts – large cells, moderate
cytoplasm, several nucleoli
4. Centrocytes predominate
5. Peripheral blood involvement is less
common,
Keyla Galloza Acevedo

6. BM infiltrates are seen in 85% of cases


7. Tumour cells infiltrate
>Splenic white pulp
>hepatic portal tracts

Immunophenotype:
a. Tumor cell express CD 19, 20, 10, surface Ig,
BCL 6 (resemble germinal center B cells)
b. Most important CD5 is absent
(differentiated from CLL/SLL, Mantle C)
c. BCL2 is expressed in 90% of case (normal B
cell are BCL2 negative)
Clinical Features:
1. Lymph Nodes- painless, enlarged, Cut
Section- uniform size follicles with loss of
Zonation,
2. Extra-nodular – mass lesion any where , but
common in waldeyer ring,
3. Liver, Spleen may develop large destructive
masses,
4. Other extra-nodal sites – GIT, skin, bone
brain
5. Bone marrow involvement is late and
uncommon
Treatment:
a. Remission is 70% and, cure in 40-50% with
intensive CT
b. Adjuvant Rx. with anti-CD20 antibodies

SLL CLL

Smudge cells
Keyla Galloza Acevedo

7.Neoplastic WBC Disorders: Lymphoid Neoplasms 2


Burkitts Lymphoma Pathogenesis:
1. MC molecular mechanism: MYC
translocations (Ch.8- TF) t(8;14)
2. Repositioning of MYC close to Ig promoter
and increased expression of MYC
Increased expression of genes of aerobic
glycolysis Warburg metabolism (increased
synthesis of nucleoties, lipids, proteins)
Increased growth and survival of tumor cells
3. Most rapidly growing tumor of humans,
fastest growing tumor of all
Morphology
a. Tumor cell –B cell with 10-25μ, (similar size
as in diffuse type), high mitotic index,
increased number of apoptotic cells

Abnormal
mitosis

b. Macrophages with clear cytoplasm starry


sky
Keyla Galloza Acevedo

c. If marrow is infiltrated tumor cells show


clumped nuclear chromatin with 2-5 nucleoli,
Royal blue cytoplasm with clear cytoplasmic
vacuoles

Immunophenotype:
a. Tumor cell express – IgM, CD 19, 20, 10, BCL6
b. Most important – BCL2 is absent in 100%
cases
Clinical:
1. MC in extra nodal site
2. Very aggressive in growth but excellent
response to Chemotherapy (cure in case of
young people)

Plasma Cell Neoplasms (Plasma Cell A. Myeloma Cell (Multiple Myeloma)


dyscrasias, Monoclonal gammopathy,
Dysproteinemia, Paraproteinemia) 1. Cytokine IL-6 controls proliferation and
survival of tumour cells
1. Tumor cell – plasma cell secrete 2. IL-6 is produced by tumour cell and marrow
monoclonal Ig /Ig fragments stromal cells
(non-neoplastic Polyclonal 3. Increased serum IL-6 levels correlate with
secrete different Igs) active disease and poor prognosis
2. Monoclonal Ig/Ig fragments – are 4. Myeloma derived MIP-1 alpha – Upregulates
tumor markers, important in RANKL (It is an activator of NF-ƙB ligand on
pathogenesis the marrow stromal cells – leads to
3. Cause for 15% of deaths from activation of osteoclasts)
lymphoid neoplasms 5. WNT pathway mediators inhibit osteoblast
4. Multiple myeloma – MC and activity
most dangerous subtype 6. Both lead to Hypercalcemia
5. M component – monoclonal Ig,
high mol. Wt. -160,000, can’t Immunophenotype:
filter through kidneys (not preset a. Tumour cells are CD-138 positive, also known
in urine) as syndecan 1, which is an adhesion molecule
6. Light chains – produced by b. CD-56 to identify small tumour cell clusters
tumor cells (ƙ>ƛ), low mol.
Keyla Galloza Acevedo

weight and present in urine – Clinical Features:


Bence–Jones proteins (BJP) 1. Increased plasma cell tumour growth in
bones
Clinico-pathologic types: 2. Excess production of Immunoglobulins
a. Multiple Myeloma/Plasma cell 3. Suppression of normal Humoral immunity
myeloma – widely distribute in 4. Bones – chronic pain, pathological fractures,
skeletal system, secretes IgG hypercalcemia (confusion, weakness,
b. Plasmacytoma – solitary, in bone lethargy, constipation, polyuria), lead to
or soft tissue renal dysfunction.
c. Smoldering myeloma – high M 5. Decreased immunity – due to decreased
component but asymptomatic normal Ig levels, leads to recurrent bacterial
d. Waldenstrom’s infections, CMI is normal
macroglobulinemia – high levels 6. Renal failure – is called myeloma kidney,
of IgM, hyperviscosity of blood, Causes – multifocal, most important is BJP
older people, associated with which are toxic to tubular epithelial cells.
Lympho-plasmacytic lymphoma BJPs are specific light chains of ƛ6, ƛ3, which
(LPL) lead to amyloidosis (Amyloid light -AL type)
e. Heavy Chain Disease - rare and Lab:
present with LPL, small marginal 1. 99% of cases have increase serum Ig (M-
zone Lymphoma, in component) and BJP in urine
malnourished – Mediterranean 2. Serum Ig causes M-spike in blood and urine,
L, secretes heavy chain diagnosed by immunofixation
fragments 3. Cut off Ig in serum – >3g/dL (IgG >IgA)
f. Primary/immunocyte associated 4. IgA, IgG3 cause hyperviscosity syndromes
Amyloidosis –ƛ chain 5. BJP in urine >6g/dL
prominence, deposit as AL 6. Up to 70% of patients have free light chains
Amyloid (Amyloid-Light Chain) and M-protein, 20% have only free light
g. MGUS – monoclonal chains, 1% are non-secretory
gammopathy of undetermined Diagnosis:
significance – asymptomatic, 1. Bone marrow plasma cell ≥30%
mild to moderate elevated M 2. Peripheral blood Normocytic Normochromic
component, older people, low Anemia, Leukopenia, Thrombocytopenia
but constant procession to MM 3. Median Survival – 4-7 years
h. MM (Plasma cell neoplasm) Poor prognostic factors:
multiple osteolytic lesions , a. Multiple bone lesions
Hypercalcemia, renal failure, b. Deletion of 17p t(4;14)
acquired immune abnormalities, Good prognostic factors:
late phase Lymphoma, a. Smoldering myeloma
visceral organs, 1% cancer b. Cyclin D-1 translocations
deaths in the west, male, African,
>65 years Treatment:
Pathogenesis: a. Proteasome inhibitors
1. Translocations – IgH heavy chain b. Immunomodulators – Thalidomide,
(ch.14q32), results in Lenolidomide (activate ubiquitin ligase,
Keyla Galloza Acevedo

abnormalities of Cyclin D1, Cyclin targeted proteolysis of proteins required by


D3, myeloma
2. Deletion of Ch 17p affects p53 c. Biphosphonates – inhibit bone resorption,
(has poor prognosis) decrease pathologic fractures, inhibit
3. MYC rearrangements – in late hypercalcemia
stage, highly aggressive tumours d. Stem cell transplantation – prolongs life
can lead to plasma cell leukemia
B. Solitary Plasmacytoma

1. 3-5% cases, extra osseous site – lung,


oronasophrynx, nasal sinuses; local excision
cures extra osseous upper respiratory tumors

C. Smoldering myeloma

1. Lies between MM and MGUS


2. Marrow plasma cells; 10-30%
3. Serum “M” component >3mg/dL
4. Asymptomatic
5. 75% progress to MM in 15 yrs.

D. MGUS – Monoclonal Gammopathy of


Undetermined
1. MC Plasma cell Dyscrasia
2. 3% at 50 yrs, 5% at 70s
3. Asymptomatic,
4. Serum “M’ > 3mg/dL
5. 1% develop MM every year (constant
conversion rate)
6. Molecular abnormalities – translocations
and deletions are same as in MM – MGUS is
early stage of MM
7. Progression is unpredictable,
8. Follow –up with Serum “M” protein, Urine
BJP

E. Lympho Plasmacytic Lymphoma

1. B cell neoplasm of 6-7th decades,


superficially resembles CLL/SLL, but there is
presence of plasma cells
2. Monoclonal “M” from plasma cell
Hyperviscosity syndrome (Waldenstrom's
Macroglobulinemia)
Keyla Galloza Acevedo

3. Complications of light chains and Bone


lesions – absent
Pathogenesis
a. MYD88 mutations (adapter protein
important in activity of NK-ƙB factor &
downstream signals of B-cell receptor -Ig
complex) lead to growth and survival of
tumor cells
Morphology
b. Marrow – Lymphocytes, Plasma Cells,
Plasmacytoid cells, and mast cell hyperplasia
c. Tumor plasma cells – PAS positive
cytoplasmic RUSSEL bodies, nuclear
DUTCHER bodies

Immunophenotype
d. Lymphoid component – CD 20 +ve, surface Ig
+ve,
e. Plasma cells – secrete IgM

Clinical features
f. Nonspecific – weakness, fatigue, weight loss
g. 50% with lymphadenopathy and
splenomegaly
Signs:
1. Myelophthesic anemia and IHA (Cold
Agglutinin IgM) in 10% of cases
2. Hyperviscosity syndrome - IgM secreting,
present with visual impairment, (retinal
venous congestion, tortuousity,
haemorrhages and exudates)
3. Neurological: Headaches, Dizziness,
Deafness, Stupor (Due to sluggish blood
flow), Bleeding – Formation of complexes
between macroglobulins and clotting factors,
Keyla Galloza Acevedo

abnormal Platelets, cryonglobulinemia –


Precipitates of macroglobulins at low
temperatures lead to Reynaud phenomena
and cold urticaria.
Clinical course:
a. Incurable and progressive
b. Most of the symptoms related to
intravascular IgM – Plasmapheresis improves
symptoms
c. Transforms to large cell lymphoma
d. Median survival is 4 years

Punched out-osteolytic lesions

Neoplastic plasma cells

F. Mantle CELL LYMPHOMA


- 2.5% of NHL (uncommon in USA)
-5-6th decade, common in male
-Tumour cells resemble B-cells of mantle zone
(Outside germinal centre)

Pathogenesis:
1. All tumour cells have t(11;14) which affects
IgH locus of ch. 14; lead to Cyclin D-1 over
expression on ch.11
2. 70% of cases diagnosed by karyotyping,
100% by FISH
Keyla Galloza Acevedo

3. Cyclin D-1 acts at G1-S phase in the cell cycle,


leads to proliferation of tumour cells

Morphology:
1. MC is generalised lymphadenopathy, 50%
have spleen, liver, marrow, gut
2. Polypoidal lesions in the intestine are called
lymphoid polyposis (unique for mantle cell
lymphoma)

Homogenous Small Lymphoid cells

3. Nodular appearance in low power and


homogenous small lymphocytes with nuclear
cleaving in high power
4. Centroblasts are absent (helps to
differentiate from Follicular lymphoma and
CLL/SLL)
5. Blastoid variant – intermediate size cells
with open chromatin and high mitosis

Immunophenotype:

Clinical Features:
1. Most common is painless lymphadenopathy
Keyla Galloza Acevedo

Atrophic Germinal Center

2. Symptoms related to spleen and gut


3. Media survival (3-4 years)
4. Incurable
5. Tumour cells infiltrate and lead to organ
dysfunction
6. Blastoid infiltrates have poor prognosis
Treatment:
a. HSC transplantation
b. Proteasome inhibitors

G. Marginal Zone Lymphoma

- Heterogeneous group with nodal and extranodal


expression
-Extranodal also known as MALTomas
-Somatic Hypermutations indicate the tumour cell is
of memory B-cell origin

Characterisics:
1. Arise at sites of chronic infection or
autoimmune background- -Sjögren's (salivary
gland), Hashimoto’s (Thyroid), MALToma
(Stomach)
2. Tumour localised for long term
3. Regress with eradicating underlying cause
Pathogenesis:
1. In extra nodal sites, lymphoma arises as
continuation of reactive hyperplasia to
lymphoma, (unknown mechanism)
2. In early stages, B-cell proliferation is
dependent on T-helper cells and
withdrawal of antigen leads to involution
Keyla Galloza Acevedo

of the tumour
>Best example is regression of MALToma
by treating H. Pylori
3. After additional mutations, like t(14;18);
leads to antigen independent tumour
cells, which has a poor response to
treatment
4. Translocations upregulate BCL10/MALT-1
which activates NK-ƙB, which leads to
a. Clonal evolution, which spreads to
distant sites, transform to diffuse
large B-cell lymphoma.
b. 2. Similar molecular abnormalities are
also seen in EBV induced lymphomas

H. Hairy Cell Leukemia

-Rare, 2% of all Leukemias


-Middle age (55y), white, common in male (M:F =
5:1)
Pathogenesis:
1. More than 90% of cases - BRAF activation
point mutations at 600, Valine to Glutamate
2. BRAF is a Serine/Threonine Kinase, acts at
juxtaposition to RAS in MAPK pathway
3. BRAF mutations are also seen in Langerhan
cell Histiocytosis, Melanomas
Morphology:
1. Tumour cells show fine, hair-like processes,
well demonstrated in phace-contrast
microscope
2. Nuclei are round, oblong, reniform, with pale
blue cytoplasm having thread-like or bleb-
like extension
Keyla Galloza Acevedo

Marrow:
a. Show more hairy cells, enmeshed by ECM,
which leads to entrapment in the marrow
b. Result in bone marrow aspirate which is dry
tap
c. Spleen white pulp is obliterated and looks
red, beefy
d. Liver – hairy cells infiltrate portal tract

Immunophenotype:

Clinical features:
a. Most common is massive splenomegaly,
liver less common, lymph nodes rare
b. Peripheral smear – Pancytopenia,
unexplained Monocytopenia

Clinical course:
1. 1/3rd of patients have atypical
mycobacterial infections
2. Indolent with excellent response with mild
chemotherapy
3. Relapses after 5 years are common, but still
respond with chemotherapy

Peripheral TC/NK cell Neoplasms A. Peripheral T cell Lymphoma, unspecified

1. Introduction – heterogeneous 1. Not easily categorised, diffuse type with loss


group, resemble mature T/NK of LN architecture, Tumour cell-
cells , T cell tumor 5-10% of pleoromorphic T cell, along with Eosinophils
NHL in USA and Europe, NK and macrophages, increased neo-
Keyla Galloza Acevedo

tumors rare, angiogenesis


2. Both T/NK cells are common in 2. Markers – CD 2, 3, 5, either ƛβ, or δ TCR, also
Eastern countries 4, 8. Many tumour cells look like mature cells
– require DNA analysis

Morphology – Generalised Lymphadenopathy,


Eosinophilia, Pruritus, fever, weight loss

Prognosis – worse than B cell tumors

Mixed populated population of small, intermediate


and large Lymphoid cells

B. Anaplastic Large cell Lymphoma (ALK


positive)

1. Uncommon, ALK gene


rearrangements(ch.2q23) lead to break in
ALK to form chimeric gene – results in
activation of TK, RAS, JAK/STAT (like CML but
not BCR-ABL)
Morphology – tumor cells – large , anaplastic , most
abundant cytoplasm, horseshoe like nuclei
(HALLMARK cell), infiltrate LN sinuses (resemble
Mets.)
Keyla Galloza Acevedo

ALK is expressed only Tumor cells – very important

IHC + ALK

Clinical course & Prognosis – seen in children and


young adults, Mc in soft tissues very good prognosis
(cure rate – 75%), older people – poor prognosis

Both ALK + and ALK – cells express CD30 (TNF


receptor family)

C. Adult T cell Leukemia/Lymphoma

CD4+ Tumor cells, only seen in adults with HTLV-1


infection, common in Japan, West Africa and
Caribbean

Clinical Features:
a. Skin lesions
b. Generalised lymphadenopathy
c. Hepatosplenomegaly
d. Hypercalcemia
e. Peripheral blood – Lymphocytosis
Keyla Galloza Acevedo

Morphology:
1. Tumor cells have multilobated nuclei –
“CLOVE LEAF or FLOWER” cells, TC contain
HTLV proviruses , Tax – viral encoded
protein activated NK-ƙ B
Clinical course:
1. fatal, death within 1 yr, some cases with
localised skin lesions – indolent course,
2. HTLV-1 causes non-neoplastic progressive
demyelininating disease of CNS

D. Mycosis Fungoidosis (MF) /Seary syndrome

a. Involves epidermis, upper dermis, tumor cells


have cerebriform nuclei (convolutions or
folding)
b. Markers – responsible homing of tumors
cells to skin – CCR4, CCR10

Clinical course
a. indolent with survival -8-9 yrs.
b. Rarely develop T cell Leukemia/lymphoma

E. Large Granular Lymphocytic Leukemia

1. 35% cases – STAT3 (TF) mutations lead to


cytokine independent STAT3 signalling.

Morphology – Tumor cells – large lymphocytes with


abundant blue cytoplasm, few coarse azuriphilic
cytoplasmic granules, Marrow – Tumour cell
infiltrates scatter (difficult to identify), Spleen, Liver
– show Tumour cell infiltrates

Clinica Features:
a. Anemia and Neutropenia
b. Marrow has marked decrease in late
precursors of myeloid series
c. Rarely – pure red cell aphasia
d. Sometimes associated with rheumatoid
conditions “FELTY Syndrome” – RA,
Splenomegaly, Neutropenia (may progress
to Leukemia)
Keyla Galloza Acevedo

Extra nodal T/NK Cell Neoplasms


A. Diffuse Large BC Lymphoma (DLBCL)
a. Rare in USA (common in Asia)
b. MC present with destructive 1. MC NHL in USA, in Elderly males
nasopharyngeal mass, also 2. MC molecular abnormality is over-
involve testis, skin expression of deregulated BCL6 {others –
c. Tumor cells compress and invade MYC, t(14:18) are uncommon}
vessels – Ischemic necrosis
d. Large Azurophilic cytoplasmic Morphology:
granules are seen (like NK cells) a. Large tumor cells, abundant cytoplasm,
e. Associated EBV and all tumor Vesicular nuclei with prominent nucleoli
cells will have EBV episomes b. Lymph node architecture is effaced
(100% association) but EBV c. Anaplastic cells- multinucleated with
infected tumor cell lack CD21? nucleolar inclusions (like RS cells)
f. Markers T cells – CD3, TCR, NK -
16, 56
g. Highly aggressive but respond
well to RT (not CT)

Immunophenotype:
a. B cell markers (CD 19, 20, 10, and BCL6,
surface Ig)
Special types:
1. Immunodeficiency type – seen in patients
with severe immunodeficiency (HIV-AIDS, BM
transplantation recipients) – EBV infected B
cell
2. Primary Effusion Lymphoma – Advanced HIV
cases, elderly, present malignant pleural
effusion all tumors cells have KSHV/HHV8
viral genome and lack B and T cell markers
3. Nodal and Extra nodal – anywhere (marrow
involvement is uncommon, late)

Treatment:
a. Intensive Chemotherapy – 70% remission,
Keyla Galloza Acevedo

50% cure)
Prognosis:
a. Wide spread and MYC translocations carry
poor prognosis

B. Hodgkin’s Lymphoma

1. Lymph nodal involvement/spread -


contiguous
2. Diagnostic cell – Reed – Sternberg cell (RS
cell) – derived from germinal or post-
germinal center B-cell.
3. Common in USA (8000) cases per year
4. Age- 32 years
5. First human cancer successfully treated with
radiotherapy and chemotherapy

Molecular abnormality – most common is activation


of NF-KB by EBV,
>EBV expresses latent membrane protein 1 (LMP-1)
which activates NF-KB.
Keyla Galloza Acevedo

RS cells secrete a variety of cytokines and attract


reactive cells like lymphocytes, plasma cells,
Eosinophils and Monocytes, RS cells are aneuploid,
and have REL proto-oncogene changes, which result
in activation of NF-KB, RS cells are infected with EBV
in 90% of cases

Treatment:
a. Radiotherapy and Chemotherapy give
excellent results
b. Late complications: Secondary malignancy of
lung, breast, skin (Melanoma), and AML
(mainly M6)
Keyla Galloza Acevedo

Nodular sclerosis

Mixed cellularity

Myeloid Neoplasms

Pathogenesis:
a. Most common – t(8;21)
disruption RUNX-1, inv(16)
disrupts CBF-B gene

b. Deep DNA sequencing shows


epigenetic alterations like DNA
methylation, post translational
modifications of Histone
proteins, which are central and
critical.

Morhology:
a. Myeloblasts description – (refer
table slide 20 of ppt-5)
b. Aleukemic Leukemia absence
of blasts in the peripheral blood

Clinical Features:
a. Pancytopenia
b. APML or AML M3 presents with
DIC
c. Infections with fungi,
pseudomonas, other
commensals (due to immune
deficiency)
Keyla Galloza Acevedo

d. Localised soft masses are also


called myeloblastoma,
granulocytic sarcoma or
chloroma

Prognosis:
1. Difficult to treat, except AML M3
or APML with t(15;17)
Keyla Galloza Acevedo
Keyla Galloza Acevedo
Keyla Galloza Acevedo

Dr. Tadepalli Review Notes

WBC
Non- Neoplastic Disorders
Penias:
Most common and most important: Neutropoenia
Most severe form is Agranulocytosis – Prone for life threatening
bacterial and fungal infections (Severe Neutropenia where the
absolute Neutrophil count is <100)
Postman Syndrome: Inherited form of Neutropenia due to
defective genes of Granulocyte differentiation
Keyla Galloza Acevedo

Features of Hypersplenism – Any condition which can cause


marked Splenomegaly, Peripheral cytopenias – Marrow
compensatory hypercellularity – removal of spleen reverts both
marrow and peripheral blood abnormality
Most common cause of Neutropenia is drug toxicity (after drug
the second MCC is Viral infections)
Large Granular Lymphocyte Leukaemia also presents with severe
Neutropenia
Necrotising, ulcerative lesions of oral cavity is Agranulocytic
Angina (pain in oral cavity)
Severe, exuberant bacterial growth which forms colonies is
called botryomycosis, is seen in Neutropenia
Lymphopenia is expected in infections, HIV, drugs –
corticosteroids and cytotoxic drugs, autoimmune disorders,
malnutrition (Most Common Cause)
Philias:
Also called Leucocytosis
Neutrophilic Leucocytosis is seen in acute bacterial infections,
fungal and acute inflammations (ej. Accute appendicitis or
cholecystitis)
Eosinophilic Leucocytosis mainly seen in parasitic infestations,
allergies like bronchial asthma, drug reactions, Hodgkin’s
Lymphoma.
Basophilic Leucocytosis is rare, mainly seen in MPDs (CML
most common)
Monocytosis is seen in chronic infections like Tuberculosis and
Lymphocytosis , seen mainly in viral infections (EBV – Infectious
Mononucleosis. These Lymphocytes are called reactive or
transformed Lymphocytes), Pertussis.
Keyla Galloza Acevedo

The most common mechanism of reactive Neutrophilia is due


to immediate release from marrow storage pool, by action of
IL-1 and TNF- alpha
The cytokine for Eosinophilia is IL-5
Toxic Granulations and Dohl bodies are seen in Neutrophils or
acute infection or inflammations like Kawasaki’s.
Table to differentiate between Leukaemia and Leukemoid
Reaction (very high yield):

Lymph Node:
Most common causes of acute non-specific Lymphadenitis are
infections
Tingible body macrophages (undigested necrotic material inside
macrophages) are characteristic, but not diagnostic of acute
Lymphadenitis (Slide 31 – important)
Features to differentiate Lymphadenitis from Follicular Lymphoma
are: size of the follicles, zonation – Light and Dark regions, and
capsular invasion, but the most important differentiating feature
is mutations of BCL2 (t 14:18)
Paracortical Hyperplasia (TC activation) is one of the
morphological types of chronic Lymphadenitis, mainly due to viral
infections, vaccines, medications like Phenytoin (Dialantin-anti
seizure/epileptic) , shows T-Lymphocyte precursors
Sinus histiocytosis is a type of chronic Lymphadenitis seen in
patients with cancers (It is not because of tumor infiltration)

Hemophagocytic Lymphocyto- Histiocytosis (HLH):


Keyla Galloza Acevedo

Cells responsible are – macrophages and CTLs


Also called Macrophage activation syndrome
Familial type is seen in young infants
Main cytokines responsible are: Interferon-gamma, TNF-alpha, IL-
6, IL-12, IL-2 receptor.
Cytokines produce SIRS or Cytokine storm
Most common trigger for HLH is infection, especially EBV
Important, but not diagnostic finding is Hemophagocytosis.
Treatment for familial cases is HSC transplantation (Slide 42 -
Image)

Neoplastic Disorders
Most common abnormality is translocations (In AML-M3, t15;17
translocation leads to mutation of Retinoic acid receptors)
AID (Activation Inducted cytokine Deaminase) leads to B-cell
lymphomas through misdirected BCL6 gene.
Recombinations of V(D)J recombinase leads to precursor T-cell
tumours
Examples of Inherited genetic instability disorders: Blooms,
Fanconi’s Anemia, Ataxia Telangiectasia, Xeroderma Pigmentosa
Genetic disorders with increased risk of childhood Leukaemia is
Down’s and NF-1.
Viruses causing neoplastic disorders are HTLV- 1 (Adult T-cell
Leukaemia/Lymphoma), EBV (Burkitt’s, Hodgkin’s, NHL, and
Nasopharyngeal carcinoma (100% association)
KSHV/HHV-8 in patients with HIV infections cause B-cell
Lymphomas
Keyla Galloza Acevedo

Chronic Inflammations due to H. Pylori [Gastric B-cell Lymphoma,


Gluten sensitive enteropathy (T-cell Lymphoma)]
Smoking doubles the risk of AML

MDS:

Clonal stem cell disorder affecting multipotent myeloid stem


cells, which produce abnormal and ineffective blood cells.
Presents with ineffective haematopoiesis and peripheral
cytopenias
Increases risk of Acute Leukaemia (AML)
t-MDS has highest risk of AML transformation, as compared to
Primary type.
Refractory Anemia is the one which does not respond to
Haematinics even after 6 months
Pseudo Pelger-Huet Neutrophils (Bilobed) are seen in peripheral
blood
Isolated mutations, that is, the addition of 5q, respond to
Thalidomide
Pawn-ball megakaryocytes (Slide 24) in bone marrow
Ring Sideroblasts
Tables: Slide 32, 36 important

MPD:

Most common mutations are in Tyrosine Kinase/Signal Pathways


which lead to gain-of-function mutations
Keyla Galloza Acevedo

CML:

BCR-ABL (t9;22) is seen in all patients of CML and chromosome is


known as Philadelphia Chromosome
BCR-ABL fusion gene has Tyrosine Kinase activity
The tumour cell in CML is pluripotent stem cell which can
produce both Acute Myeloid and Lymphoid Leukaemia
Bone Marrow shows sea blue histiocytes
Peripheral blood shows marked Leucocytosis and absolute
Basophilia
Slide 13, 14
Massive splenomegaly causes dragging sensation on left side of
abdomen
IKAROS – transcription that regulates differentiation of
hemopoietic precursors. If it undergoes mutation, it leads to ALL.
CML is different from other MPDs in: 1. mutations of Tyrosine
Kinase, 2. Tumour cell is pluripotent stem cell, 3. Can produce
both AML and ALL.
Features of accelerated phase in CML are worsening Anemia,
Thrombocytopenia and increased Absolute Basophilia
PCV:
Tumour cell- Myeloid progenitor cell
Significant clinical features are due to abnormal features of
erythroid series
Blood levels of erythropoietin are important to differentiate
between Primary (low EPO) and Secondary PCV (High)
Most common molecular abnormality is point mutations of
JAK2
Keyla Galloza Acevedo

Double mutated copies of JAK2 lead to PCV with CML like


features (Splenomegaly and Marked leucocytosis)
Pruritus (Marked itching), Peptic ulcers are due to histamine
from Basophils
Hyperuricemia leads to secondary gout, due to high cell
turnover
2% of PCV transform to AML and Tumour cells lack JAK2
mutations
ET

Common mutations are JAK2 (50%), MPL (Thrombopoietin


receptor), Calreticulin in remaining
Polycythaemia and Myelofibrosis are absent in ET
JAK, MPL mutations make tumour cells (Megakaryocyte
progenitors) thrombopoietin independent
Erythromelalgia is a feature seen in ET as well as PCV

MF

Most important finding is obliterative marrow fibrosis and most


common mutations are JAK2.
Tumour cell is neoplastic Megakaryocytes but fibrosis is Non-
Neoplastic Fibroblasts
Nuclei of Megakaryocytes have a cloud-like irregular pattern
Best and most important diagnostic test in Bone Marrow biopsy
AML transformation is 5-20% (Slide 50)
Cerebral Edema Hydrocephalus
Condition Cerebral Edema Hydrocephalus Herniation

Overview Cerebral edema is when fluid builds up around the brain, causing an increase in pressure known as intracranial Hydrocephalus refers to the accumulation of excessive CSF within the ventricular system. When the volume of tissue and fluid inside the skull increases beyond the limit permitted by compression
pressure. Swelling or inflammation is part of the body's natural response to injury.
This disorder most often is a consequence of impaired flow or resorption; over- of veins and displacement of CSF, intracranial pressure rises. The cranial vault is subdivided by rigid dural
production of CSF, typically seen with tumors of the choroid plexus, only rarely causes folds (falx and tentorium), and a focal expansion of the brain dis- places it in relation to these partitions. If
hydrocephalus. If there is a localized obstacle to CSF flow within the ventricular system, the expansion is sufficiently large, herniation occurs. Herniation often leads to “pinching” and vascular
then a portion of the ventricles enlarges while the remainder does not. This pattern is compromise of the compressed tissue, producing infarction, additional swelling, and further herniation.
referred to as noncom- municating hydrocephalus and most commonly is caused by masses
obstructing the foramen of Monro or compressing the cerebral aqueduct. In communicating
hydrocephalus, the entire ventricular system is enlarged; it is usually caused by reduced
CSF resorption.
Pathophsyiology Vasogenic edema occurs when the integrity of the normal blood-brain barrier is Hydrocephalus is the accumulation of excessive CSF within the ventricular -Displacement of brain tissue past rigid dural folds (THE FALX AND
disrupted, allowing fluid to shift from the vascular compartment into the system. Usually impaired flow and resorption of the fluid.

extracellular spaces of the brain. Vasogenic edema can be either local- ized (e.g., 
 TENTORIUM) or through openings in the skull bc of increased inter cranial
increased vascular permeability due to inflam- mation or in tumors) or generalized. -Overproduction may be caused by tumour in choroid plexus 
 pressure.


 

Cytotoxic edema is an increase in intracellular fluid secondary to neuronal and glial 

cell membrane injury, as might follow generalized hypoxic-ischemic insult or after
If hydrocephalus develops in infancy before closure of the cranial sutures, the As volume of brain increases, CSF is displaced and the vasculature is
exposure to some toxins.
head enlarges. Once the sutures fuse, hydrocephalus causes ventricular compressed, leading to increasing pressure within the cranial cavity

The edematous brain is softer than normal and often appears to “over fill” the cranial
expansion and increased intracranial pressure, but no change in head cir- 

vault. In generalized edema the gyri are flattened, the intervening sulci are narrowed, cumference. In contrast with these states, in which increased CSF volume is
and the ventricular cavities are compressed the primary process, a compensatory increase in CSF volume can also follow When the increase is beyond the limit permitted by compression of veins
the loss of brain parenchyma (hydrocephalus ex vacuo), as after infarcts or with and displacement of CSF, tissue herniates between compartments across
degenerative diseases.
the pressure gradient

Symptoms/
Subfalcine (cingulate) herniation occurs when unilateral or asymmetric
Clinical features
Hydrocephalus ex vacuo- Increase in expansion of a cerebral hemisphere displaces the cingulate gyrus under the edge
ventricular volume secondary to a loss of falx. This may be associated with compression of the anterior cerebral artery.
Higher cerebellar dysfunction (dysphasia, dyscalculia). Visuaospatial disorder.
of brain parenchyma Ipsilateral motor and sensory deficits

Transtentorial (uncinate) herniation occurs when the medial aspect of the


temporal lobe is compressed against the free margin of the tentorium. As the
temporal lobe is displaced, the third cranial nerve is compromised, resulting in
pupillary dilation and impaired ocular movements on the side of the lesion
(“blown pupil”). The posterior cerebral artery may also be compressed, resulting
in ischemic injury to tissue supplied by that vessel, including the primary visual
cortex. If the amount of displaced temporal lobe is large enough, the pressure on
the midbrain can compress the contralateral cerebral peduncle against the
tentorium, resulting in hemiparesis ipsilateral to the side of the herniation (a
so-called false localizing sign). The compression of the peduncle creates a
deformation known as Kernohan’s notch. Progression of transtentorial herniation
is often accompanied by linear or flame-shaped hemorrhages in the midbrain and
pons, termed Duret hemorrhages. These lesions usually occur in the midline and
paramedian regions and are believed to be the result of tearing of penetrating
veins and arteries supplying the upper brain stem.
Tonsillar herniation refers to displacement of the cerebellar tonsils through the
foramen magnum. This type of herniation is life-threatening, because it causes
brain stem compression and compromises vital respiratory and cardiac centers
in the medulla.

Lab diagnostics
Imaging
Table 1
Condition Spina Bifida Meningocele Encephalocele Arnold-Chiari malformation Neural tube defect general
Overview Spina bifida occulta is when a baby's backbone (spine) does A meningocele is a birth defect where there is a sac protruding from the spinal column. The Encephalocele is a sac-like protrusion or projection of the A Chiari malformation, previously called an Arnold-Chiari On of the earliest steps in brain development is the formation of the neural tube,
not fully form during pregnancy. The baby is born with a small sac includes spinal fluid, but does not contain neural tissue. It may be covered with skin or brain and the membranes that cover it through an opening malformation, is where the lower part of the brain pushes
gap in the bones of the spine. Spina bifida occulta is common with meninges (the membranes that cover the central nervous system).
 in the skull. Encephalocele happens when the neural tube down into the spinal canal. There are 4 main types, but
which gives rise to the ventricular system, brain and spinal cord. Partial failure
and happens in about 1 out of 10 people. Usually, spina bifida 
 does not close completely during pregnancy. type 1, called Chiari I, is the most common. In someone or reversal of neural tube closure may lead to one of several malformations, each
occulta causes no health problems. Meningomyelocele: Protrusion of the membranes that cover the spine but some of the with Chiari I, the lowest part of the back of the brain characterized by abnormalities involving some combination of neural tissue,
spinal cord itself through a defect in the bony encasement of the vertebral column. The bony extends into the spinal canal.
defect is spina bifida.
meninges, and overlying bone or soft tissues.

Collectively, neural tube defects constitute the most frequent type of CNS
malformation. The overall recurrence risk in subsequent pregnancies is 4% to
5%, suggesting a genetic component. Folate deficiency during the initial weeks
of gestation also increases risk through uncertain mechanisms; of clinical
importance, pre- natal vitamins containing folate can reduce the risk of neural
tube defects by up to 70%. The combination of imaging studies and maternal
screening for elevated α-fetoprotein has increased the early detection of neural
tube defects.
Pathophsyiology -Local regions of the spinal neural tube are infused or there is Myelomeningocele is an extension of CNS tissue through a defect in the -Diverticulum of malformed brain tissue extending through -Underdevelopment of the occiput

failure of formation of the vertebral neural arches
 a defect in the cranium
 


vertebral column that occurs most commonly in the lumbosacral 
 -Usually there is obstruction to passage of CSF at the
-Usually lumbosacral region
 region. Patients have motor and sensory defi- cits in the lower -Most often occurs in the posterior fossa level of Forman magnum

extremities and problems with bowel and bladder control. The clinical 

Type 1: Cerebellar tonsillar herniation without
problems derive from the abnormal spinal cord segment and often are displacement of medulla

compounded by infections extending from the thin or ulcerated 

overly- ing skin. Type 2: Caudal displacement of the medulla


Type 3: Encephalocele and breaking of the quadrigeminal
plate (associated with syringomyelia)


Symptoms/ May see dimple or tuft of hair or skin overlying L5-S1
 -Generally involves Lumbosacral regions
Clinical features 

-Bowel and bladder abnormalities, foot drop
Lab diagnostics
Imaging

Predisposing
factors
Treatments
Condition
ETC
Table 1

Diffuse astrocytoma Fibrillary Gemistocytic Pilocyctic Oligodendroglioma Pleomorphic Brainstem Gliomas Glioblastomas Medulloblastoma
Overview

astrocytoma astrocytoma Astrocytoma Xanthoastrocytoma


Pathophysiology/ Infiltration beyond outer margins is always present
 Intrincic pontine gliomas- Aggressive -May follow astrocytoma 

Usually in 3rd ventricle or Anaplastic oligodendroglioma-
-Occurs in cerebellum
Variations

 -Arise from cerebral -May progress to -Superficial location on cerebrum
course and short survival
 

cerebellum. 
 Densley cellular, nuclear (temporal lobe) Gliomatosis cerebri- Is a diffuse glioma with extensive infiltration of multiple regions of the
Behave as glioblastomas 
 glioblastoma 


Anaplastic Astrocytoma- Densely cellular, Nuclear
hemispheres 
 pleomorphism, mitotic figures, Cervicomedullary junction tumors- Often
brain, in some cases the entire brain


exophytic, with a less aggressive course
 Variation in the appearance of the tumor from region to region. Firm and white to yellow and
pleomorphism, mitotic figures are often observed


May appear along optic necrosis 
 cystic


Dorsally exophytic gliomas- an even more
nerves benign course which may arise in the
Necrosis in serpentine fashion along areas of hypercellularity . Tumour cells collect along
edges of the necrotic regions- 

tectum of the midbrain, pons or medulla 

Pseudopalisades - Neoplastic vascular cell proliferation characterized by tufts lined by double
layer of endothelial cells


Glomeruloid body may be formed 


Symptoms/ -Poorly defined grey infiltrative tumours that expand and -MC in children -Found in temporal lobes of children and young adults 

Clinical features
distort the invaded brain 

-History of seizures

Lab diagnostics
-Increase cell density
 -The predominant neoplastic -May be found with NF1 -Sheets of regular cells with spherical -Neoplastic occasionally bizarre astrocytes Neural class

Neural markers: NEFL, GABRA1, SYT1, SCL12A5

-Variable degrees of pleomorphism 
 nuclei with fine chromatin surrounded by a which are sometimes filled with lipids 

astrocyte has bright mutations 
 

-GFAP+ astrocytic processes between clear halo of Vacuolated cytoplasm (Fried 
 Proneural class

tumor cell nuclei -create a eosinophils cell body from -GFAP(+) pili
 -Abundance of rectulin deposits, relative
IDH1 mutations and PDGFR amplifications 


-Rosenthal Fibres
 egg appearance)
 



fibrillary background 
 which emanate abundant circumscription, and chronic inflammatory cell Mesenchymal Class


-P53 and PDGF-A over expression stout processes
 -Eosinophilic Granular bodies
 infiltrates, along with the absence of NF1 mutations 

TNF and NF-kB highly expressed

-Tumour cells may satellite neurons necrosis and mitotic activity, distinguish this

 -Tumour is Biphasic 

Classical Class

tumor from more malignant types
PTEN mutation

EGFR

CDKN2A


Imaging

Histology

Demarcating Fibrin (pink fibre) background Nuclei compressed with bright pink eosinophilicc body Rosenthal fibers 
 Pseudopallisading necrosis
feature Fired egg appearnce

Treatments

Condition

ETC

You might also like